SlideShare a Scribd company logo
1 of 160
Download to read offline
WWW.VNMATH.COM 
SÐ GIO DÖC V€ €O T„O 
THANH HÂA 
 
· Ch½nh Thùc 
· thi gçm câ 01 trang 
Kœ THI V€O LÎP 10 CHUY–N LAM SÌN 
N‹M HÅC 2012 - 2013 
 
Mæn: To¡n (chuy¶n) 
(Dòng cho th½ sinh thi v o chuy¶n To¡n) 
Thíi gian l m b i: 150 phót (khæng kº thíi gian giao ·) 
Ng y thi: 18 th¡ng 6 n«m 2012 
C¥u 1: (2,0 iºm) 
Cho a = x + 
1 
x 
; b = y + 
1 
y 
; c = xy + 
1 
xy 
vîi c¡c sè thüc x,y thäa m¢n xy6= 0 
T½nh gi¡ trà biºu thùc: A = a2 + b2 + c2  abc 
C¥u 2: (2,0 iºm) 
Cho ph÷ìng tr¼nh (x  1)(x  2)(x  3)(x  6) = mx2 (m l  tham sè). 
Gi£ sû m nhªn c¡c gi¡ trà sao cho ph÷ìng tr¼nh câ 4 nghi»m x1; x2; x3; x4 ·u kh¡c 0. 
Chùng minh r¬ng biºu thùc P = 
1 
x1 
+ 
1 
x2 
+ 
1 
x3 
+ 
1 
x4 
khæng phö thuëc m. 
C¥u 3: (2,0 iºm) 
T¼m sè nguy¶n d÷ìng n sao cho 
n(2n  1) 
26 
l  sè ch½nh ph÷ìng. 
C¥u 4: (3,0 iºm) 
1) Cho tam gi¡c ABC vuæng t¤i A, ÷íng cao AH. Gåi (I), (K) l¦n l÷ñt l  c¡c ÷íng trán nëi 
ti¸p tam gi¡c ABH, ACH. ÷íng th¯ng KI c­t c¤nh AB t¤i M v  c¤nh AC t¤i N. 
a) Chùng minh 
HI 
HK 
= 
HB 
HA 
. 
b) Chùng minh r¬ng AM = AN. 
2) Cho tam gi¡c nhån ABC, D l  iºm tr¶n c¤nh AB (D6= A,B), trung tuy¸n AM c­t CD t¤i E. 
Chùng minh r¬ng n¸uDBM +DEM = 180 th¼ BC  AC 
p 
2. 
C¥u 5: (1,0 iºm) 
Cho x,y l  c¡c sè thüc thay êi thäa m¢n: 
( 
x  1; y  1 
x + y  4 
T¼m gi¡ trà nhä nh§t cõa biºu thùc: 
P = 
x4 
(y  1)3 + 
y4 
(x  1)3 
 H¸t  
WWW.VNMATH.COM
UBND TỈNH BẮC NINH 
SỞ GIÁO DỤC VÀ ĐÀO TẠO 
ĐỀ THI TUYỂN SINH VÀO LỚP 10 THPT CHUYÊN 
Năm học 2012 – 2013 
Môn thi: Toán (Dành cho thí sinh thi vào chuyên Toán, Tin) 
Thời gian làm bài: 150 phút (Không kể thời gian giao đề) 
Ngày thi: 30 tháng 6 năm 2012. 
ĐỀ CHÍNH THỨC 
Bài 1 (2,5 điểm) 
1/ Rút gọn biểu thức sau: 
A  4  10  2 5  4  10  2 5 . 
2/ Giải phương trình: 
x2  x2  2x 19  2x+39 . 
Bài 2 (2,0 điểm) 
1/ Cho ba số a, b, c thỏa mãn: 4a 5b  9c  0 . Chứng minh rằng phương trình 
ax2  bx  c  0 luôn có nghiệm. 
    
xy y2 x 7y 
x x y 12 
y 
 
2/ Giải hệ phương trình:   
   
Bài 3 (1,5 điểm) 
1/ Cho ba số dương a, b, c thỏa mãn: a  b  c 1. Chứng minh rằng: 
1 a1 b1 c  81 a 1 b1 c . 
2/ Phân chia chín số: 1,2,3,4,5,6,7,8,9 thành ba nhóm tùy ý, mỗi nhóm ba số. Gọi 1 T 
là tích ba số của nhóm thứ nhất, 2 T là tích ba số của nhóm thứ hai, 3 T là tích ba số của nhóm 
thứ ba. Hỏi tổng 1 2 3 T  T  T có giá trị nhỏ nhất là bao nhiêu? 
Bài 4 (2,5 điểm) 
Cho đường tròn tâm O bán kính R và dây cung BC cố định khác đường kính. Gọi A là 
một điểm chuyển động trên cung lớn BC của đường tròn (O) sao cho tam giác ABC nhọn; 
AD,BE,CF là các đường cao của tam giác ABC. Các đường thẳng BE, CF tương ứng cắt (O) 
tại các điểm thứ hai là Q, R. 
1/ Chứng minh rằng QR song song với EF. 
2/ Chứng minh rằng diện tích tứ giác AEOF bằng EF. R 
2 
. 
3/ Xác định vị trí của điểm A để chu vi tam giác DEF lớn nhất. 
Bài 5 (1,5 điểm) 
1/ Tìm hai số nguyên a,b để a4  4b4 là số nguyên tố. 
2/ Hãy chia một tam giác bất kì thành 7 tam giác cân trong đó có 3 tam giác bằng nhau. 
-----------------------Hết----------------------- 
(Đề thi gồm có 01 trang) 
Họ và tên thí sinh:………………………..…………………..Số báo danh:……….……….
UBND TỈNH BẮC NINH 
SỞ GIÁO DỤC VÀ ĐÀO TẠO 
HƯỚNG DẪN CHẤM 
ĐỀ THI TUYỂN SINH VÀO LỚP 10 THPT CHUYÊN 
Năm học 2012 – 2013 
Môn thi: Toán (Dành cho thí sinh thi vào chuyên toán, tin) 
Bài Đáp án Điểm 
1/ Rút gọn biểu thức sau: A  4  10  2 5  4  10  2 5 . 1,5 
Nhận xét rằng A  0. 0,25 
A2  4 10  2 5  4  10  2 5  2 4  10  2 5 4  10  2 5  0,25 
 8  2 6  2 5 0,25 
 2 
 8  2 5 1 
0,25 
 2 
 6  2 5  5 1 . 
0,25 
Vậy A 1 5 0,25 
Giải phương trình: x2  x2  2x 19  2x+39 (*) 1,0 
Đặt t  x2  2x 19  0. 0,25 
(*) trở thành: t2  t  20  0  t  
4 ( 
    
nhËn) 
lo¹i 
t 5 ( ) 
0,25 
t  4x2  2x 19 16x2  2x 35  0. 0,25 
1 
(2,5 
điểm) 
x 7 
x 5 
  
     
. 0,25 
1/ Cho 4a 5b 9c  0 , chứng minh phương trình ax2  bx  c  0 luôn có nghiệm. 1,0 
Xét trường hợp a = 0. Nếu b = 0 thì từ 4a 5b 9c  0 , ta suy ra c = 0, do đó 
phương trình (1) nghiệm đúng với mọi 0,25 
x   . Còn nếu b  0 , phương trình (1) trở thành bx  c  0 , có nghiệm x   c 
. 
b 
Trường hợp a  0 , (1) là phương trình bậc hai. Từ 4a 5b 9c  0 , ta có 
 
b 4a 9c 
 . Suy ra, 
5 
0,25 
2 2 2 2 2 2 
b 2 4ac (4a  9c) 4a 16a 28ac  81c (2a  7c) 12a 
 
32c 
25 2 
5 
c 
5 
0 
2 
    
 
 
 
   . 0,25 
Do đó, (1) có hai nghiệm phân biệt. 
Vậy trong mọi trường hợp, (1) luôn có nghiệm. 0,25 
    
xy y2 x 7y 
x x y 12 
y 
 
2/ Giải hệ phương trình:   
   
1,0 
2 
(2,0 
điểm) 
ĐK: y  0 
Hệ tương đương với 
x y x 7 
 
    y 
x  x  y  
 
12 
y 
 
   ta có hệ: u v 7 
, đặt u x y, v x 
y 
   
  
uv 12 
0,25
u 3 u 4 
v 4 v 3 
    
       
0,25 
Với u  4, v  3ta có hệ 
x 3 x 3 
y 
             
y 1 
x y 4 
0,25 
Với u  3, v  4 ta có hệ 
x x 12 4 5 y 
y 3 x y 3 
5 
         
      
0,25 
1/ Cho ba số dương a, b, c thỏa mãn: a  b  c 1. Chứng minh rằng: 
1 a1 b1 c  81 a 1 b1 c . 
1,0 
Từ a + b + c = 1 ta có 1 + a = (1 – b) + (1 – c)  2 (1 b)(1 c) 
(Vì a, b, c 1 nên 1 – b ; 1 – c ; 1 – a là các số dương). 
0,25 
Tương tự ta có 1 + b  2 (1 c)(1 a) và 1 + c  2 (1 a)(1 b). 0,25 
Nhân các vế của ba BĐT ta có: 
1 a1 b1 c  81 a 1 b1 c  đpcm. 0,25 
Dấu bằng xảy ra khi và chỉ khi a  b  c  1 
. 0,25 
3 
2/ Phân chia chín số: 1,2,3,4,5,6,7,8,9 thành ba nhóm tùy ý, mỗi nhóm ba số. Gọi 
1 T là tích ba số của nhóm thứ nhất, 2 T là tích ba số của nhóm thứ hai, 3 T là tích ba 
số của nhóm thứ ba. Hỏi tổng 1 2 3 T  T  T có giá trị nhỏ nhất là bao nhiêu? 
0,5 
1 2 3 1 2 3 T  T  T  3 T .T .T 
Ta có: 3 
3 
1 2 3 T .T .T 1.2.3.4.5.6.7.8.9  72.72.70  71 
0,25 
3 
(1,5 
điểm) 
Do đó, 1 2 3 T  T  T  213 mà 1 2 3 T ,T ,T nguyên nên 1 2 3 T  T  T  214 . 
Ngoài ra, 214  72  72  70 1.8.9  3.4.6  2.5.7 . 
Nên giá trị nhỏ nhất của 1 2 3 T  T  T là 214. 
0,25 
Cho đường tròn tâm O bán kính R và dây cung BC cố định khác đường 
kính. Gọi A là một điểm chuyển động trên cung lớn BC của đường tròn (O) 
sao cho tam giác ABC nhọn; AD,BE,CF là các đường cao của tam giác ABC. 
Các đường thẳng BE, CF tương ứng cắt (O) tại các điểm thứ hai là Q, R. 
1/ Chứng minh rằng QR song song với EF. 
1,0 
O 
R 
Q 
F 
E 
A 
B D C 
Vì BEC  BFC  900 nên tứ giác BCEF 
nội tiếp đường tròn đường kính BC. 
0,25 
4 
(2,5 
điểm) 
Suy ra, BEF  BCF. 0,25
Mà BCF BQR 1 sđ BR 
      
2 
  
nên BEF  BQR . 0,25 
Suy ra, QR / /EF . 0,25 
2/ Chứng minh rằng diện tích tứ giác AEOF bằng EF. R 
2 
. 0,5 
Vì tứ giác BCEF nội tiếp nên EBF  ECF mà EBF 1 sđ AQ,ECF 1 sđ A 
  R nên 
2 2 
AQ  AR . 
0,25 
Do đó, OA  QR mà QR / /EF nên OA  EF. 
Vì OA  EF nên AEOF 
  0,25 
S EF.OA EF.R . 
2 2 
3/ Xác định vị trí của điểm A để chu vi tam giác DEF lớn nhất. 1,0 
Tương tự câu 2, BFOD CDOE 2S  FD.R, 2S  DE.R . 
Mà tam giác ABC nhọn nên O nằm trong tam giác ABC. 
0,25 
Suy ra, 2S  2S  2S  2S  R  DE  EF  FD  . ABC AEOF BFOD CDOE 0,25 
Vì R không đổi nên đẳng thức trên suy ra chu vi tam giác DEF lớn nhất khi và chỉ 
khi diện tích tam giác ABC lớn nhất. 0,25 
Mà S  1 BC.AD 
với BC không đổi nên S lớn nhất khi AD lớn nhất. Khi đó, 
ABC 
ABC 2 
A là điểm chính giữa của cung lớn BC. 
0,25 
1/ Tìm hai số nguyên a, b để a4  4b4 là số nguyên tố. 1,0 
a4  4b4  a2  2ab  2b2 a2  2ab  2b2 . 0,25 
Vì a2  2ab  2b2  0;a2  2ab  2b2  0 . 
0,25 
Nên a4  4b4 nguyên tố  Một thừa số là 1 còn thừa số kia là số nguyên tố . 
    
           
TH1:   
  
a b 1 
  
2 
2 
2 2 2 2 
  2 
 
 
 2 
 
(1) 
b 0 
a 2ab 2b 1 a b b 1 
a b 0 
(2) 
b 1 
*Với (1)b  0a2 1M 1 (loại). 
   a  b  
1 
*Với 2 
      
a b 1 
(thỏa mãn). 
0,25 
5 
(1,5 
điểm) 
    
           
TH2:   
  
a b 1 
  
2 
2 
2 2 2 2 
  2 
 
 
 2 
 
(3) 
b 0 
a 2ab 2b 1 a b b 1 
a b 0 
(4) 
b 1 
*Với (3) b  0a2  1 M  1 (loại). 
   a  1  a   
1 
*Với 4 
         
b 1 b 1 
(thỏa mãn). 
Vậy các cặp số a; b cần tìm là: 1;1,1;1,1;1,1;1. 
0,25
2/ Hãy chia một tam giác bất kì thành 7 tam giác cân trong đó có 3 tam giác bằng 
nhau. 
0,5 
F E 
O 
G 
D 
C 
A B 
Trường hợp 1:Tam giác ABC không cân. 
Giả sử AB là cạnh lớn nhất của tam giác ABC. 
Vẽ cung tròn tâm A, bán kính AC cắt AB tại D. 
Vẽ cung tròn tâm B, bán kính BD cắt BC tại E. 
Vẽ cung tròn tâm C, bán kính CE cắt AC tại F. 
Vẽ cung tròn tâm A, bán kính AF cắt AB tại G. 
Dễ dàng chứng minh 5 điểm C,D,E,F,G thuộc đường tròn tâm O với O là tâm 
đường tròn nội tiếp tam giác ABC. 
Nối 5 điểm đó với O, nối A, B với O, nối F với G, D với E ta được 7 tam giác 
cân: AGF,OGF,ODG,BDE,ODE,OCE,OCF. 
Trong đó, có ba tam giác bằng nhau là: OCE,OCF,OGD. 
0,25 
A 
I 
G H 
F 
E 
D 
B C 
Trường hợp 2: Tam giác ABC cân. 
Giả sử tam giác ABC cân tại A. Gọi D, E, F, G, H, I lần lượt là trung điểm các 
đoạn thẳng: AB, BC, CA, DE, EF, FD. Khi đó, ta có 7 tam giác cân ADF, BDE, 
CEF, DGI, EGH, FHI, GHI trong đó ba tam giác bằng nhau là: ADF, BDE, CEF. 
0,25 
Các chú ý khi chấm: 
1. Bài làm của học sinh phải chi tiết, lập luận chặt chẽ, tính toán chính xác mới được 
điểm tối đa. 
2. Với các cách giải đúng nhưng khác đáp án, tổ chấm trao đổi và thống nhất điểm chi 
tiết (đến 0,25 điểm) nhưng không được vượt quá số điểm dành cho bài hoặc phần đó. Trong 
trường hợp sai sót nhỏ có thể cho điểm nhưng phải trừ điểm chỗ sai đó. 
3. Với Bài 4 và Bài 5.2 không cho điểm bài làm nếu học sinh không vẽ hình. 
4. Mọi vấn đề phát sinh trong quá trình chấm phải được trao đổi trong tổ chấm và chỉ 
cho điểm theo sự thống nhất của cả tổ. 
5. Điểm toàn bài là tổng số điểm các phần đã chấm, không làm tròn điểm.
1
Së gi¸o dôc vμ ®μo t¹o 
Hμ nam 
Kú thi tuyÓn sinh líp 10 THPT chuyªn 
N¨m häc 2012-2013 
M«n thi : to¸n 
Thêi gian lμm bμi: 120 phót 
§Ò chÝnh thøc 
Bμi 1. (2,0 ®iÓm) Cho biÓu thøc 
  1  3 a  b 3     a3b  ab3 
              
 M . 
    
3b a b b 3a a b a b 
1. T×m ®iÒu kiÖn cña a, b ®Ó M x¸c ®Þnh vμ rót gän M 
2. TÝnh gi¸ trÞ cña M khi a = 
2 5 
    
a 5 2, b 
3 3 
Bμi 2. (2,0®iÓm) 
Cho ph­ 
¬ng tr×nh x4 2(m2 3)x2 m4 5 0 (mlμ thamsè) 
1. Chøng minh r»ng ph­ 
¬ng tr×nh cã bèn nghiÖm x1; x2; x3; x4 víi mäi m thuéc 
R 
2. X¸c ®Þnh m ®Ó 2 2 2 2 
1 2 3 4 1 2 3 4 2x x x x (x x x x )28 
Bμi 3. (1,5 ®iÓm) 
T×m tÊt c¶ c¸c sè nguyªn x, y tho¶ m·n ph­ 
¬ng tr×nh: 
x3 – x2y + 3x – 3y – 5 = 0 
Bμi 4. (3,5 ®iÓm) 
Cho tam gi¸c ®Òu ABC néi tiÕp trong ®­êng 
trßn (O). Mét ®­êng 
thÈng d thay 
®æi ®i qua A, c¾t (O) t¹i ®iÓm thø hai lμ E, c¾t hai tiÕp tuyÕn kÎ tõ B vμ C cña ®­êng 
trßn (O) lÇn l­ît 
t¹i M vμ N sao cho A, M, N n»m ë cïng nöa mÆt ph¼ng bê BC. Gäi 
giao ®iÓm cña hai ®­êng 
th¼ng MC vμ BN lμ F. Chøng minh r»ng: 
1. Hai tam gi¸c MBA vμ CAN ®ång d¹ng vμ tÝch MB.CN kh«ng ®æi. 
2. Tø gi¸c BMEF néi tiÕp trong mét ®­êng 
trßn. 
3. §­êng 
th¼ng EF lu«n ®i qua mét ®iÓm cè ®Þnh khi (d) thay ®æi 
Bμi 5. (1.0 ®iÓm) 
Cho bèn sè thùc a, b, c, d tho¶ m·n: ad – bc = 3 . 
Chøng minh r»ng: a2 b2 c2 d2 acbd3. 
DÊu b»ng x¶y ra khi nμo? 
-----------------------HÕt----------------------- 
Hä vμ tªn thÝ sinh : ......................................................Sè b¸o danh :....................... 
Ch÷ kÝ cña gi¸m thÞ 1 : .............................Ch÷ kÝ cña gi¸m thÞ 2:............................
ĐẠI HỌC QUỐC GIA HÀ NỘI ĐỀ THI TUYỂN SINH LỚP 10 
TRƯỜNG THPT CHUYÊN NĂM 2012 
MÔN THI: TOÁN (cho tất cả các thí sinh) 
Thời gian làm bài: 120 phút (Không kể thời gian phát đề) 
Câu I. 1) Giải phương trình 
 x  9  2012 x  6  2012  x  9x  6 
2) Giải hệ phương trình 
   
2 2 2 4 
x y xy 
x y y 
   
2    
4 
Câu II. 1) Tìm tất cả các cặp số nguyên x; y thỏa mãn đẳng thức: 
x  y 1xy  x  y  5  2x  y 
2) Giả sử x, y là các số thực dương thỏa mãn điều kiện  x 1 y 1 4 
Tìm giá trị nhỏ nhất của biểu thức : 
y 
2 2 
x 
P x 
  
y 
Câu III. Cho tam giác nhọn ABC nội tiếp đường tròn tâm O .Gọi M là một điểm trên cung 
nhỏ BC ( M khác B,C và AM không đi qua O).Giả sử P là một điểm thuộc đoạn thẳng AM 
sao cho đường tròn đường kính MP cắt cung nhỏ BC tại điểm N khác M. 
1) Gọi D là điểm đối xứng với điểm M qua O .Chứng minh rằng ba điểm N,P,D 
thẳng hàng 
2) Đường tròn đường kính MP cắt MD tại Q khác M. Chứng minh rằng Q là tâm 
đườn tròn nội tiếp tam giác AQN. 
Câu IV. Giả sử a,b,c là các số thực dương thỏa mãn a  b  3  c;c  b 1;a  b  c 
Tìm giá trị nhỏ nhất của biểu thức: 
Q ab a b c ab 
2    (  
1) 
(  1)(  1)(  
1) 
 
a b c 
Cán bộ coi thi không giải thich gì thêm.
ĐÁP ÁN VÀ BIỂU ĐIỂM 
Bài thi Toán vào lớp 10 chuyên( chuyên TN và chuyên ngữ) 2012-2013 
Câu Đáp án Điểm 
I 
 
1) x  9  2012 x  6  2012  x  9x  6 (1) Đặt x  9  a  0 , x  6  b  0 
 
(1) 
a 
 
2012 
 a  2012 b  2012  ab  ( a  2012 )( b  1 )  0   
b 
 
1 
  
 9  
2012 
x 
 6  
1 
x 
9 4048144 
x 
 
  
  
  
6 1 
x 
x 4048135 
vậy tập nghiệm pt là S  4048135;5 
 
  
  
 
5 
x 
2 
2) 
   
2 2 2 4(1) 
x y xy 
x y y 
   
2    
4 
 
   
2 2 2 4 
x y xy 
x y y 
   
4  2  2  
8 
 x2  y2  2xy  4x  4y 12 
x y 
 
  
 0 12 ) ( 4 ) ( 2      y x y x  0 ) 2 )( 6 (      y x y x   
   
2 
6 
x y 
 
 
* x  y  6  x  6  y thế vào (1) có (6  y)2  y2  2y  4  2 y 2  17 y  16  0 
 y2  7y 16  0 vô nghiệm 
* x  y  2  x  2  y thế vào (1) có (2  y)2  y2  2y  4  2y2  2y  0 
 
y 
 
0 
 
x  2; y  
0 
  Vậy tập nghiệm của hệ S  (2;0);(1;1) 
y 
 
1 
x  1; y 
 
1 
1,5 
II 
1. x  y 1xy  x  y  5  2x  y  x  y 1xy  x  y  2x  y 1 3 
 x  y 1xy  x  y  2  3  3.1 1.3  (3).(1)  (1).(3) 
Xét các trường hợp: 
a) 
   
1 3 
x y 
   
2 1 
xy x y 
    
 
   
x y 
  
 
1 
2 
xy 
 
   
x b) 
1 
y 
 
 
1 
   
1 1 
x y 
    
2 3 
xy x y 
     
 
   
x y 
   
 
1 
2 
xy 
 
   
1 
  
1 
  
x 
y 
c) 
   
1 1 
x y 
   
2 3 
xy x y 
    
 
   
x y vô nghiệm d) 
  
 
5 
0 
xy 
   
x  y  1   
3 
 
2 1 
xy x y 
     
   
x y vô nghiệm 
   
 
5 
4 
xy 
Vậy tập nghiệm của phương trình S  (1;1);(1;1) 
1,5 
2 2 
y 
P x 
2. Áp dụng BĐT Cauchy-Schwarz ta có x 
x  
y   
 
  x y 
y 
x y 
 
( )2 (1) 
Đẳng thức có  x=y 
Gt  x 1 y 1 4  4  x  y  xy 1 1 
x  y 1 
x y = x  y  2 (BĐT 
2 2 
2 
1  
 
 
 
 
Cauchy) 
 x  y  2 (2) Đẳng thức có x=y=1 
Từ (1) và (2)  P  x  y  2 . P=2 x=y=1. Vậy 2 min P  x=y=1 
1
III 
1. M(O) góc MND=900 DNMN (1) 
M(O') góc MNP=900  PNMN (2) 
Từ (1) và (2) D, P, N thẳng hàng 
2. Góc DNA=DMA (3) (=1/2sđ cung AD) 
Góc PNQ=PMQ (4) (=1/2sđ cung PQ) 
Từ (3) và (4)  gócPNA=PNQNP là p giác góc ANQ(5) 
Góc MAN=MDN (6) (=1/2sđ cung MN) 
Tứ giác ADQP có góc PAD=DQP=900 nội tiếp được  
góc PDQ=PAQ (7) 
Q 
A 
P 
N 
O 
B C 
M 
D 
O' 
Từ (6) và (7)  góc PAN= PAQAP là phân giác góc NAQ (8) 
Từ (5) và (8)  P là tâm đường tròn nội tiếp tam giác ANQ 
1,5 
1,5 
IV 
Q 2 ab  a  b  c ( ab  
1) 
= 
Ta có (  1)(  1)(  
1) 
 
a b c 
a b ab c 
( 1).( 1) ( 1).( 1) 
     
( a  1).( b  1).( c 
 
1) 
= 
1 
 
ab 
( 1).( 1) 
1 
1 
  
 
 a b 
c 
 
1 
1 
1 
1 
 
ab 
   
 
a b 
  ab a b 
Do a  b  c  b 1 a 1 ; b  a 1  (a 1).(b 1)  0  ab  a  b 1 c 1 2 
Suy ra 
ab 
 
1 
2( 1) 
2 
1 
 
 
 
 
ab 
ab 
Q 
Đặt ab  x  x  2  
1 
 
x 
2( 1) 
2 
1 
 
 
 
 
x 
x 
Q 
 
5 
x x ( do x  2 )  
(  2).(  
5)  
Q 0 
12 
x 
5  
1 
 
2( 1) 
2 
1 
12 
 
 
 
  
x 
x 
12(  1).(  
2) 
 
x x 
Q  5 
12 
Q  5 a=1; b=2; c=3. Vậy 
12 
5 
min Q  a=1; b=2; c=3. 
12 
1
1 
Së gi¸o dôc vμ ®μo t¹o 
H­ng 
yªn 
®Ò chÝnh thøc 
(§Ò thi cã 01 trang) 
kú thi tuyÓn sinh vμo líp 10 thpt chuyªn 
N¨m häc 2012 - 2013 
M«n thi: To¸n 
(Dμnh cho thÝ sinh dù thi c¸c líp chuyªn: To¸n, Tin) 
Thêi gian lμm bμi: 150 phót 
Bài 1: (2 điểm) 
a) Cho A = 20122  20122.20132  20132 . Chứng minh A là một số tự nhiên. 
b) Giải hệ phương trình 
x 1 x 3 
 
 2 
   y 2 
y 
x  1  x  
3 
y y 
 
Bài 2: (2 điểm) 
a) Cho Parbol (P): y = x2 và đường thẳng (d): y = (m +2)x – m + 6. Tìm m để đường thẳng (d) cắt 
Parabol (P) tại hai điểm phân biệt có hoành độ dương. 
b) Giải phương trình: 5 + x + 2 (4  x)(2x  2)  4( 4  x  2x  2) 
Bài 3: (2 điểm) 
a) Tìm tất cả các số hữu tỷ x sao cho A = x2 + x+ 6 là một số chính phương. 
b) Cho x  1 và y  1. Chứng minh rằng : 
(x3  y3)  (x2  
y2 ) 8 
(x 1)(y 1) 
 
  
Bài 4 (3 điểm) 
Cho tam giác ABC nhọn nội tiếp đường tròn tâm O, đường cao BE và CF. Tiếp tuyến tại B và C cắt nhau tại S, 
gọi BC và OS cắt nhau tại M 
a) Chứng minh AB. MB = AE.BS 
b) Hai tam giác AEM và ABS đồng dạng 
c) Gọi AM cắt EF tại N, AS cắt BC tại P. CMR NP vuông góc với BC 
Bài 5: (1 điểm) 
Trong một giải bóng đá có 12 đội tham dự, thi đấu vòng tròn một lượt (hai đội bất kỳ thi đấu với nhau đúng một 
trận). 
a) Chứng minh rằng sau 4 vòng đấu (mỗi đội thi đấu đúng 4 trận) luôn tìm được ba đội bóng đôi một chưa thi 
đấu với nhau. 
b) Khẳng định trên còn đúng không nếu các đội đã thi đấu 5 trận? 
HƯỚNG DẪN GIẢI 
Bài 1: (2 điểm)
2 
a) Cho A = 20122  20122.20132  20132 
Đặt 2012 = a, ta có 20122  20122.20132  20132  a2  a2 (a 1)2  (a 1)2 
 (a2  a 1)2  a2  a 1 
b) Đặt 
x a 
y 
x 1 b 
y 
   
  
 
Ta có 
x 1 x 3 
 
 2 
   y 2 
y 
x  1  x  
3 
y y 
 
  
         
2 x 1 x 3 
y y 
x 1 x 3 
 
   y y 
nên 
       
   
      
b2 a 3 b2 b 6 0 
b a 3 b a 3 
a 6 a 1 
    
 v 
     
 b 3 b 2 
Bài 2: 
a) ycbt tương đương với PT x2 = (m +2)x – m + 6 hay x2 - (m +2)x + m – 6 = 0 có hai nghiệm dương 
phân biệt. 
b) Đặt t = 4  x  2x  2 
Bài 3: 
a) x = 0, x = 1, x= -1 không thỏa mãn. Với x khác các giá trị này, trước hết ta chứng minh x phải là số 
nguyên. 
+) x2 + x+ 6 là một số chính phương nên x2 + x phải là số nguyên. 
m 
+) Giả sử x  với m và n có ước nguyên lớn nhất là 1. 
n 
Ta có x2 + x = 
2 2 
2 2 
m m m  
mn 
n n n 
  là số nguyên khi m2 mn chia hết cho n2 
nên m2 mn chia hết cho n, vì mn chia hết cho n nên m2 chia hết cho n và do m và n có ước nguyên 
lớn nhất là 1, suy ra m chia hết cho n( mâu thuẫn với m và n có ước nguyên lớn nhất là 1). Do đó x 
phải là số nguyên. 
Đặt x2 + x+ 6 = k2 
Ta có 4x2 + 4x+ 24 = 4 k2 hay (2x+1)2 + 23 = 4 k2 tương đương với 4 k2 - (2x+1)2 = 23 
(x3  y3)  (x2  y2 ) x2 (x  1)  y2 (y  
1) 
 
(x  1)(y  1) (x  1)(y  
1) 
= 
x2  
y2 
y  1 x  
1 
(x  1)2  2(x  1)  1 (y  1)2  2(y  1)  
1 
  
y  1 x  
1 
  2  2     (x 1) (y 1) 2(y 1)  2(x  1)   1  
     1 
 y  1 x  1   x  1 y  1     y  1 x  1 
  
. 
Theo BĐT Côsi
3 
(x  1)2 (y  2  1) 2 (x  1)2 . (y  
1)2  2 (x  1)(y  
1) 
y 1 x 1 y 1 x 1 
    
2(y  1) 2(x   1)  2(y  1)  
. 2(x 1)  
4 
x 1 y 1 x 1 y 1 
    
1  1  
2 1 . 1 
y 1 x 1 y 1 x 1 
    
2 1 . 1 (x 1)(y 1) 2.2 1 . 1 . (x 1)(y 1) 4 
  
         y  1 x  1   y  1 x  
1 
Nên có đpcm 
Bài 4 
a) Suy ra từ hai tam giác đồng dạng là ABE và BSM 
b) Từ câu a) ta có AE MB 
 (1) 
AB BS 
Mà MB = EM( do tam giác BEC vuông tại E có M là trung điểm của BC 
Nên AE  
EM 
AB BS 
Có MOB  BAE,EBA  BAE  900 ,MBOMOB  900 
Nên MBO  EBA do đó MEB  OBA( MBE) 
Suy ra MEA  SBA(2) 
Từ (1) và (2) suy ra hai tam giác AEM và ABS đồng dạng(đpcm.) 
c) Dễ thấy SM vuông góc với BC nên để chứng minh bài toán ta chứng minh NP //SM. 
+ Xét hai tam giác ANE và APB: 
Từ câu b) ta có hai tam giác AEM và ABS đồng dạng nên NAE  PAB, 
Mà AEN  ABP ( do tứ giác BCEF nội tiếp) 
AN AE 
Do đó hai tam giác ANE và APB đồng dạng nên  
AP AB 
P 
N 
F 
E 
M 
S 
O 
A 
B 
C 
Q
4 
Lại có AM AE 
 ( hai tam giác AEM và ABS đồng dạng) 
AS AB 
Suy ra AM AN 
 nên trong tam giác AMS có NP//SM( định lí Talet đảo) 
AS AP 
Do đó bài toán được chứng minh. 
Bài 5 
a. Giả sử kết luận của bài toán là sai, tức là trong ba đội bất kỳ thì có hai đội đã đấu với nhau rồi. Giả sử 
đội đã gặp các đội 2, 3, 4, 5. Xét các bộ (1; 6; i) với i Є{7; 8; 9;…;12}, trong các bộ này phải có ít nhất một 
cặp đã đấu với nhau, tuy nhiên 1 không gặp 6 hay i nên 6 gặp i với mọi i Є{7; 8; 9;…;12} , vô lý vì đội 6 như 
thế đã đấu hơn 4 trận. Vậy có đpcm. 
b. Kết luận không đúng. Chia 12 đội thành 2 nhóm, mỗi nhóm 6 đội. Trong mỗi nhóm này, cho tất cả 
các đội đôi một đã thi đấu với nhau. Lúc này rõ ràng mỗi đội đã đấu 5 trận. Khi xét 3 đội bất kỳ, phải có 2 đội 
thuộc cùng một nhóm, do đó 2 đội này đã đấu với nhau. Ta có phản ví dụ. 
Có thể giải quyết đơn giản hơn cho câu a. như sau: 
Do mỗi đội đã đấu 4 trận nên tồn tại hai đội A, B chưa đấu với nhau. Trong các đội còn lại, vì A và B chỉ 
đấu 3 trận với họ nên tổng số trận của A, B với các đội này nhiều nhất là 6 và do đó, tồn tại đội C trong số các 
đội còn lại chưa đấu với cả A và B. Ta có A, B, C là bộ ba đội đôi một chưa đấu với nhau.
ĐẠI HỌC QUỐC GIA HÀ NỘI ĐỀ THI TUYỂN SINH LỚP 10 
TRƯỜNG THPT CHUYÊN NĂM 2012 
MÔN THI: TOÁN (cho tất cả các thí sinh) 
Thời gian làm bài: 120 phút (Không kể thời gian phát đề) 
Câu I. 1) Giải phương trình 
 x  9  2012 x  6  2012  x  9x  6 
2) Giải hệ phương trình 
   
2 2 2 4 
x y xy 
x y y 
   
2    
4 
Câu II. 1) Tìm tất cả các cặp số nguyên x; y thỏa mãn đẳng thức: 
x  y 1xy  x  y  5  2x  y 
2) Giả sử x, y là các số thực dương thỏa mãn điều kiện  x 1 y 1 4 
Tìm giá trị nhỏ nhất của biểu thức : 
y 
2 2 
x 
P x 
  
y 
Câu III. Cho tam giác nhọn ABC nội tiếp đường tròn tâm O .Gọi M là một điểm trên cung 
nhỏ BC ( M khác B,C và AM không đi qua O).Giả sử P là một điểm thuộc đoạn thẳng AM 
sao cho đường tròn đường kính MP cắt cung nhỏ BC tại điểm N khác M. 
1) Gọi D là điểm đối xứng với điểm M qua O .Chứng minh rằng ba điểm N,P,D 
thẳng hàng 
2) Đường tròn đường kính MP cắt MD tại Q khác M. Chứng minh rằng Q là tâm 
đườn tròn nội tiếp tam giác AQN. 
Câu IV. Giả sử a,b,c là các số thực dương thỏa mãn a  b  3  c;c  b 1;a  b  c 
Tìm giá trị nhỏ nhất của biểu thức: 
Q ab a b c ab 
2    (  
1) 
(  1)(  1)(  
1) 
 
a b c 
Cán bộ coi thi không giải thich gì thêm.
ĐÁP ÁN VÀ BIỂU ĐIỂM 
Bài thi Toán vào lớp 10 chuyên( chuyên TN và chuyên ngữ) 2012-2013 
Câu Đáp án Điểm 
I 
 
1) x  9  2012 x  6  2012  x  9x  6 (1) Đặt x  9  a  0 , x  6  b  0 
 
(1) 
a 
 
2012 
 a  2012 b  2012  ab  ( a  2012 )( b  1 )  0   
b 
 
1 
  
 9  
2012 
x 
 6  
1 
x 
9 4048144 
x 
 
  
  
  
6 1 
x 
x 4048135 
vậy tập nghiệm pt là S  4048135;5 
 
  
  
 
5 
x 
2 
2) 
   
2 2 2 4(1) 
x y xy 
x y y 
   
2    
4 
 
   
2 2 2 4 
x y xy 
x y y 
   
4  2  2  
8 
 x2  y2  2xy  4x  4y 12 
x y 
 
  
 0 12 ) ( 4 ) ( 2      y x y x  0 ) 2 )( 6 (      y x y x   
   
2 
6 
x y 
 
 
* x  y  6  x  6  y thế vào (1) có (6  y)2  y2  2y  4  2 y 2  17 y  16  0 
 y2  7y 16  0 vô nghiệm 
* x  y  2  x  2  y thế vào (1) có (2  y)2  y2  2y  4  2y2  2y  0 
 
y 
 
0 
 
x  2; y  
0 
  Vậy tập nghiệm của hệ S  (2;0);(1;1) 
y 
 
1 
x  1; y 
 
1 
1,5 
II 
1. x  y 1xy  x  y  5  2x  y  x  y 1xy  x  y  2x  y 1 3 
 x  y 1xy  x  y  2  3  3.1 1.3  (3).(1)  (1).(3) 
Xét các trường hợp: 
a) 
   
1 3 
x y 
   
2 1 
xy x y 
    
 
   
x y 
  
 
1 
2 
xy 
 
   
x b) 
1 
y 
 
 
1 
   
1 1 
x y 
    
2 3 
xy x y 
     
 
   
x y 
   
 
1 
2 
xy 
 
   
1 
  
1 
  
x 
y 
c) 
   
1 1 
x y 
   
2 3 
xy x y 
    
 
   
x y vô nghiệm d) 
  
 
5 
0 
xy 
   
x  y  1   
3 
 
2 1 
xy x y 
     
   
x y vô nghiệm 
   
 
5 
4 
xy 
Vậy tập nghiệm của phương trình S  (1;1);(1;1) 
1,5 
2 2 
y 
P x 
2. Áp dụng BĐT Cauchy-Schwarz ta có x 
x  
y   
 
  x y 
y 
x y 
 
( )2 (1) 
Đẳng thức có  x=y 
Gt  x 1 y 1 4  4  x  y  xy 1 1 
x  y 1 
x y = x  y  2 (BĐT 
2 2 
2 
1  
 
 
 
 
Cauchy) 
 x  y  2 (2) Đẳng thức có x=y=1 
Từ (1) và (2)  P  x  y  2 . P=2 x=y=1. Vậy 2 min P  x=y=1 
1
III 
1. M(O) góc MND=900 DNMN (1) 
M(O') góc MNP=900  PNMN (2) 
Từ (1) và (2) D, P, N thẳng hàng 
2. Góc DNA=DMA (3) (=1/2sđ cung AD) 
Góc PNQ=PMQ (4) (=1/2sđ cung PQ) 
Từ (3) và (4)  gócPNA=PNQNP là p giác góc ANQ(5) 
Góc MAN=MDN (6) (=1/2sđ cung MN) 
Tứ giác ADQP có góc PAD=DQP=900 nội tiếp được  
góc PDQ=PAQ (7) 
Q 
A 
P 
N 
O 
B C 
M 
D 
O' 
Từ (6) và (7)  góc PAN= PAQAP là phân giác góc NAQ (8) 
Từ (5) và (8)  P là tâm đường tròn nội tiếp tam giác ANQ 
1,5 
1,5 
IV 
Q 2 ab  a  b  c ( ab  
1) 
= 
Ta có (  1)(  1)(  
1) 
 
a b c 
a b ab c 
( 1).( 1) ( 1).( 1) 
     
( a  1).( b  1).( c 
 
1) 
= 
1 
 
ab 
( 1).( 1) 
1 
1 
  
 
 a b 
c 
 
1 
1 
1 
1 
 
ab 
   
 
a b 
  ab a b 
Do a  b  c  b 1 a 1 ; b  a 1  (a 1).(b 1)  0  ab  a  b 1 c 1 2 
Suy ra 
ab 
 
1 
2( 1) 
2 
1 
 
 
 
 
ab 
ab 
Q 
Đặt ab  x  x  2  
1 
 
x 
2( 1) 
2 
1 
 
 
 
 
x 
x 
Q 
 
5 
x x ( do x  2 )  
(  2).(  
5)  
Q 0 
12 
x 
5  
1 
 
2( 1) 
2 
1 
12 
 
 
 
  
x 
x 
12(  1).(  
2) 
 
x x 
Q  5 
12 
Q  5 a=1; b=2; c=3. Vậy 
12 
5 
min Q  a=1; b=2; c=3. 
12 
1
WWW.VNMATH.COM 
x+ x +2012 y+ y +2012 2012 
x + z - 4(y+z)+8 0 
WWW.VNMATH.COM 
1 
SỞ GIÁO DỤC VÀ ĐÀO TẠO 
HẢI DƯƠNG 
KÌ THI TUYỂN SINH LỚP 10 TRƯỜNG THPT CHUYÊN 
NGUYỄN TRÃI NĂM HỌC 2012- 2013 
Môn thi: TOÁN (chuyên) 
Thời gian làm bài: 150 phút 
Đề thi gồm : 01 trang 
Ngày thi 20 tháng 6 năm 2012 
ĐỀ CHÍNH THỨC 
Câu I (2,0 điểm) 
1) Phân tích đa thức sau thành nhân tử a2 (b-2c)+b2 (c-a)+2c2 (a-b)+abc . 
2) Cho x, y thỏa mãn x  3 y- y2+1+3 y+ y2+1 . Tính giá trị của biểu thức 
A  x4+x3y+3x2+xy- 2y2+1. 
Câu II ( 2,0 điểm) 
1) Giải phương trình (x2 - 4x+11)(x4 - 8x2+21)  35 . 
2) Giải hệ phương trình  2  2  
2 2 
 
 
 
. 
Câu III (2,0 điểm) 
1) Chứng minh rằng với mọi số nguyên n thì (n2 + n + 1) không chia hết cho 9. 
2) Xét phương trình x2 – m2x + 2m + 2 = 0 (1) (ẩn x). Tìm các giá trị nguyên 
dương của m để phương trình (1) có nghiệm nguyên. 
Câu IV (3,0 điểm) 
Cho tam giác ABC vuông tại A có AB  AC ngoại tiếp đường tròn tâm O. 
Gọi D, E, F lần lượt là tiếp điểm của (O) với các cạnh AB, AC, BC; BO cắt EF 
tại I. M là điểm di chuyển trên đoạn CE. 
1) Tính BIF. 
2) Gọi H là giao điểm của BM và EF. Chứng minh rằng nếu AM = AB thì 
tứ giác ABHI nội tiếp. 
3) Gọi N là giao điểm của BM với cung nhỏ EF của (O), P và Q lần lượt là 
hình chiếu của N trên các đường thẳng DE, DF. Xác định vị trí của điểm M để 
PQ lớn nhất. 
Câu V (1,0 điểm)
WWW.VNMATH.COM 
Cho 3 số a, b, c thỏa mãn 0  a  b  c  1. Tìm giá trị lớn nhất của biểu 
thức B (a+b+c+3) 1 + 1 + 1 
  
  
  
 . 
                     
x x 
      
x 
x 
( 2) 7 7 
WWW.VNMATH.COM 
2 
a+1 b+1 c+1 
----------------------------Hết---------------------------- 
Họ và tên thí sinh………………………………. Số báo danh………………...……………… 
Chữ kí của giám thị 1: ……………………… Chữ kí của giám thị 2: …………………… 
SỞ GIÁO DỤC VÀ ĐÀO TẠO 
HẢI DƯƠNG 
KÌ THI TUYỂN SINH LỚP 10 TRƯỜNG THPT 
CHUYÊN NGUYỄN TRÃI NĂM HỌC 2012 - 2013 
HƯỚNG DẪN VÀ BIỂU ĐIỂM CHẤM MÔN TOÁN (chuyên) 
Hướng dẫn chấm gồm : 03 trang 
I) HƯỚNG DẪN CHUNG. 
- Thí sinh làm bài theo cách riêng nhưng đáp ứng được yêu cầu cơ bản vẫn cho đủ điểm. 
- Việc chi tiết điểm số (nếu có) so với biểu điểm phải được thống nhất trong Hội đồng 
chấm. 
- Sau khi cộng điểm toàn bài, điểm lẻ đến 0,25 điểm. 
II) ĐÁP ÁN VÀ BIỂU ĐIỂM CHẤM. 
Câu Nội dung Điểm 
Câu I (2,0đ) 
a2 (b - 2c) +b2 (c - a) + 2c2 (a - b) + abc=2c2 (a - b)+ab(a-b)-c(a2 b2 )  ac(a b) 0,25 
 (a  b)[2c2  2ac  ab bc] 0,25 
 (a  b)[2c(c  a)  b(a  c)] 0,25 
1) 1,0 điểm 
 (a  b)(a  c)(b  2c) 0,25 
Có x = 3 y- y2+ 1  3 y+ y2+ 1 
 x3 = 2y +33 y - y2 + 1 . 3 y+ y2+ 1  3 y- y2+1  3 y+ y2+1  
  
0,25 
 x3 + 3x -2y = 0 0,25 
A = x4 + x3y + 3x2 - 2xy + 3xy - 2y2 + 1 = (x4+3x2 -2xy) +(x3y+3xy - 2y2 ) 1 0,25 
2) 1,0 điểm 
 x(x3+3x-2y) +y(x3+3x - 2y) 1 1 0,25 
Câu II (1,0đ) 
phương trình đã cho tương đương với (x  2)2  7 (x2  4)2  5  35(1) 0,25 
( 2) 2 
7 7 
Do 
2 2 2 
2 2 
( 2) 7 ( 4) 5 35 
( 4) 5 5 
x x x 
x x 
0,25 
2 
2 2 
(1) 
( 4) 5 5 
    
0,25 
1)1,0 điểm 
=x=2 0,25
WWW.VNMATH.COM 
 (x+ x +2012)(y+ y +2012)  2012 (1) 
 
x + z - 4(y+z)+8=0 (2) 
(1)x  x2  2012 y  y2  2012  y2  2012  y 2012 y2  2012  y 
(Do y2  2012  y  0y ) 
 2   2  
 
2 2 
x x 2012 2012 2012 y 2012 y x x 2012 y 2012 
y 
x y y x 
            
   2   2 
 
2012 2012 
2012 2012 2012 2012 
y x y x 
      
y  x y   y  x   
x x y x y 
( ) 0 
      
y x y x 
                
      
y y y y 
 ( x  2)  0  x 
  2 
           
 x  x  
m 
 
 x x  m 
 
1 2 x  x  m  * 
WWW.VNMATH.COM 
3 
2)1,0 điểm 
2 2 
2 2 
 2 2  2 2 
 
2 2 
2012 2012 
x y 
y x 
   
   
2 2 2 2 
2012 2012 
2 2 2 2 
2012 2012 2012 2012 
      
0,25 
Do 
2 
2 2 
2 
2012 | | 
2012 2012 0 
2012 | | 
y y x x y x 
x x x x 
0,25 
Thay y=-x vào(2) x2  z2  4x  4z 8  0(x  2)2  (z  2)2  0 0,25 
2 
2 
2 
( 2) 0 2 
y x 
z z 
Vậy hệ có nghiệm (x;y;z)=(-2;2;2). 
0,25 
Câu III (2,0đ) 
Đặt A = n2 + n + 1 do n n = 3k; n = 3k + 1; n = 3k + 2 (k  ) 0,25 
* n = 3k = A không chia hết cho 9 (vì A không chia hết cho 3) 0,25 
* n = 3k + 1 = A = 9k2 + 9k + 3 không chia hết cho 9. 0,25 
1)1,0 điểm 
* n = 3k +2 = A = 9k2 +9k+7 không chia hết cho 9 
Vậy với mọi số nguyên n thì A = n2 + n + 1 không chia hết cho 9. 
0,25 
Gi¶ sö tån t¹i m * ®Ó ph­ 
¬ng tr×nh cã nghiÖm x1, x2 
Theo vi-et: 
2 
1 2 
1 2 2 2 
 (x1 - 1) (x2 - 1) = - m2 + 2m + 3 
0,25 
Với m* . Ta cã x1x2 1vμ x1 + x2  4 mà x1hoÆc x2 nguyªn vμ 
2 * 
1 2 1 2  x , x  (x 1)(x 1)  0 
m2  2m 2  0(m1)(m3)  0 m  3m{1;2;3} 
0,25 
Víi m = 1; m = 2 thay vμo ta thÊy ph­ 
¬ng tr×nh ®· cho v« nghiÖm. 0,25 
2)1,0 điểm 
Víi m = 3 thay vμo ph­ 
¬ng tr×nh ta ®­îc 
nghiÖm cña ph­ 
¬ng tr×nh ®· cho 
lμ x =1; x = 8 tho¶ m·n. VËy m= 3 
0,25 
Câu IV (2,0đ) 
1) 1,0 điểm Vẽ hình đúng theo yêu cầu chung của đề 0,25
WWW.VNMATH.COM 
I 
WWW.VNMATH.COM 
4 
M 
H 
K 
A C 
E 
B 
O 
D 
F 
Gọi K là giao điểm của BO với DF = ΔIKF vuông tại K 0,25 
Có DFE= 1 DOE=450 
2 
0,25 
BIF  450 0,25 
Khi AM = AB thì ΔABM vuông cân tại A = DBH=450 .Có DFH=450 
0,25 
= Tứ giác BDHF nội tiếp 
= 5 điểm B, D, O, H, F cùng thuộc một đường tròn. 0,25 
= BFO=BHO  900 = OH  BM, mà OA  BM = A, O, H thẳng hàng 0,25 
2) 1,0 điểm 
BAH=BIH  450 = Tứ giác ABHI nội tiếp. 0,25 
3) 1,0 điểm 
P 
Q 
N 
C 
B 
A 
O 
D 
E 
F 
M 
Có tứ giác PNQD nội tiếp =  QPN=QDN=EFN . 
Tương tự có NQP=NDP=FEN = ΔNEF và ΔNQP đồng dạng 
0,25
WWW.VNMATH.COM 
  )=3+3  x  x  y  y  z  
z 
                  
   
                  
   
                    
x y x y x y x y x 
y z y z y z y z z 
z y z y z y z y z 
y x y x y x y x x 
x y z y x z x x y y z z x z 
y z y x z x y z x z x y z x 
x z t t t t t t 
z x t t t t 
     
        
Do (2 t  1)( t 
 
1 t  2  2) 
t 
 5 
    
WWW.VNMATH.COM 
5 
= PQ = NQ 1 PQ EF 
EF NE 
   0,25 
Dấu “=” xẩy ra khi và chỉ khi P  F; Q E = DN là đường kính của (O) 
= PQ lớn nhất bằng EF. 
0,25 
Cách xác định điểm M : Kẻ đường kính DN của (O), BN cắt AC tại M thì 
PQ lớn nhất. 
0,25 
Đặt x=1+c, y=1+b, z=1+a do 0  a  b  c 1 = 1  z  y  x  2 
Khi đó A= (x+y+z)( 1 1 1 
x y z 
y z x z x y 
0,25 
1 1 0 1 . 0 1 
. 
1 1 0 1 . 0 1 
. 
2 2 2 
  
0,25 
Đặt x 
z 
= t =1 t  2 
1 2 1 2 2 5 2 5 (2 1)( 2) 5 
2 2 2 2 
2 
 0  x z 
z x 
2 
 
A 3 2. 5 2 10 
2 
0,25 
Câu V (1,0đ) 
Ta thấy khi a=b=0 và c=1 thì A=10 nên giá trị lớn nhất của A là 10 0,25 
WWW.VNMATH.COM
WWW.VNMATH.COM 
x x  
    
x 
x y 
  
  
  
  . Tìm giá trị lớn nhất của biểu thức 
1 1 
2 2 
  
WWW.VNMATH.COM 
6 
SỞ GIÁO DỤC VÀ ĐÀO TẠO 
HẢI DƯƠNG 
KÌ THI TUYỂN SINH LỚP 10 THPT CHUYÊN 
NGUYỄN TRÃI NĂM HỌC 2012- 2013 
Môn thi: TOÁN (không chuyên) 
Thời gian làm bài: 120 phút 
Ngày thi 19 tháng 6 năm 2012 
Đề thi gồm : 01 trang 
Câu I (2,0 điểm) 
1) Giải phương trình 1 1 
3 
  . 
3) Giải hệ phương trình 3 3 3 0 
3 2 11 
   
. 
Câu II ( 1,0 điểm) 
Rút gọn biểu thức P = 1 + 1 : a + 1 
2 a - a 2 - a a - 2 a 
với a  0 và a  4 . 
Câu III (1,0 điểm) 
Một tam giác vuông có chu vi là 30 cm, độ dài hai cạnh góc vuông hơn kém nhau 
7cm. Tính độ dài các cạnh của tam giác vuông đó. 
Câu IV (2,0 điểm) 
Trong mặt phẳng tọa độ Oxy, cho đường thẳng (d): y = 2x -m+1 và parabol (P): y = 1 x2 
2 
. 
1) Tìm m để đường thẳng (d) đi qua điểm A(-1; 3). 
2) Tìm m để (d) cắt (P) tại hai điểm phân biệt có tọa độ (x1; y1) và (x2; y2) sao cho 
  1 2 1 2 x x y + y  48  0 . 
Câu V (3,0 điểm) 
Cho đường tròn tâm O đường kính AB. Trên đường tròn lấy điểm C sao cho AC  
BC (C  A). Các tiếp tuyến tại B và C của (O) cắt nhau ở điểm D, AD cắt (O) tại E (E  A) 
. 
1) Chứng minh BE2 = AE.DE. 
2) Qua C kẻ đường thẳng song song với BD cắt AB tại H, DO cắt BC tại F. Chứng 
minh tứ giác CHOF nội tiếp . 
3) Gọi I là giao điểm của AD và CH. Chứng minh I là trung điểm của CH. 
Câu VI ( 1,0 điểm) 
Cho 2 số dương a, b thỏa mãn 1 1 2 
a b 
4 2 2 4 2 2 
Q 
a  b  ab b  a  
ba 
. 
ĐỀ CHÍNH THỨC
WWW.VNMATH.COM 
x x x x  
  
  
   
  
P= 1 + 1 : a +1 
WWW.VNMATH.COM 
7 
----------------------------Hết---------------------------- 
Họ và tên thí sinh……………………………………. Số báo 
danh………………...………… 
Chữ kí của giám thị 1: ……………………….……… Chữ kí của giám thị 2: 
………………… 
SỞ GIÁO DỤC VÀ ĐÀO TẠO 
HẢI DƯƠNG 
KÌ THI TUYỂN SINH LỚP 10 THPT CHUYÊN 
NGUYỄN TRÃI NĂM HỌC 2012 - 2013 
HƯỚNG DẪN VÀ BIỂU ĐIỂM CHẤM MÔN TOÁN (không chuyên) 
Hướng dẫn chấm gồm : 02 trang 
I) HƯỚNG DẪN CHUNG. 
- Thí sinh làm bài theo cách riêng nhưng đáp ứng được yêu cầu cơ bản vẫn cho đủ điểm. 
- Việc chi tiết điểm số (nếu có) so với biểu điểm phải được thống nhất trong Hội đồng 
chấm. 
- Sau khi cộng điểm toàn bài, điểm lẻ đến 0,25 điểm. 
II) ĐÁP ÁN VÀ BIỂU ĐIỂM CHẤM. 
Câu Nội dung Điểm 
Câu I (2,0đ) 
1   1   1  3(  
1) 
3 
0,25 
 x 1  3x  3 0,25 
  2x  4 0,25 
1) 1,0 điểm 
 x  2.Vậy phương trình đã cho có một nghiệm x = -2 0,25 
    
x 
x y 
3 3 3 0(1) 
3 2 11 (2) 
   
Từ (1)= x 3  3 3 
0,25 
=x=3 0,25 
Thay x=3 vào (2)=3.3 2y 11 =2y=2 0,25 
2) 1,0 điểm 
=y=1 . Vậy hệ phương trình đã cho có nghiệm (x;y)=(3;1) 0,25 
  
a 2- a 2- a a 2 a 
0,25 
a a 
= 1+ a 2 
a 
 
 
a (2  
) a +1 
0,25 
  
  
a a 2 
= 
 
a 2- a 
0,25 
Câu II (1,0đ) 
 
= a 2 
2- a 
=-1 
0,25 
Câu III (1,0đ) Gọi độ dài cạnh góc vuông nhỏ là x (cm) (điều kiện 0 x  15) 
= độ dài cạnh góc vuông còn lại là (x + 7 )(cm) 
0,25
WWW.VNMATH.COM 
Vì chu vi của tam giác là 30cm nên độ dài cạnh huyền là 30–(x + x +7)= 23–2x (cm) 
Theo định lí Py –ta- go ta có phương trình x2 + (x + 7)2 = (23 - 2x)2 0,25 
x2 - 53x + 240 = 0 (1) Giải phương trình (1) được nghiệm x = 5; x = 48 0,25 
Đối chiếu với điều kiện có x = 5 (TM đk); x = 48 (không TM đk) 
Vậy độ dài một cạnh góc vuông là 5cm, độ dài cạnh góc vuông còn lại là 12 cm, độ 
dài cạnh huyền là 30 – (5 + 12) = 13cm 
WWW.VNMATH.COM 
8 
0,25 
Câu IV (2,0đ) 
Vì (d) đi qua điểm A(-1; 3) nên thay x = -1 và y = 3 vào hàm số y = 2x – m + 1 ta có 
2.(-1) – m +1 = 3 
0,25 
 -1 – m = 3 0,25 
 m = -4 0,25 
1) 1,0 điểm 
Vậy m = -4 thì (d) đi qua điểm A(-1; 3) 0,25 
Hoành độ giao điểm của (d) và (P) là nghiệm của phương trình 1 x2 2 1 
2 
 x m 
0,25 
x2  4x  2m 20 (1) ; Để (d) cắt (P) tại hai điểm phân biệt nên (1) có hai 
nghiệm phân biệt ' 062m 0m  3 
0,25 
Vì (x1; y1) và (x2; y2) là tọa độ giao điểm của (d) và (P) nên x1; x2 là nghiệm của 
phương trình (1) và 1 1 y = 2x m1, 2 2 y = 2x m1 
Theo hệ thức Vi-et ta có 1 2 1 2 x + x = 4, x x = 2m-2 .Thay y1,y2 vào 
  1 2 1 2 x x y +y  48  0 có   1 2 1 2 x x 2x +2x -2m+2  48  0 
(2m - 2)(10 - 2m) + 48 = 0 
0,25 
2) 1,0 điểm 
 m2 - 6m - 7 = 0m=-1(thỏa mãn m3) hoặc m=7(không thỏa mãn m3) 
Vậy m = -1 thỏa mãn đề bài 
0,25 
Câu V (3,0đ) 
Vẽ đúng hình theo yêu cầu chung của đề bài 
0,25 
E 
D 
C 
A O 
B 
VìBD là tiếp tuyến của (O) nên BD  OB = ΔABD vuông tại B 0,25 
Vì AB là đường kính của (O) nên AE  BE 0,25 
1) 1,0 điểm 
Áp dụng hệ thức lượng trong ΔABD (ABD=900 ;BE  AD) ta có BE2 = AE.DE 0,25
WWW.VNMATH.COM 
  
1  
1 (2) 
C 
   . 
     mà a  b  2 ab ab 1 2 
Q  . Vậy giá trị lớn nhất của biểu thức là 1 
WWW.VNMATH.COM 
9 
Có DB= DC (t/c hai tiếp tuyến cắt nhau), OB = OC (bán kính 
của (O)) 
= OD là đường trung trực của đoạn BC = OFC=900 (1) 
0,25 
Có CH // BD (gt), mà AB  BD (vì BD là tiếp tuyến của (O)) 0,25 
= CH  AB = OHC=900 (2) 0,25 
2) 1,0 điểm 
Từ (1) và (2) ta có OFC+ OHC = 1800 = tứ giác CHOF nội tiếp 0,25 
Có CH //BD=HCB=CBD (hai góc ở vị trí so le trong) mà 
ΔBCD cân tại D = CBD  DCB nên CB là tia phân giác của HCD 
0,25 
do CA  CB = CA là tia phân giác góc ngoài đỉnh C của ΔICD AI = CI 
 (3) 
AD CD 
0,25 
Trong ΔABDcó HI // BD = AI = HI 
AD BD 
(4) 
0,25 
3)1,0 điểm 
Từ (3) và (4) = CI = HI 
CD BD 
mà CD=BDCI=HI I là trung điểm của CH 
0,25 
Với a  0;b  0 ta có: (a2 b)2  0 a4  2a2b  b2  0a4  b2  2a2b 
a4  b2  2ab2  2a2b  2ab2 1 1 (1) 
a 4  b 2  2ab 2 2ab  a  
b 
 
0,25 
Tương tự có b 4  a 2  2a 2 b 2ab  a  
b 
 
Q 1 
. Từ (1) và (2)   
ab  a  
b 
 
0,25 
Vì 1 1 2 a b 2ab 
a b 
1 1 
2( ) 2 
Q 
ab 
0,25 
Câu VI 
(1,0đ) 
Khi a = b = 1 thì 1 
2 
2 
0,25 
E 
I 
F 
D 
A H O 
B
ĐỀ CHÍNH THỨC 
A x x x 
   
   
x x x x 
x  x    x  x  
4 1 1 
     
    
1 
SỞ GIÁO DỤC VÀ ĐÀO TẠO 
HẢI PHÒNG 
KÌ THI TUYỂN SINH LỚP 10 THPT CHUYÊN 
NĂNG KHIẾU TRẦN PHÚ NĂM HỌC 2012- 2013 
Môn thi: TOÁN (chuyên)Thời gian làm bài: 150 phút 
Ngày thi 25 tháng 6 năm 2012 
Đề thi gồm : 01 trang 
Câu I (2,0 điểm) 
1) Cho 15 11 3 2 2 3 
2 3 1 3 
    
Rút gọn và tìm giá trị lớn nhất của A 
2) Cho phương trình x2  ax  b  0 có hai nghiệm nguyên dương biết a,b là hai số 
dương thỏa mãn 5a + b = 22.Tìm hai nghiệm đó. 
Câu II ( 2,0 điểm) 
1) Giải phương trình:4 2 6 1 3 16 4 4 2 1 
3 
2) Giải hệ phương trình: 
2 
2 2 
4 
x x 
y 
y y xy 
a 4b 9c Câu III (1,0 điểm) Cho ba số dương a,b,c .Chứng minh rằng:    
4 
b  c c  a a  
b 
Câu IV (2,0 điểm) Cho tam giác ABC ( AB  AC) có trực tâm H, nội tiếp đường tròn tâm O, 
đường kính AA’.Gọi AD là đường phân giác trong của góc BAC (DBC).M,I lần lượt là 
trung điểm của BC và AH. 
1) Lấy K đối xứng với H qua AD.Chứng minh K thuộc đường thẳng AA’. 
2) Gọi P là giao điểm của AD với HM.Đường thẳng HK cắt AB và AC lần lượt tại Q 
và R.Chứng minh rằng Q và R lần lượt là hình chiếu vuông góc của P lên AB,AC. 
Câu V (3,0 điểm) 
1) Tìm nghiệm nguyên của phương trình x4  y4  z4  2012 
2) Cho hình vuông 12x12, được chia thành lưới các hình vuông đơn vị. Mỗi đỉnh của 
hình vuông đơn vị này được tô bằng một trong hai màu xanh đỏ. Có tất cả 111 đỉnh màu đỏ. 
Hai trong số những đỉnh màu đỏ này nằm ở đỉnh hình vuông lớn, 22 đỉnh màu đỏ khác nằm 
trên cạnh cạnh của hình vuông lớn (không trùng với đỉnh của hình vuông lớn ) hình vuông đơn 
vị được tô màu theo các quy luật sau: cạnh có hai đầu mút màu đỏ được tô màu đỏ, cạnh có hai 
đầu mút màu xanh được tô màu xanh, cạnh có một đầu mút màu đỏ và một đầu mút màu xanh 
thì được tô màu vàng. Giả sứ có tất cả 66 cạnh vàng. Hỏi có bao nhiêu cạnh màu xanh. 
----------------------------Hết---------------------------- 
Họ và tên thí sinh……………………………………. Số báo danh………………...………… 
Chữ kí của giám thị 1: ……………………….……… Chữ kí của giám thị 2: ………………… 
Từ :Nguyễn Hồng Vân – THPT Trần Hưng Đạo – Hải Phòng- http://trakhuc66.violet.vn/
A x x x x x 
       
 x  x   x  x    x  x  x  x  
Dễ thấy 4x2  2x 1 3x2  (x 1)2  0,x4x2  2x 1 3x2  (x 1)2  0,x nên đặt 
a  4x2  2x 1,b  4x2  2x 1  b,a  0,b  0 
Ta có phương trình 2 2 2 3 
 
           
a 
b x x x 
a x x TM 
b 
3 
2 4 2 1 1 1 
3 4 2 1 3 2 ,( ) 
3 
2 
Lời giải một số câu 
Câu I 
1) 15 11 3 2 2 3 
A x x x 
   
   
x x x x 
2 3 1 3 
    
15 11 (3 2)( 3) (2 3)( 1) 
x x 
( 1)( 3) 
  
  
5 17 
3 
A 
x 
    
 
, A lớn nhất  x  0 khi đó A lớn nhất bằng 2 
3 
. 
2) Gọi x1, x2 là hai nghiệm nguyên dương của phương trình (x1  x2) 
Ta có a = –x1 – x2 và b = x1x2 nên 
5(–x1 – x2) + x1x2 = 22 
 x1(x2 – 5) – 5(x2 – 5) = 47 
 (x1 – 5)(x2 – 5) = 47 (*) 
Vì 1 1 x Z x 1 nên với giả sử 1 2 x  x 
Ta có: –4 ≤ x1 – 5  x2 – 5 nên 
(*)  1 
x 5 1 
x 5 47 
   2 
  
 
x 6 
x 52 
  
  
 1 
2 
. 
Khi đó: a = – 58 và b = 312 thoả 5a + b = 22. Vậy hai nghiệm cần tìm là x1 = 6; x2 = 52. 
Câu II: 
1) 
4 x 2  6 x  1   3 16 x 4  4 x 2  
1 
3 
2(4 2 2 1) (4 2 2 1) 3 (4 2 2 1)(4 2 2 1) 
3 
a  b   ab 
3 
6a2  3ab  3b2  0 
 6(a)2  3(a)  3  
0 
b b 
2 
2 
   
  
2)Giải hệ phương trình
        
Nếu y = 0 thì (2) vô lí nên y  0 vậy     
 
2 
  và (1 ;2) 
      
b c a c a b b  c a  c a  
b 
           
b c a c a b b c a c a b 
3 
4 2 
1 1 (1) 
2 2 
4 (2) 
x x 
y 
y y xy 
(2) 1 1 x 4 
y y 
Đặt 1 b 
y 
 ta có hệ 
 4 x 2 
 x  b 
 1 (1') 
 4 b 2 
 b  x 
 
1 (2') 
Lấy ( 1’) – ( 2’) ta có (x-b) (2x+2b-1) = 0 
1 *) Nếu x = b  
ta có hai nghiệm ( , 2) 
2 
2 
*) Nếu 2x + 2b = 1 thì hệ vô nghiệm 
Vậy hệ có hai nghiệm ( 1 , 2) 
  và (1 ;2) 
2 
2 
Câu V 
1) 
Giả sử một số nguyên là số chẵn có dạng 2k thì (2k)4 16k4  0(mod8) 
Nếu Số nguyên là số nguyên lẻ có dạng 2k + 1 thì 
(2k 1)4  (4t 1)2 16h 11(mod8) nên với k ,t,h là các số nguyên 
x, y, zZ  x4  y4  z4  0,1,2,3(mod8) 
Nhưng 2012  4 ( mod 8) 
Vậy phương trình đã cho không có nghiệm nguyên. 
2) Có 111 đỉnh màu đỏ,trong đó có 22 đỉnh nằm trên cạnh của hình vuông,, 87 đỉnh nằm 
lọt trong hình vuông lớn.Từ đó ta thấy có hai điểm màu xanh ở hai góc của hỉnh vuông 
lớn, 22 điểm màu xanh trên các cạnh của hình vuông lớn không nằm trên đỉnh của hình 
vuông lớn còn lại có 34 điểm màu xanh nằm lọt trong hình vuông.Với 312 cạnh của cả 
hình, ta cho đình của mỗi cạnh như sau: trong 2 mút của nó có i điểm màu xanh thì cho 
i điểm.Gọi tổng số điểm là S, ta có S = 2 ( số cạnh màu xanh) + số cạnh vàng.Ta lại có 
thể đếm số S theo cách khác:Mỗi điểm xanh ở góc là mút của hai đoạn, các điểm còn lại 
là mút của 4 đoạn.Vậy S = 2 x 2 + 22 x 3+ 34 x 4 = 206, suy ra số cạnh xanh là : ( 206 – 
66):2 = 70 cạnh màu xanh. 
Câu III: Chứng minh rằng: 
a  4b  9c  
4 
b  c c  a a  
b 
(a b c)( 1 4 9 ) 18 
b  c a  c a  
b 
Thật vậy: 
[( ) ( ) )]( 1 4 9 ) ( 4( ) 9( ) )2 36 
( ) ( ) 
      
(a b c)( 1 4 9 ) 18 
      
b  c a  c a  
b 
Điều phải chứng minh 
.
Bài hình: 1) Tam giác ABA’ có: ABC A'BC  900,ABC  BAN A'BC  BAN 
4 
P 
R 
Q 
K 
M 
I 
H 
D 
o 
A 
A' 
B 
C 
Lại có 
  ' ' A AC A BC  ( cùng chắn cung ' 
A C ) nên  ' 
BAN  A AC 
Cũng có BAD  CAD BAD  BAN  CAD CAN  
Mặt khác H đối xứng với K qua AD HAD  KAD , H thuộc AN nên K thuộc AA’ 
2) Bạn tự giải nhé.
1 
ĐẠI HỌC QUỐC GIA TH HCM ĐỀ THI TUYỂN SINH VÀO LỚP 10 NĂM 2012 
TRƯỜNG LÊ HỒNG PHONG TP HCM MÔN : TOÁN ( Chuyên) 
Câu 1: Giải phương trình : 8x 1  46 10x  x3  5x2  4x 1 
Câu 2: Cho đa thức f(x) = ax3 + bx2 + cx + d. với a là số nguyên dương, biết: f(5) – f(4) = 2012 . 
Chứng minh: f(7) – f(2) là hợp số. 
2 2 2 A 14 a b c ab bc ca 
Câu 3: Cho ba số dương a; b và c thỏa a + b + c = 1. Tìm GTNN của :  2 2 2  
  
a b b c c a 
    
  
Câu 4:Cho tứ giác ABCD nội tiếp (O; R) có AC vuông góc BD tại H. Trên cạnh AB lấy điểm M sao cho: 
AM = 1/3 AB. Trên cạnh HC lấy trung điểm N. chứng minh MH vuông góc với DN. 
Câu 5: Cho đường tròn tâm O và đường tròn tâm I cắt nhau tại hai điểm A và B(O và I khác phía đối với A và B). 
IB cắt (O) tại E: OB cắt (I) tại F. Qua B vẽ MN // EF( M thuộc (O) và N thuộc (I). 
a) Chứng minh :Tứ giác OAIE nội tiếp ; 
b) Chứng minh :AE + AF = MN 
Câu 6: Trên mặt phẳng cho 2013 điểm tùy ý sao cho khi 3 điểm bất kỳ thì tồn tại 2 điểm mà khoảng cách giữa 2 
điểm đó luôn bé hơn 1. Chứng minh rằng tồn tại một đường tròn có bán kính bằng 1 chứa ít nhất 1007 điểm( kể cả 
biên). 
…………………………………. Hết ………………………………….
2 
GỢI Ý 
Câu 1: 
Giải phương trình : 8x 1  46 10x  x3  5x2  4x 1 
 
Điều kiện : 1  x  
46 
8 10 
3 2 3 2 
x x x x x x x x x x 
8  1  46  10    5  4  1  8  1  3  46  10  6    5  4  
8 
 8 x  1  3  8 x  1  3   46  10 x  6  46  10 x 
 
6 
    
       
      
    
      
  
  
2 
2 
     2 
  
1 4 8 
8 1 3 46 10 6 
8 1 10 1 
1 4 8 
   
8 1 3 46 10 6 
1 0 1 
    
   
8 10 4 8 2 
     
8 1 3 46 10 6 
x x x 
x x 
x x 
x x x 
x x 
x 
x x 
x x 
Từ (1) suy ra: x = 1 . 
Từ (2), ta có : x2 – 4x + 8 = (x – 2)2 + 4  4 với mọi x 
46  10  0  46  10  6  6  10  10  
5 
46 10 6 6 3 
x x 
x 
  
10  
suy ra :  8  10  8  
5 
46  10 x  6 8 x  1  3 46  10 x  6 8 x 
 1  
3 3 
Vậy : 10  
 8  2  4  
8 
46 10 6 8 1 3 
x x 
x x 
    
, với mọi x. 
Suy ra phương trình có nghiệm duy nhất : x = 1. 
Câu 2: 
Cho đa thức f(x) = ax3 + bx2 + cx + d. với a là số nguyên dương, biết: f(5) – f(4) = 2012 . 
Chứng minh: f(7) – f(2) là hợp số. 
Ta có : 
f(5) – f(4) = 2012  (125a + 25b + 5c + d) – ( 64a + 16b + 4c + d) = 2012  61a + 9b + c = 2012. 
f(7) – f(2) = (343a + 49b + 7c + d) – ( 8a + 4b + 2c + d) = 335a + 45b + 5c 
= 305a + 45b + 5c +30a = 5(61a + 9b + c) + 30a = 2012 + 30a = 2( 1006 + 15a) 
Vì a là số nguyên nên ta được : 2( 1006 + 15a) chia hết cho 2. 
Vậy f(7) – f(2) là hợp số 
2 2 2 A 14 a b c ab bc ca 
Câu 3: Cho ba số dương a; b và c thỏa a + b + c = 1. Tìm GTNN của :  2 2 2  
  
a b b c c a 
    
  
Ta có : (a + b + c)2 = a2 + b2 + c2 + 2(ab + bc + ca)  
1  a 2 b 2 c 
2  
2 
ab bc ca 
   
   
Ta có: a2 + b2 + c2 = (a + b + c) (a2 + b2 + c2) = a3 +b2a+ b3 + bc2 + c3 + ca2 + a2b + b2c + c2a. 
Áp dụng bất đẳng thức Cô – Si: 
a3 + b2a ≥ 2a2b ; b3 + bc2 ≥ 2b2c ; c3 + ca2 ≥ 2c2a , dấu “=” xảy ra khi a = b = c. 
suy ra: a2 + b2 + c2 = a3 +b2a+ b3 + bc2 + c3 + ca2 + a2b + b2c + c2a ≥ 3(a2b + b2c + c2a) 
suy ra: 
   2 2 2 
 
ab bc ca ab bc ca a b c 
1 3 3 3 3 
       
  
2 2 2 2 2 2 2 2 2 2 2 2 2 2 2 
a b b c c a a b c a b b c c a a b c 2 
a b c 
    
          
Đặt : t = a2 + b2 + c2, ta có : 3(a2 + b2 + c2) ≥ (a + b + c)2 = 1  t ≥ 1 
3 
, dấu “=” xảy ra khi a = b = c = 1 
3 
. 
t t t t t t 
 
Ta được : A = 14 3 3 28 3 3 27 3 3 
        . 
t t t t 
2 2 2 2 2 2 2 2
3 
t t 
   dấu “=” xảy ra khi : t = 1 
Áp dụng bất đẳng thức Cô – Si : 27 3 2 27 . 3 9 
t t 
2 2 2 2 
3 
. 
t       t     
Mặt khác : 3 1 3 4 vì : 1 
2 2 6 2 3 3 
  
Suy ra: A 9 4 23 
   dấu “=” xảy ra khi : a2 + b2 + c2 = 1 
3 3 
3 
và a = b = c suy ra: a = b = c = 1 
3 
. 
Vậy A đạt giá trị nhỏ nhất bằng 23 
3 
, khi a= b = c = 1 
3 
. 
Câu 4: 
Cho tứ giác ABCD nội tiếp (O; R) có AC vuông góc BD tại H. Trên cạnh AB lấy điểm M sao cho: 
AM = 1/3 AB. Trên cạnh HC lấy trung điểm N. chứng minh MH vuông góc với DN 
+ Gọi E; F lần lượt là trung điểm của HB và MB, 
Suy ra: AM = MF = FB = 1/3 AB. 
+ Gọi K và G lần lượt là giao điểm của MH với DN và AE. 
+ Ta có:  AHB ~  DHC = AH : HB = DH : HC 
= AH : (2HE) = DH : ( 2HN)  AH : HE = DH : HN 
=  AHE ~  DHN = NDH  EAH 
+ Ta có : EF là đường trung bình của tam giác HMB = HM // EF 
+ Xét  AEF : AM = MF và MG // EF = AG = GE. 
+ Xét  AEH: vuông tại H có G là trung điểm của AE, suy ra: 
AG = HG = EG =  AHG cân tại G = AHG  EAH 
+ Ta có : KDH DHK  EAH DHK  AHG  DHK  900 , suy ra  DHK vuông tại K. 
Vậy MH vuông góc với DN.(đpcm) 
Câu 5: 
Cho đường tròn tâm O và đường tròn tâm I cắt nhau tại hai điểm A và B(O và I khác phía đối với A và B). 
IB cắt (O) tại E: OB cắt (I) tại F. Qua B vẽ MN // EF( M thuộc (O) và N thuộc (I). 
a) Chứng minh :Tứ giác OAIE nội tiếp ; 
b) Chứng minh :AE + AF = MN 
a) 
+  BOE cân tại O = OBE  OEB; 
+  BIF cân tại I = IBF  IFB; 
Do : OBE  IBFOEB  IFB , suy ra: tứ giác OIFE nội tiếp. 
+ Do : AOI = BOI ( c – c – c) = OAI  OBI 
+ Ta có : 
OAI OEI  OBI OBE 1800 , suy ra tứ giác AOEI nội tiếp 
Vậy 5 điểm O; A; I; E; F nằm trên cùng một đường tròn. 
Vậy Tứ giác OAIE nội tiếp được. 
b) 
+ Xét đường tròn (O) : AMB FOI 1 Sd AB 
2 
  
A 
+ Do : MN // EF ta được : BEF  MBE ( slt) 
+ Do 5 điểm O; A; I; E; F nằm trên cùng một đường tròn, suy ra: BEF  FOI 
Suy ra: AMB  FOI  BEF  MBE suy ra: AM // EB. 
Vậy tứ giác MABE là hình thang và nội tiếp đường tròn (O) suy ra: MABE là hình thang cân = MB = AE. 
+ Chứng minh tương tự ta được : NB = AF, suy ra: AE + AF = MB + NB = MN. ( đpcm). 
Câu 6: 
Trên mặt phẳng cho 2013 điểm tùy ý sao cho khi 3 điểm bất kỳ thì khoảng cách giữa hai điểm luôn bé hơn 
1. Chứng minh rằng tồn tại một đường tròn có bán kính bằng 1 chứa ít nhất 1007 điểm( kể cả biên). 
Gọi các điểm là : A1; A2; A3; …; Ai; Ai + 1 ; A2012; A2013. Ta chia các cặp điểm như sau: (A1; A2013); 
( A2; A2012); …( Ai; A2013 – i)…;(A1006; A1008) , và điểm A1007. 
O 
G 
K 
F 
E 
N 
H 
M 
D 
C 
B 
A 
O I 
F 
E 
N 
M B
4 
Xét điểm A1007 với các cặp điểm đã cho, theo giả thiết trong mỗi cặp điểm tồn tại một điểm Am sao 
cho đoạn thẳng A1007Am có độ dài nhỏ hơn 1. Không mất tính tổng quát giả sử các điểm A1; A2; …; A1006 
có khoảng cách đến điểm A1007 nhỏ hơn 1, suy ra các điểm A1; A2; …; A1006 nằm trong đường tròn tâm 
A1007 bán kính bằng 1. 
Vậy tồ tại đường tròn có bán kính bằng 1 chứa 1007 điểm trong 2013 điểm đã cho. (đpcm). 
Rất mong nhận được sự góp ý của các bạn. 
Giáo viên : 
Hà Gia Có – trường THCS Lý Thường Kiệt – Định Quán – Đồng Nai.
TUYỂN TẬP ĐỀ TOÁN – VÀO 10 – CHUYÊN LAM SƠN – THANH HOÁ
TUYỂN TẬP ĐỀ TOÁN – VÀO 10 – CHUYÊN LAM SƠN – THANH HOÁ
TUYỂN TẬP ĐỀ TOÁN – VÀO 10 – CHUYÊN LAM SƠN – THANH HOÁ
TUYỂN TẬP ĐỀ TOÁN – VÀO 10 – CHUYÊN LAM SƠN – THANH HOÁ
TUYỂN TẬP ĐỀ TOÁN – VÀO 10 – CHUYÊN LAM SƠN – THANH HOÁ
TUYỂN TẬP ĐỀ TOÁN – VÀO 10 – CHUYÊN LAM SƠN – THANH HOÁ 
Đề thi vào lớp 10 chuyên Lê Quý Đôn, TP. Đà Nẵng 
Số :411 - 9/2011
TUYỂN TẬP ĐỀ TOÁN – VÀO 10 – CHUYÊN LAM SƠN – THANH HOÁ
TUYỂN TẬP ĐỀ TOÁN – VÀO 10 – CHUYÊN LAM SƠN – THANH HOÁ
TUYỂN TẬP ĐỀ TOÁN – VÀO 10 – CHUYÊN LAM SƠN – THANH HOÁ
TUYỂN TẬP ĐỀ TOÁN – VÀO 10 – CHUYÊN LAM SƠN – THANH HOÁ
TUYỂN TẬP ĐỀ TOÁN – VÀO 10 – CHUYÊN LAM SƠN – THANH HOÁ 
Lời giải Đề thi vào lớp 10 trường THPT chuyên ĐHSP Hà Nội 
Số :410 - 8/2011
TUYỂN TẬP ĐỀ TOÁN – VÀO 10 – CHUYÊN LAM SƠN – THANH HOÁ
TUYỂN TẬP ĐỀ TOÁN – VÀO 10 – CHUYÊN LAM SƠN – THANH HOÁ
TUYỂN TẬP ĐỀ TOÁN – VÀO 10 – CHUYÊN LAM SƠN – THANH HOÁ
TUYỂN TẬP ĐỀ TOÁN – VÀO 10 – CHUYÊN LAM SƠN – THANH HOÁ 
Đề thi vào lớp 10 chuyên ĐHSP Hà Nội và Giải đề thi vào lớp 10 PTNK, ĐHQG TP. HCM 2010 
– 2011 
Số :409 - 7/2011
TUYỂN TẬP ĐỀ TOÁN – VÀO 10 – CHUYÊN LAM SƠN – THANH HOÁ
TUYỂN TẬP ĐỀ TOÁN – VÀO 10 – CHUYÊN LAM SƠN – THANH HOÁ
TUYỂN TẬP ĐỀ TOÁN – VÀO 10 – CHUYÊN LAM SƠN – THANH HOÁ
TUYỂN TẬP ĐỀ TOÁN – VÀO 10 – CHUYÊN LAM SƠN – THANH HOÁ 
Đề thi vào lớp 10 trường PTNK, ĐHQG TP. HCM, năm học 2010 – 2011 
Số :408 - 6/2011
TUYỂN TẬP ĐỀ TOÁN – VÀO 10 – CHUYÊN LAM SƠN – THANH HOÁ
TUYỂN TẬP ĐỀ TOÁN – VÀO 10 – CHUYÊN LAM SƠN – THANH HOÁ
TUYỂN TẬP ĐỀ TOÁN – VÀO 10 – CHUYÊN LAM SƠN – THANH HOÁ
TUYỂN TẬP ĐỀ TOÁN – VÀO 10 – CHUYÊN LAM SƠN – THANH HOÁ 
Đề thi vào lớp 10 chuyên Quang Trung, Bình Phước vàLời giải Đề thi vào lớp 10 chuyên Lê Quý 
Đôn, Bình Định, năm học 2010 – 2011 
Số :407 - 5/2011
TUYỂN TẬP ĐỀ TOÁN – VÀO 10 – CHUYÊN LAM SƠN – THANH HOÁ
TUYỂN TẬP ĐỀ TOÁN – VÀO 10 – CHUYÊN LAM SƠN – THANH HOÁ
TUYỂN TẬP ĐỀ TOÁN – VÀO 10 – CHUYÊN LAM SƠN – THANH HOÁ 
Đề thi vào lớp 10 chuyên Lê Quý Đôn, Bình Định và giải Đề thi vào lớp 10 chuyên Lê Khiết, 
Quảng Ngãi năm học 2010 – 2011 
Số :406 - 4/2011
TUYỂN TẬP ĐỀ TOÁN – VÀO 10 – CHUYÊN LAM SƠN – THANH HOÁ
TUYỂN TẬP ĐỀ TOÁN – VÀO 10 – CHUYÊN LAM SƠN – THANH HOÁ
TUYỂN TẬP ĐỀ TOÁN – VÀO 10 – CHUYÊN LAM SƠN – THANH HOÁ 
Đề thi vào lớp 10 THPT chuyên Lê Khiết, Quảng Ngãi và giải Đề thi vào lớp 10 THPT chuyên 
Lam Sơn, Thanh Hóa, năm học 2010 – 2011 
Số :405 - 2/2011
TUYỂN TẬP ĐỀ TOÁN – VÀO 10 – CHUYÊN LAM SƠN – THANH HOÁ
TUYỂN TẬP ĐỀ TOÁN – VÀO 10 – CHUYÊN LAM SƠN – THANH HOÁ
TUYỂN TẬP ĐỀ TOÁN – VÀO 10 – CHUYÊN LAM SƠN – THANH HOÁ 
Đề thi vào lớp 10 trường THPT chuyên Lam Sơn, Thanh Hóa vàHướng dẫn giải Đề thi vào lớp 
10 trường THPT chuyên Phan Bội Châu, Nghệ An, năm học 2010 – 2011 
Số :404 - 2/2011
TUYỂN TẬP ĐỀ TOÁN – VÀO 10 – CHUYÊN LAM SƠN – THANH HOÁ
TUYỂN TẬP ĐỀ TOÁN – VÀO 10 – CHUYÊN LAM SƠN – THANH HOÁ
TUYỂN TẬP ĐỀ TOÁN – VÀO 10 – CHUYÊN LAM SƠN – THANH HOÁ 
Đề thi vào lớp 10 THPT chuyên Phan Bội Châu, NA và giải Đề thi chọn học sinh giỏi lớp 9 tỉnh 
Vĩnh Phúc - năm 2010 
Số :403 - 1/2011
TUYỂN TẬP ĐỀ TOÁN – VÀO 10 – CHUYÊN LAM SƠN – THANH HOÁ 
Đề thi chọn học sinh giỏi lớp 9 tỉnh Vĩnh Phúc năm 2010
TUYỂN TẬP ĐỀ TOÁN – VÀO 10 – CHUYÊN LAM SƠN – THANH HOÁ 
Số :402 - 12/2010
TUYỂN TẬP ĐỀ TOÁN – VÀO 10 – CHUYÊN LAM SƠN – THANH HOÁ
TUYỂN TẬP ĐỀ TOÁN – VÀO 10 – CHUYÊN LAM SƠN – THANH HOÁ
TUYỂN TẬP ĐỀ TOÁN – VÀO 10 – CHUYÊN LAM SƠN – THANH HOÁ 
Đề thi vào lớp 10 Khối THPT chuyên ĐH Vinh năm học 2010-2011 và Lời giải Đề HSG lớp 9 
TP.HCM năm 2010 
Số :398 - 8/2010
TUYỂN TẬP ĐỀ TOÁN – VÀO 10 – CHUYÊN LAM SƠN – THANH HOÁ
TUYỂN TẬP ĐỀ TOÁN – VÀO 10 – CHUYÊN LAM SƠN – THANH HOÁ
TUYỂN TẬP ĐỀ TOÁN – VÀO 10 – CHUYÊN LAM SƠN – THANH HOÁ
TUYỂN TẬP ĐỀ TOÁN – VÀO 10 – CHUYÊN LAM SƠN – THANH HOÁ 
Đề thi học sinh giỏi lớp 9 TP. HCM và Lời giải đề vào lớp 10 chuyên toán chuyên Lê Quý Đôn, 
Bình Định, năm học 2009 – 2010 
Số :397 - 7/2010
TUYỂN TẬP ĐỀ TOÁN – VÀO 10 – CHUYÊN LAM SƠN – THANH HOÁ
TUYỂN TẬP ĐỀ TOÁN – VÀO 10 – CHUYÊN LAM SƠN – THANH HOÁ
TUYỂN TẬP ĐỀ TOÁN – VÀO 10 – CHUYÊN LAM SƠN – THANH HOÁ 
Đề vào lớp 10 chuyên Lê Quý Đôn và Lời giả ivào lớp 10 chuyên Quang Trung 
Số :396 - 6/2010
TUYỂN TẬP ĐỀ TOÁN – VÀO 10 – CHUYÊN LAM SƠN – THANH HOÁ
TUYỂN TẬP ĐỀ TOÁN – VÀO 10 – CHUYÊN LAM SƠN – THANH HOÁ
TUYỂN TẬP ĐỀ TOÁN – VÀO 10 – CHUYÊN LAM SƠN – THANH HOÁ 
Đề thi vào lớp 10 chuyên Toán Quang Trung, Bình Phước năm học 2009 - 2010 
Số :395 - 5/2010
TUYỂN TẬP ĐỀ TOÁN – VÀO 10 – CHUYÊN LAM SƠN – THANH HOÁ
TUYỂN TẬP ĐỀ TOÁN – VÀO 10 – CHUYÊN LAM SƠN – THANH HOÁ 
Lời giải đề thi vào lớp 10 chuyên toán Phan Bội Châu, Nghệ An năm học 2009 - 2010 
Số :394 - 4/2010
TUYỂN TẬP ĐỀ TOÁN – VÀO 10 – CHUYÊN LAM SƠN – THANH HOÁ
TUYỂN TẬP ĐỀ TOÁN – VÀO 10 – CHUYÊN LAM SƠN – THANH HOÁ
TUYỂN TẬP ĐỀ TOÁN – VÀO 10 – CHUYÊN LAM SƠN – THANH HOÁ
TUYỂN TẬP ĐỀ TOÁN – VÀO 10 – CHUYÊN LAM SƠN – THANH HOÁ 
Đề thi vào lớp 10 chuyên toán THPT chuyên Phan Bội Châu năm học 2009-2010 
Số :393 - 3/2010
TUYỂN TẬP ĐỀ TOÁN – VÀO 10 – CHUYÊN LAM SƠN – THANH HOÁ
TUYỂN TẬP ĐỀ TOÁN – VÀO 10 – CHUYÊN LAM SƠN – THANH HOÁ
TUYỂN TẬP ĐỀ TOÁN – VÀO 10 – CHUYÊN LAM SƠN – THANH HOÁ 
Đề thi vào lớp 10 chuyên toán Lê Hồng Phong, Nam Định và Lời giải Đề thi vào lớp 10 chuyên 
toán Vĩnh Phúc, năm học 2009 - 2010 
Số :392 - 2/2010
TUYỂN TẬP ĐỀ TOÁN – VÀO 10 – CHUYÊN LAM SƠN – THANH HOÁ
TUYỂN TẬP ĐỀ TOÁN – VÀO 10 – CHUYÊN LAM SƠN – THANH HOÁ
TUYỂN TẬP ĐỀ TOÁN – VÀO 10 – CHUYÊN LAM SƠN – THANH HOÁ
TUYỂN TẬP ĐỀ TOÁN – VÀO 10 – CHUYÊN LAM SƠN – THANH HOÁ
TUYỂN TẬP ĐỀ TOÁN – VÀO 10 – CHUYÊN LAM SƠN – THANH HOÁ
TUYỂN TẬP ĐỀ TOÁN – VÀO 10 – CHUYÊN LAM SƠN – THANH HOÁ
TUYỂN TẬP ĐỀ TOÁN – VÀO 10 – CHUYÊN LAM SƠN – THANH HOÁ 
Lời giải Đề thi vào lớp 10 chuyên toán Lam Sơn, Thanh Hóa và Đề thi vào lớp 10 chuyên toán 
Vĩnh Phúc, năm học 2009 – 2010 
Số :391 - 1/2010
TUYỂN TẬP ĐỀ TOÁN – VÀO 10 – CHUYÊN LAM SƠN – THANH HOÁ
TUYỂN TẬP ĐỀ TOÁN – VÀO 10 – CHUYÊN LAM SƠN – THANH HOÁ
TUYỂN TẬP ĐỀ TOÁN – VÀO 10 – CHUYÊN LAM SƠN – THANH HOÁ
TUYỂN TẬP ĐỀ TOÁN – VÀO 10 – CHUYÊN LAM SƠN – THANH HOÁ 
Giải Đề thi vào lớp 10 chuyên Lê Khiết, Quảng Ngãi vàĐề thi vào lớp 10 chuyên Toán Lam 
Sơn, Thanh Hóa, năm học 2009 - 2010 
Số :390 - 12/2009
TUYỂN TẬP ĐỀ TOÁN – VÀO 10 – CHUYÊN LAM SƠN – THANH HOÁ
TUYỂN TẬP ĐỀ TOÁN – VÀO 10 – CHUYÊN LAM SƠN – THANH HOÁ
TUYỂN TẬP ĐỀ TOÁN – VÀO 10 – CHUYÊN LAM SƠN – THANH HOÁ 
THI HỌC SINH GIỎI VÀ TRƯỜNG CHUYÊN 
Số :389 - 11/2009
TUYỂN TẬP ĐỀ TOÁN – VÀO 10 – CHUYÊN LAM SƠN – THANH HOÁ
TUYỂN TẬP ĐỀ TOÁN – VÀO 10 – CHUYÊN LAM SƠN – THANH HOÁ 
THI HỌC SINH GIỎI VÀ TRƯỜNG CHUYÊN 
Số :389 - 11/2009
TUYỂN TẬP ĐỀ TOÁN – VÀO 10 – CHUYÊN LAM SƠN – THANH HOÁ
TUYỂN TẬP ĐỀ TOÁN – VÀO 10 – CHUYÊN LAM SƠN – THANH HOÁ 
CÁC KÌ THI HỌC SINH GIỎI VÀ TRƯỜNG CHUYÊN 
Số :388 - 10/2009
TUYỂN TẬP ĐỀ TOÁN – VÀO 10 – CHUYÊN LAM SƠN – THANH HOÁ
TUYỂN TẬP ĐỀ TOÁN – VÀO 10 – CHUYÊN LAM SƠN – THANH HOÁ
TUYỂN TẬP ĐỀ TOÁN – VÀO 10 – CHUYÊN LAM SƠN – THANH HOÁ
TUYỂN TẬP ĐỀ TOÁN – VÀO 10 – CHUYÊN LAM SƠN – THANH HOÁ
TUYỂN TẬP ĐỀ TOÁN – VÀO 10 – CHUYÊN LAM SƠN – THANH HOÁ
TUYỂN TẬP ĐỀ TOÁN – VÀO 10 – CHUYÊN LAM SƠN – THANH HOÁ
TUYỂN TẬP ĐỀ TOÁN – VÀO 10 – CHUYÊN LAM SƠN – THANH HOÁ
TUYỂN TẬP ĐỀ TOÁN – VÀO 10 – CHUYÊN LAM SƠN – THANH HOÁ 
CÁC KÌ THI HỌC SINH GIỎI VÀ TRƯỜNG CHUYÊN 
Số :387 - 9/2009
TUYỂN TẬP ĐỀ TOÁN – VÀO 10 – CHUYÊN LAM SƠN – THANH HOÁ
TUYỂN TẬP ĐỀ TOÁN – VÀO 10 – CHUYÊN LAM SƠN – THANH HOÁ
TUYỂN TẬP ĐỀ TOÁN – VÀO 10 – CHUYÊN LAM SƠN – THANH HOÁ 
Lời giải đề thi vào Lớp 10 Chuyên Tóan Lam Sơn, Thanh Hóa và Đề thi vào Lớp 10 Chuyên 
Toán Trần Phú, Hải Phòng 
Số :386 - 8/2009
TUYỂN TẬP ĐỀ TOÁN – VÀO 10 – CHUYÊN LAM SƠN – THANH HOÁ
TUYỂN TẬP ĐỀ TOÁN – VÀO 10 – CHUYÊN LAM SƠN – THANH HOÁ
TUYỂN TẬP ĐỀ TOÁN – VÀO 10 – CHUYÊN LAM SƠN – THANH HOÁ 
CÁC KÌ THI HỌC SINH GIỎI VÀ TRƯỜNG CHUYÊN 
Số :385 - 7/2009
TUYỂN TẬP ĐỀ TOÁN – VÀO 10 – CHUYÊN LAM SƠN – THANH HOÁ
TUYỂN TẬP ĐỀ TOÁN – VÀO 10 – CHUYÊN LAM SƠN – THANH HOÁ
TUYỂN TẬP ĐỀ TOÁN – VÀO 10 – CHUYÊN LAM SƠN – THANH HOÁ 
Đề thi vào Lớp 10 Khối chuyên THPT ĐHSP Hà Nội năm học 2008-2009 Vòng 2 và Lời giải 
Vòng 1. 
Số :384 - 6/2009
TUYỂN TẬP ĐỀ TOÁN – VÀO 10 – CHUYÊN LAM SƠN – THANH HOÁ
TUYỂN TẬP ĐỀ TOÁN – VÀO 10 – CHUYÊN LAM SƠN – THANH HOÁ
TUYỂN TẬP ĐỀ TOÁN – VÀO 10 – CHUYÊN LAM SƠN – THANH HOÁ
TUYỂN TẬP ĐỀ TOÁN – VÀO 10 – CHUYÊN LAM SƠN – THANH HOÁ
TUYỂN TẬP ĐỀ TOÁN – VÀO 10 – CHUYÊN LAM SƠN – THANH HOÁ
TUYỂN TẬP ĐỀ TOÁN – VÀO 10 – CHUYÊN LAM SƠN – THANH HOÁ
1 
Së gi¸o dôc vμ ®μo t¹o 
H­ng 
yªn 
®Ò chÝnh thøc 
(§Ò thi cã 01 trang) 
kú thi tuyÓn sinh vμo líp 10 thpt chuyªn 
N¨m häc 2012 - 2013 
M«n thi: To¸n 
(Dμnh cho thÝ sinh dù thi c¸c líp chuyªn: To¸n, Tin) 
Thêi gian lμm bμi: 150 phót 
Bài 1: (2 điểm) 
a) Cho A = 20122  20122.20132  20132 . Chứng minh A là một số tự nhiên. 
b) Giải hệ phương trình 
x 1 x 3 
 
 2 
   y 2 
y 
x  1  x  
3 
y y 
 
Bài 2: (2 điểm) 
a) Cho Parbol (P): y = x2 và đường thẳng (d): y = (m +2)x – m + 6. Tìm m để đường thẳng (d) cắt 
Parabol (P) tại hai điểm phân biệt có hoành độ dương. 
b) Giải phương trình: 5 + x + 2 (4  x)(2x  2)  4( 4  x  2x  2) 
Bài 3: (2 điểm) 
a) Tìm tất cả các số hữu tỷ x sao cho A = x2 + x+ 6 là một số chính phương. 
b) Cho x  1 và y  1. Chứng minh rằng : 
(x3  y3)  (x2  
y2 ) 8 
(x 1)(y 1) 
 
  
Bài 4 (3 điểm) 
Cho tam giác ABC nhọn nội tiếp đường tròn tâm O, đường cao BE và CF. Tiếp tuyến tại B và C cắt nhau tại S, 
gọi BC và OS cắt nhau tại M 
a) Chứng minh AB. MB = AE.BS 
b) Hai tam giác AEM và ABS đồng dạng 
c) Gọi AM cắt EF tại N, AS cắt BC tại P. CMR NP vuông góc với BC 
Bài 5: (1 điểm) 
Trong một giải bóng đá có 12 đội tham dự, thi đấu vòng tròn một lượt (hai đội bất kỳ thi đấu với nhau đúng một 
trận). 
a) Chứng minh rằng sau 4 vòng đấu (mỗi đội thi đấu đúng 4 trận) luôn tìm được ba đội bóng đôi một chưa thi 
đấu với nhau. 
b) Khẳng định trên còn đúng không nếu các đội đã thi đấu 5 trận? 
HƯỚNG DẪN GIẢI 
Bài 1: (2 điểm)
2 
a) Cho A = 20122  20122.20132  20132 
Đặt 2012 = a, ta có 20122  20122.20132  20132  a2  a2 (a 1)2  (a 1)2 
 (a2  a 1)2  a2  a 1 
b) Đặt 
x a 
y 
x 1 b 
y 
   
  
 
Ta có 
x 1 x 3 
 
 2 
   y 2 
y 
x  1  x  
3 
y y 
 
  
         
2 x 1 x 3 
y y 
x 1 x 3 
 
   y y 
nên 
       
   
      
b2 a 3 b2 b 6 0 
b a 3 b a 3 
a 6 a 1 
    
 v 
     
 b 3 b 2 
Bài 2: 
a) ycbt tương đương với PT x2 = (m +2)x – m + 6 hay x2 - (m +2)x + m – 6 = 0 có hai nghiệm dương 
phân biệt. 
b) Đặt t = 4  x  2x  2 
Bài 3: 
a) x = 0, x = 1, x= -1 không thỏa mãn. Với x khác các giá trị này, trước hết ta chứng minh x phải là số 
nguyên. 
+) x2 + x+ 6 là một số chính phương nên x2 + x phải là số nguyên. 
m 
+) Giả sử x  với m và n có ước nguyên lớn nhất là 1. 
n 
Ta có x2 + x = 
2 2 
2 2 
m m m  
mn 
n n n 
  là số nguyên khi m2 mn chia hết cho n2 
nên m2 mn chia hết cho n, vì mn chia hết cho n nên m2 chia hết cho n và do m và n có ước nguyên 
lớn nhất là 1, suy ra m chia hết cho n( mâu thuẫn với m và n có ước nguyên lớn nhất là 1). Do đó x 
phải là số nguyên. 
Đặt x2 + x+ 6 = k2 
Ta có 4x2 + 4x+ 24 = 4 k2 hay (2x+1)2 + 23 = 4 k2 tương đương với 4 k2 - (2x+1)2 = 23 
(x3  y3)  (x2  y2 ) x2 (x  1)  y2 (y  
1) 
 
(x  1)(y  1) (x  1)(y  
1) 
= 
x2  
y2 
y  1 x  
1 
(x  1)2  2(x  1)  1 (y  1)2  2(y  1)  
1 
  
y  1 x  
1 
  2  2     (x 1) (y 1) 2(y 1)  2(x  1)   1  
     1 
 y  1 x  1   x  1 y  1     y  1 x  1 
  
. 
Theo BĐT Côsi
3 
(x  1)2 (y  2  1) 2 (x  1)2 . (y  
1)2  2 (x  1)(y  
1) 
y 1 x 1 y 1 x 1 
    
2(y  1) 2(x   1)  2(y  1)  
. 2(x 1)  
4 
x 1 y 1 x 1 y 1 
    
1  1  
2 1 . 1 
y 1 x 1 y 1 x 1 
    
2 1 . 1 (x 1)(y 1) 2.2 1 . 1 . (x 1)(y 1) 4 
  
         y  1 x  1   y  1 x  
1 
Nên có đpcm 
Bài 4 
a) Suy ra từ hai tam giác đồng dạng là ABE và BSM 
b) Từ câu a) ta có AE MB 
 (1) 
AB BS 
Mà MB = EM( do tam giác BEC vuông tại E có M là trung điểm của BC 
Nên AE  
EM 
AB BS 
Có MOB  BAE,EBA  BAE  900 ,MBOMOB  900 
Nên MBO  EBA do đó MEB  OBA( MBE) 
Suy ra MEA  SBA(2) 
Từ (1) và (2) suy ra hai tam giác AEM và ABS đồng dạng(đpcm.) 
c) Dễ thấy SM vuông góc với BC nên để chứng minh bài toán ta chứng minh NP //SM. 
+ Xét hai tam giác ANE và APB: 
Từ câu b) ta có hai tam giác AEM và ABS đồng dạng nên NAE  PAB, 
Mà AEN  ABP ( do tứ giác BCEF nội tiếp) 
AN AE 
Do đó hai tam giác ANE và APB đồng dạng nên  
AP AB 
P 
N 
F 
E 
M 
S 
O 
A 
B 
C 
Q
4 
Lại có AM AE 
 ( hai tam giác AEM và ABS đồng dạng) 
AS AB 
Suy ra AM AN 
 nên trong tam giác AMS có NP//SM( định lí Talet đảo) 
AS AP 
Do đó bài toán được chứng minh. 
Bài 5 
a. Giả sử kết luận của bài toán là sai, tức là trong ba đội bất kỳ thì có hai đội đã đấu với nhau rồi. Giả sử 
đội đã gặp các đội 2, 3, 4, 5. Xét các bộ (1; 6; i) với i Є{7; 8; 9;…;12}, trong các bộ này phải có ít nhất một 
cặp đã đấu với nhau, tuy nhiên 1 không gặp 6 hay i nên 6 gặp i với mọi i Є{7; 8; 9;…;12} , vô lý vì đội 6 như 
thế đã đấu hơn 4 trận. Vậy có đpcm. 
b. Kết luận không đúng. Chia 12 đội thành 2 nhóm, mỗi nhóm 6 đội. Trong mỗi nhóm này, cho tất cả 
các đội đôi một đã thi đấu với nhau. Lúc này rõ ràng mỗi đội đã đấu 5 trận. Khi xét 3 đội bất kỳ, phải có 2 đội 
thuộc cùng một nhóm, do đó 2 đội này đã đấu với nhau. Ta có phản ví dụ. 
Có thể giải quyết đơn giản hơn cho câu a. như sau: 
Do mỗi đội đã đấu 4 trận nên tồn tại hai đội A, B chưa đấu với nhau. Trong các đội còn lại, vì A và B chỉ 
đấu 3 trận với họ nên tổng số trận của A, B với các đội này nhiều nhất là 6 và do đó, tồn tại đội C trong số các 
đội còn lại chưa đấu với cả A và B. Ta có A, B, C là bộ ba đội đôi một chưa đấu với nhau.
SỞ GIÁO DỤC ĐÀO TẠO KỲ THI VÀO LỚP 10 CHUYÊN LÝ TỰ TRỌNG 
1 
NGHỆ AN NĂM HỌC 2012 - 2013 
ĐỀ CHÍNH THỨC Môn thi : TOÁN 
(Đề gồm có 1 trang) (Dùng cho thí sinh thi vào lớp chuyên) 
Thời gian làm bài :150 phút (Không kể thời gian giao đề) 
Ngày thi : 18 tháng 6 năm 2012 
Câu 1: (2.0 điểm ) 
       
                 
A x x x x 
Cho biểu thức : 2 3 2 : 2 
x x x x x 
5 6 2 3 1 
1/ Rút gọn biểu thức A. 
2/ Tìm các giá trị của x để 1   
5 
A 2 
Câu 2 (2,0 điểm ) 
Trong mặt phẳng toạ độ Oxy, cho Parabol (P) : y = ax2 a  0và đường 
thẳng (d): y = bx + 1 
1/ Tìm các giá trị của a và b để (P) và (d) cùng đi qua điểm M(1; 2) 
2/ Với a, b vừa tìm được, chứng minh rằng (P) và (d) còn có một điểm chung 
N khác M. Tính diện tích tam giác MON (với O là gốc toạ độ) 
Câu 3 (2.0 điểm) 
1/ Cho phương trình: x2 (2m1)x m2 m6  0 (m là tham số). Tìm m để 
phương trình có hai nghiệm dương phân biệt 
2/ Giải hệ phương trình: 
     
x y 
1 1 2 
1 1 1 
x y 
 
   
Câu 4 (3.0 điểm) : Cho A là điểm cố định nằm ngoài đường tròn (O). Từ A kẻ tiếp 
tuyến AP và AQ tới đường tròn (P và Q là các tiếp điểm). Đường thẳng đi qua O và 
vuông góc với OP cắt đường thẳng OQ tại M. 
1/ Chứng minh rằng: MO = MA 
2/ Lấy điểm N trên cung lớn PQ của đường tròn (O) sao cho tiếp tuyến với (O) 
tại N cắt các tia AP, AQ lần lượt tại B và C. Chứng minh rằng: 
a) AB  AC  BC không phụ thuộc vào vị trí của điểm N. 
b) Nếu tứ giác BCQP nội tiếp được trong một đường tròn thì PQ//BC 
Câu 5 (1.0 điểm) 
Cho x, y là các số thực dương thoả mãn : 
1 2 2 
x y 
  . Chứng minh rằng : 
5x2  y  4xy  y2  3 
---------- Hết ---------- 
Họ tên thí sinh …………………………………………….. Số báo danh: ………………………… 
Chữ ký giám thị 1: ………………………………… Chữ ký giám thị 2: ……………………
2 
Bài giải 
Câu 1: (2.0 điểm ) 
       
                 
A x x x x 
Cho biểu thức : 2 3 2 : 2 
x x x x x 
5 6 2 3 1 
1/ Rút gọn biểu thức A. 
       
                 
A x 2 x 3 x 2 : 2 
x 
x x x x x 
5 6 2 3 1 
(ĐK: x  0, x  4, x  9 ) 
A = … = 1 
4 
x 
x 
 
 
2/ Tìm các giá trị của x để 
1   
5 
A 2 
1 5 4 5 2 8 5 5 
2 1 2 
2 5 3 0 3 1 0 1 
2 2 
0 1 
4 
x x x 
A x 
x x x x 
x 
 
          
 
           
   
Kết hợp với ĐK  
0 1 
4 
 x  
Câu 2 (2,0 điểm ) 
Trong mặt phẳng toạ độ Oxy, cho Parabol (P) : y = ax2 a  0và đường 
thẳng (d): y = bx + 1 
1/ Tìm các giá trị của a và b để (P) và (d) cùng đi qua điểm M(1; 2) 
M (P)  …  a = 2  y = 2x2 
M  (d)  …  b = 1  y = x + 1 
2/ Với a, b vừa tìm được, chứng minh rằng (P) và (d) còn có một điểm chung 
N khác M. Tính diện tích tam giác MON (với O là gốc toạ độ) 
Xét pt hoành độ gđ: 2x2 = x + 1  2x2 - x - 1 = 0 
    
    
            
 
x y 
1 2 
1 1 M 1;2 ; N 
1 ; 1 2 2 
2 2 
x y 
  1 2 ... 0,75 (dvv) MON thang S S S S       
Câu 3 (2.0 điểm) 
1/ Cho phương trình: x2 (2m1)x m2 m6  0(m là tham số). Tìm m để 
phương trình có hai nghiệm dương phân biệt? 
phương trình có hai nghiệm dương phân biệt
3 
 
      m 
                  
           
 0 25 0 3 
. 0 6 0 2 2 
a c m 2 
m m m 
b 0 2 m 1 0 m 
1 a 
2 
2/ Giải hệ phương trình: 
     
x y 
1 1 2 (1) 
1 1 1 (2) 
x y 
 
   
(ĐK: x  1; y  1) 
(2)  x + y = xy (3) 
Hai vế của (1) đều dương ta bình phương hai vế ta có: 
   
x y x y 
x y xy x y 
2 2 1 1 4 
2 2 1 4 
      
        
  
Thay (3) vào ta có: x + y = 4 kết hợp với (3) có hệ: 
x+y=4 
xy=4 
 
Áp dụng hệ thức Vi Ét ta có x; y là hai nghiệm của pt: X2 - 4x + 4 = 0 
 x = 2; y = 2 
Câu 4 (3.0 điểm) : Cho A là điểm cố định nằm ngoài đường tròn (O). Từ A kẻ tiếp 
tuyến AP và AQ tới đường tròn (P và Q là các tiếp điểm). Đường thẳng đi qua O và 
vuông góc với OP cắt đường thẳng OQ tại M. 
1 
1 
1 
2 
1 
1 
C 
B 
M 
P 
O 
Q 
A 
N 
1/ Chứng minh rằng: MO = MA 
A1 = O1 và A1 = A2  A2 = O1 
 MAO cân  MO = MA 
2/ Lấy điểm N trên cung lớn PQ của đường tròn (O) sao cho tiếp tuyến với (O) 
tại N cắt các tia AP, AQ lần lượt tại B và C. Chứng minh rằng: 
a) AB  AC  BC không phụ thuộc vào vị trí của điểm N. 
Theo t/c hai tia tiếp tuyến ta có …  AB + AC - BC = … = 2.AP (không đổi) 
b) Nếu tứ giác BCQP nội tiếp được trong một đường tròn thì PQ//BC 
Nếu tứ giác BCQP nội tiếp được  P1 = C1
  (1) 
4 
mà P1 = Q1  C1 = Q1  PQ//BC 
Câu 5 (1.0 điểm) 
Cho x, y là các số thực dương thoả mãn : 
1 2 2 
x y 
  . Chứng minh rằng : 
5x2  y  4xy  y2  3 
* Ta có: 
2 2 
x y xy y 
x xy y x y 
x y x y 
5   4   
3 
4 2 4 2 2 
3 0 
2 3 0 
       
      
  
2 2 
* 
x y x 
1 2 2 2 2 1 2 2 1 2 
x y y x y x x 
2 1 
 
         
 
Vì : y  0 ; x  0  2x - 1  0  x  1/2 Thay y = … vào x2  y 3  0 
Ta có: 
3 2 
x y x x x x x x 
2 2 2 2     
  3  0    3  0  2 6 3  
0 
x x 
2 1 2 1 
Vì 2x - 1  0  (1)  2x3  x2  2x  6x  3  0 2x3  x2  4x  3  0 
Mà 2x3  x2  4x 3 
3 2 2 
   
x x x x x 
x x x 
2 2 3 3 
      
  1 2 2 
  
3 
  x 1 2  2x  3   0 x  0 
Vậy 2x  y2  x2  y  3  0 x  0; y  0
1 
SỞ GIÁO DỤC VÀ ĐÀO TẠO 
PHÚ THỌ 
KÌ THI TUYỂN SINH VÀO LỚP 10 
TRUNG HỌC PHỔ THÔNG CHUYÊN HÙNG VƯƠNG 
NĂM HỌC 2012-2013 
Môn Toán 
(Dành cho thí sinh thi vào chuyên Toán) 
Thời gian làm bài :150 phút không kể thời gian giao đề 
Đề thi có 1 trang 
Câu 1 ( 2,0 điểm) 
Tính giá trị của biểu thức A  29  30 2  9  4 2  5 2 
Câu 2 ( 2,0 điểm) 
Cho phương trình x2 +mx+1=0 ( m là tham số) 
a) Xác định các giá trị của m để phương trình có nghiệm 
b) Tim m để phương trình có 2 nghiệm x1 
, x2 Thỏa mãn 
2 2 
1 2 
2 2 
2 1 
x  x  
7 
x x 
Câu 3 ( 2,0 điểm) 
a) Giải hệ phương trình 
 x 2 
 xy  x  y 
   x   
y  x 
 
2 2 5 2 0 
4 2 2 
2 3 
b)Giải phương trình 
x 1  x 16  x  4  x  9 
Câu 4( 4 điểm) 
Cho đường tròn (O;R) có dây AB  R 2 , M là điểm chuyển động trên cung 
lớn AB sao cho tam giác MAB nhọn.Gọi H là trực tâm tam giác MAB, C,D lần 
lượt là giao điểm thứ 2 của AH và BH với đường tròn (O).Giải sử N là giao của 
BC và AD 
a) Tính số đo góc AOB, góc MCD 
b) Chứng minh CD là đường kính của đường tròn (O) và HN có độ dài 
không đổi 
c) Chứng minh HN luôn đi qua điểm cố định 
Câu 5 (1,0điểm) 
Cho x.y.z là các số không âm thỏa mãn 3 
x  y  z  .Tìm giá trị nhỏ nhất 
2 
S  x3  y3  z3  x2 y2z2 
----------------Hết--------------------- 
ĐỀ CHÍNH THỨC
Câu 1(1đ) tính A = 29  30 2  9  4 2  5 2 
HD 
A  29  30 2  9  4 2  5 2  29  30 2  2 2 1  5 2  59  30 2  5 2  5 2  3  5 2 
2 
Câu 2(2đ) Cho phương trình x2 +mx +1=0 
a)Xác định m để phương trình có nghiệm. 
2 
2 
2 
1 
x  x 
7 
b) Tìm m để phương trình có nghiệm x1 ; x2 thỏa mãn 2 
1 
2 
2 
x 
x 
 
HD 
a)Có =m2 -4 để pt có nghiệm thì 0 m2 -4 0  
       
  
2 
2 
m 
m 
2 
2 
2 
1 
x  x 
7  9 
b) Có 2 
1 
2 
2 
x 
x 
( ) 2 2 
   
x x x x (*) 
1 2 
1 2 
2 
 
2 1   
 
x x 
2 2 2 
 m   
 
theo viet ta có x1 +x2 =-m ; x1x2 =1 = (*)  9 
1 
   
  
 
  
 
  
2 3 2 
   
 
  
  
   
5 
5 
5 
2 3 
2 
2 
m 
m 
m 
m 
m 
Câu 3 (2đ) a) giải hệ pt 
 
  
2 
x y x 
x xy x y 
2  2  5   2  
0(1) 
2 2 
4   2  
3(2) 
b) giải pt x 1  x 16  x  4  x  9 (*) 
  HD 
 
a) Từ (1) ta được (2x-1)(x+y-2)=0   
1 
(3) 
2 
 
2 (4) 
x   
y 
x 
Thay (3) vào (2) ta được y=1 hoặc y=-1 
Thay (4) vào (2) ta được 5y2 -18y+17=0 ( vô nghiệm) 
Vậy hệ có 2 nghiệm x=1/2, y=1 hoặc x=1/2, y=-1 
b) ĐK x  -1 
(*)  2x+17+2 (x 1)(x 16) =2x+13+2 (x  4)(x  9) 
 2+ (x 1)(x 16) = (x  4)(x  9) 4+x2 +17x+16+4 (x 1)(x 16) =x2 +13x+36 
 (x 1)(x 16) =4-x (x  4) 
 x2 +17x+16=x2 +16-18x  25x=0 x=0 
Vậy pt có nghiệm x=0, 
Câu 4 (4đ) Cho (O;R) có dây cung AB=R 2 cố định. Lấy M di động trên cung lớn AB 
sao cho tam giác AMB có 3 góc nhọn. Gọi H là trực tâm tam giác AMB và C;D lần lượt 
là giao điểm thứ 2 của các đường thẳng AH;BH với (O) Giả sử N là giao điểm của đường 
thẳng BC và DA. 
a) Tính số đo góc AOB và MCD
3 
b) CMR : CD là đường kính của (O) và đoạn NH có độ dài không đổi. 
c) CMR : NH luôn đi qua 1 điểm cố định. 
HD 
Gọi K;L lần lượt là trân đương cao hạ từ B; A của tam giác ABM 
a) có OA2 + OB2 = 2R2 =AB2 = Tam giác OBA vuông tại O = góc AOB=900 
có góc BMA=45 = BKM vuông cân tại K = góc DBM =45= gócDCM 
=45(1) 
L 
K 
O 
H 
D 
M 
C 
B 
P 
A 
N 
b) tương tự ta có ALM vuông cân tại L = gócLAM=45=gócCDM (2) 
Từ (1) và(2) = DCM vuông tại M = CD là đường kính của (O) 
NHB và DCB có góc BNH=gócBDC =NHB đồng dạng DCB (g-g) 
 NH/DC=HB/BC (3) 
Lại có HBC vuông tại C mà gócBCA=1/2gócAOB=45=HBC vuông cân tại B 
 BH=HC (4) 
Từ (3) và (4) = NH/DC=1 = NH=CD không đổi. 
c) Gọi P là trung điểm của NH 
 PB=PA=1/2NH (AHN và BHN vuôngtại A và B) 
Mà OB=OA=1/2CD 
 OB=OA=PA=PB ( vì CD=HN) 
Lại cố gócAOB=90 
 OBPA là hình vuông , mà B; O; A không đổi =P không đổi = PO=AB=R 2 
không đỏi. 
Vậy NH luôn đi qua điêm P cố định 
Câu 5 (1đ) 
Cho x.y.z là các số không âm thỏa mãn 3 
x  y  z  .Tìm giá trị nhỏ nhất 
2 
S= x3+y3+z3+x2y2z2
4 
HD 
Áp dụng BĐT Bunhia cho 2 dãy 
Dãy 1 x x; y y; z z dãy 2 x; y; z 
Ta có ( x 2  y 2  z 2 )[(x x) 2  (y y ) 2  (z z ) 2 ]  (x 2  y 2  z 2 ) 
2 3 ( x 3  y 3  z 3 )  ( x 2  y 2  z 2 )2  x 3  y 3  z 3  2 ( x 2  y 2  z 
2 )2 (*) 
2 3 
Mặt khác 
2 2 2 2 
x  x  ( y  z )  x  ( x  y  z )( x  y  
z 
)(1) 
y 2  y  x  z y  x  z z 2 
 z  y  x z  y  
x 
( )( )(2); ( )( )(3) 
Từ (1), (2), (3) ta có 
( )( )( ) 3 2 3 2 3 2 
                       
xyz x y z x z y y z x z x y 
      
27 9  x y z  6  xy yz xz  
8 
xyz 
8 2 
9 27 3 3 (**) 
        
    
xyz x y z x y z x y z 
  
2 2 2 
2 2 2 2 
2 2 2 2 2 2 
          
8 8 3 
  
Mặt khác Bunhia cho x; y; z và 1;1;1; ta có 
2 
t x y z x y z 
2 2 2 (   
) 3 (***) 
     
3 4 
Từ (*) , (**) , (***)ta có 
2 2 2 2 2 
S  t    t   t  t  t   t  t    t    t       
2 2 3 2 9 7 9 1 3 11 2 3 25 
3 8 3 3 9 4 64 9 4 64 6 4 8 64 64 
    
( ) 25 3 1 
Min S  t   x  y  z  
64 4 2 
GV Trần Bình Trân THCS Phượng Lâu –Việt Trì - Phú Thọ 
mọi góp ý lời giải liên hệ gmail: tbtran1234@gmail.com 
số điện thoại: 0988280207
SỞ GIÁO DỤC VÀ ĐÀO TẠO KỲ THI TUYỂN SINH LỚP 10 THPT CHUYÊN 
QUẢNG NAM Năm học: 2012-2013 
Khóa thi: Ngày 4 tháng 7 năm 2012 
Môn: TOÁN (Chuyên Toán) 
Thời gian làm bài: 150 phút (không kể thời gian giao đề) 
ĐỀ CHÍNH THỨC 
Câu 1: (1,5 điểm) 
a) Rút gọn biểu thức: A = a  a  
6  
1 
4  a a  
2 
(với a ≥ 0 và a ≠ 4). 
b) Cho x  28  
16 3 
3  
1 
. Tính giá trị của biểu thức: P  (x2  2x 1)2012 . 
Câu 2: (2,0 điểm) 
a) Giải phương trình: 3(1 x)  3  x  2 . 
b) Giải hệ phương trình: 
2 
2 
      
x xy 4x 6 
y xy 1 
    
Câu 3: (1,5 điểm) 
Cho parabol (P): y = − x2 và đường thẳng (d): y = (3 − m)x + 2 − 2m (m là tham số). 
a) Chứng minh rằng với m ≠ −1 thì (d) luôn cắt (P) tại 2 điểm phân biệt A, B. 
b) Gọi yA, yB lần lượt là tung độ các điểm A, B. Tìm m để |yA − yB| = 2. 
Câu 4: (4,0 điểm) 
Cho hình chữ nhật ABCD có AB = 4 cm, AD = 2 cm. Đường thẳng vuông góc với 
AC tại C cắt các đường thẳng AB và AD lần lượt tại E và F. 
a) Chứng minh tứ giác EBDF nội tiếp trong đường tròn. 
b) Gọi I là giao điểm của các đường thẳng BD và EF. Tính độ dài đoạn thẳng ID. 
c) M là điểm thay đổi trên cạnh AB (M khác A, M khác B), đường thẳng CM cắt 
đường thẳng AD tại N. Gọi S1 là diện tích tam giác CME, S2 là diện tích tam giác AMN. 
Xác định vị trí điểm M để S 1  3S 
2 
. 
2 
Câu 5: (1,0 điểm) 
Cho a, b là hai số thực không âm thỏa: a + b ≤ 2. 
2  Chứng minh: a  1  
2b  
8 
1  a 1  
2b 7 
. 
--------------- Hết --------------- 
Họ và tên thí sinh: ......................................................... Số báo danh: ...................................
De chuyen2012
De chuyen2012
De chuyen2012
De chuyen2012
De chuyen2012
De chuyen2012
De chuyen2012
De chuyen2012
De chuyen2012
De chuyen2012
De chuyen2012

More Related Content

What's hot

đề Thi và đáp án trường chuyên ams truonghocso.com
đề Thi và đáp án trường chuyên ams   truonghocso.comđề Thi và đáp án trường chuyên ams   truonghocso.com
đề Thi và đáp án trường chuyên ams truonghocso.comThế Giới Tinh Hoa
 
30 dechuyen2007
30 dechuyen200730 dechuyen2007
30 dechuyen2007Toan Isi
 
32 đề thi vào lớp 10 dh khtn ha noi 1989 2005 truonghocso.com
32 đề thi vào lớp 10 dh khtn ha noi 1989 2005   truonghocso.com32 đề thi vào lớp 10 dh khtn ha noi 1989 2005   truonghocso.com
32 đề thi vào lớp 10 dh khtn ha noi 1989 2005 truonghocso.comThế Giới Tinh Hoa
 
TUYỂN TẬP ĐỀ THI VÀO CÁC TRƯỜNG CHUYÊN TP HÀ NỘI (Ams-NguyenHue-ChuVanAn)
TUYỂN TẬP ĐỀ THI VÀO CÁC TRƯỜNG CHUYÊN TP HÀ NỘI (Ams-NguyenHue-ChuVanAn)TUYỂN TẬP ĐỀ THI VÀO CÁC TRƯỜNG CHUYÊN TP HÀ NỘI (Ams-NguyenHue-ChuVanAn)
TUYỂN TẬP ĐỀ THI VÀO CÁC TRƯỜNG CHUYÊN TP HÀ NỘI (Ams-NguyenHue-ChuVanAn)Nhật Hiếu
 
30 đề toán luyện thi vào trương chuyên truonghocso.com
30 đề toán luyện thi vào trương chuyên   truonghocso.com30 đề toán luyện thi vào trương chuyên   truonghocso.com
30 đề toán luyện thi vào trương chuyên truonghocso.comThế Giới Tinh Hoa
 
10 de tang hsg quan huyen thay hong tri quang
10 de tang hsg quan   huyen thay hong tri quang10 de tang hsg quan   huyen thay hong tri quang
10 de tang hsg quan huyen thay hong tri quangHồng Quang
 
9 [htq] de thi hsg 2
9 [htq] de thi hsg 29 [htq] de thi hsg 2
9 [htq] de thi hsg 2Hồng Quang
 
9 [htq] de thi hsg 3 2 lopluyenthi
9 [htq] de thi hsg 3 2 lopluyenthi9 [htq] de thi hsg 3 2 lopluyenthi
9 [htq] de thi hsg 3 2 lopluyenthiHồng Quang
 
CHUYÊN ĐỀ BẤT ĐẲNG THỨC ÔN THI CHUYỂN CẤP LỚP 9
CHUYÊN ĐỀ BẤT ĐẲNG THỨC ÔN THI CHUYỂN CẤP LỚP 9CHUYÊN ĐỀ BẤT ĐẲNG THỨC ÔN THI CHUYỂN CẤP LỚP 9
CHUYÊN ĐỀ BẤT ĐẲNG THỨC ÔN THI CHUYỂN CẤP LỚP 9Hoàng Thái Việt
 
Bai tap menh de tap hop lop 10 co ban
Bai tap menh de tap hop lop 10 co banBai tap menh de tap hop lop 10 co ban
Bai tap menh de tap hop lop 10 co bandiemthic3
 
đề Cương ôn tập toán học kỳ i1 toán 7
đề Cương ôn tập  toán học kỳ i1 toán 7đề Cương ôn tập  toán học kỳ i1 toán 7
đề Cương ôn tập toán học kỳ i1 toán 7Dang thi thuha
 
Đề thi vào lớp 10 THPT môn Toán tỉnh Hưng Yên năm 2015
Đề thi vào lớp 10 THPT môn Toán tỉnh Hưng Yên năm 2015Đề thi vào lớp 10 THPT môn Toán tỉnh Hưng Yên năm 2015
Đề thi vào lớp 10 THPT môn Toán tỉnh Hưng Yên năm 2015Linh Nguyễn
 
Đề Thi THPTQG Toán 2017 Mã Đề 102
Đề Thi THPTQG Toán 2017 Mã Đề 102Đề Thi THPTQG Toán 2017 Mã Đề 102
Đề Thi THPTQG Toán 2017 Mã Đề 102nmhieupdp
 
9 03 de thi tet 2016
9 03 de thi tet 20169 03 de thi tet 2016
9 03 de thi tet 2016Hồng Quang
 
3 đề thi thử môn Toán năm 2017 có đáp án chi tiết - Mẫn Ngọc Quang
3 đề thi thử môn Toán năm 2017 có đáp án chi tiết - Mẫn Ngọc Quang3 đề thi thử môn Toán năm 2017 có đáp án chi tiết - Mẫn Ngọc Quang
3 đề thi thử môn Toán năm 2017 có đáp án chi tiết - Mẫn Ngọc Quanghaic2hv.net
 
39 đề luyện thi học sinh giỏi toán 9
39 đề luyện thi học sinh giỏi toán 939 đề luyện thi học sinh giỏi toán 9
39 đề luyện thi học sinh giỏi toán 9Jackson Linh
 

What's hot (19)

đề Thi và đáp án trường chuyên ams truonghocso.com
đề Thi và đáp án trường chuyên ams   truonghocso.comđề Thi và đáp án trường chuyên ams   truonghocso.com
đề Thi và đáp án trường chuyên ams truonghocso.com
 
30 dechuyen2007
30 dechuyen200730 dechuyen2007
30 dechuyen2007
 
32 đề thi vào lớp 10 dh khtn ha noi 1989 2005 truonghocso.com
32 đề thi vào lớp 10 dh khtn ha noi 1989 2005   truonghocso.com32 đề thi vào lớp 10 dh khtn ha noi 1989 2005   truonghocso.com
32 đề thi vào lớp 10 dh khtn ha noi 1989 2005 truonghocso.com
 
TUYỂN TẬP ĐỀ THI VÀO CÁC TRƯỜNG CHUYÊN TP HÀ NỘI (Ams-NguyenHue-ChuVanAn)
TUYỂN TẬP ĐỀ THI VÀO CÁC TRƯỜNG CHUYÊN TP HÀ NỘI (Ams-NguyenHue-ChuVanAn)TUYỂN TẬP ĐỀ THI VÀO CÁC TRƯỜNG CHUYÊN TP HÀ NỘI (Ams-NguyenHue-ChuVanAn)
TUYỂN TẬP ĐỀ THI VÀO CÁC TRƯỜNG CHUYÊN TP HÀ NỘI (Ams-NguyenHue-ChuVanAn)
 
30 đề toán luyện thi vào trương chuyên truonghocso.com
30 đề toán luyện thi vào trương chuyên   truonghocso.com30 đề toán luyện thi vào trương chuyên   truonghocso.com
30 đề toán luyện thi vào trương chuyên truonghocso.com
 
K10+11+12
K10+11+12K10+11+12
K10+11+12
 
10 de tang hsg quan huyen thay hong tri quang
10 de tang hsg quan   huyen thay hong tri quang10 de tang hsg quan   huyen thay hong tri quang
10 de tang hsg quan huyen thay hong tri quang
 
đề thi vào lớp 10
đề thi vào lớp 10đề thi vào lớp 10
đề thi vào lớp 10
 
9 [htq] de thi hsg 2
9 [htq] de thi hsg 29 [htq] de thi hsg 2
9 [htq] de thi hsg 2
 
9 [htq] de thi hsg 3 2 lopluyenthi
9 [htq] de thi hsg 3 2 lopluyenthi9 [htq] de thi hsg 3 2 lopluyenthi
9 [htq] de thi hsg 3 2 lopluyenthi
 
CHUYÊN ĐỀ BẤT ĐẲNG THỨC ÔN THI CHUYỂN CẤP LỚP 9
CHUYÊN ĐỀ BẤT ĐẲNG THỨC ÔN THI CHUYỂN CẤP LỚP 9CHUYÊN ĐỀ BẤT ĐẲNG THỨC ÔN THI CHUYỂN CẤP LỚP 9
CHUYÊN ĐỀ BẤT ĐẲNG THỨC ÔN THI CHUYỂN CẤP LỚP 9
 
Bai tap menh de tap hop lop 10 co ban
Bai tap menh de tap hop lop 10 co banBai tap menh de tap hop lop 10 co ban
Bai tap menh de tap hop lop 10 co ban
 
đề Cương ôn tập toán học kỳ i1 toán 7
đề Cương ôn tập  toán học kỳ i1 toán 7đề Cương ôn tập  toán học kỳ i1 toán 7
đề Cương ôn tập toán học kỳ i1 toán 7
 
Đề thi vào lớp 10 THPT môn Toán tỉnh Hưng Yên năm 2015
Đề thi vào lớp 10 THPT môn Toán tỉnh Hưng Yên năm 2015Đề thi vào lớp 10 THPT môn Toán tỉnh Hưng Yên năm 2015
Đề thi vào lớp 10 THPT môn Toán tỉnh Hưng Yên năm 2015
 
Đề Thi THPTQG Toán 2017 Mã Đề 102
Đề Thi THPTQG Toán 2017 Mã Đề 102Đề Thi THPTQG Toán 2017 Mã Đề 102
Đề Thi THPTQG Toán 2017 Mã Đề 102
 
9 03 de thi tet 2016
9 03 de thi tet 20169 03 de thi tet 2016
9 03 de thi tet 2016
 
3 đề thi thử môn Toán năm 2017 có đáp án chi tiết - Mẫn Ngọc Quang
3 đề thi thử môn Toán năm 2017 có đáp án chi tiết - Mẫn Ngọc Quang3 đề thi thử môn Toán năm 2017 có đáp án chi tiết - Mẫn Ngọc Quang
3 đề thi thử môn Toán năm 2017 có đáp án chi tiết - Mẫn Ngọc Quang
 
39 đề luyện thi học sinh giỏi toán 9
39 đề luyện thi học sinh giỏi toán 939 đề luyện thi học sinh giỏi toán 9
39 đề luyện thi học sinh giỏi toán 9
 
đề Thi cấp huyện
đề Thi cấp huyệnđề Thi cấp huyện
đề Thi cấp huyện
 

Viewers also liked

De tsl10 toan quang nam(chuyen) 13 14
De tsl10 toan quang nam(chuyen) 13 14De tsl10 toan quang nam(chuyen) 13 14
De tsl10 toan quang nam(chuyen) 13 14Toan Isi
 
De chuyen2009
De chuyen2009De chuyen2009
De chuyen2009Toan Isi
 
De tsl10 toan nghe an chuyen 13-14
De tsl10 toan nghe an chuyen  13-14De tsl10 toan nghe an chuyen  13-14
De tsl10 toan nghe an chuyen 13-14Toan Isi
 
De tsl10 toan hung yen chuyen toan 13-14_giai_
De tsl10 toan hung yen  chuyen toan  13-14_giai_De tsl10 toan hung yen  chuyen toan  13-14_giai_
De tsl10 toan hung yen chuyen toan 13-14_giai_Toan Isi
 
De tsl10 toan lang son chuyen 13-14_giai_
De tsl10 toan lang son chuyen  13-14_giai_De tsl10 toan lang son chuyen  13-14_giai_
De tsl10 toan lang son chuyen 13-14_giai_Toan Isi
 
77 de thi lop10 truong chuyen 2013
77 de thi lop10 truong chuyen 201377 de thi lop10 truong chuyen 2013
77 de thi lop10 truong chuyen 2013Toan Isi
 
De tsl10 toan hai duong(chuyen ng trai) 13 14(giai)
De tsl10 toan hai duong(chuyen ng trai) 13 14(giai)De tsl10 toan hai duong(chuyen ng trai) 13 14(giai)
De tsl10 toan hai duong(chuyen ng trai) 13 14(giai)Toan Isi
 
De tsl10 toan quang ninh chuyen 13-14_giai_
De tsl10 toan quang ninh chuyen  13-14_giai_De tsl10 toan quang ninh chuyen  13-14_giai_
De tsl10 toan quang ninh chuyen 13-14_giai_Toan Isi
 
De tsl10 toan bac giang chuyen 13-14_giai_
De tsl10 toan bac giang chuyen  13-14_giai_De tsl10 toan bac giang chuyen  13-14_giai_
De tsl10 toan bac giang chuyen 13-14_giai_Toan Isi
 
De tsl10 toan ha nam chuyen 13-14_giai_
De tsl10 toan ha nam chuyen  13-14_giai_De tsl10 toan ha nam chuyen  13-14_giai_
De tsl10 toan ha nam chuyen 13-14_giai_Toan Isi
 
De tsl10 toan amsterdam chuyen 13-14
De tsl10 toan amsterdam chuyen  13-14De tsl10 toan amsterdam chuyen  13-14
De tsl10 toan amsterdam chuyen 13-14Toan Isi
 
77 dechuyen2013
77 dechuyen201377 dechuyen2013
77 dechuyen2013Toan Isi
 
De tsl10 toan vinh phuc chuyen 13-14_giai_
De tsl10 toan vinh phuc chuyen  13-14_giai_De tsl10 toan vinh phuc chuyen  13-14_giai_
De tsl10 toan vinh phuc chuyen 13-14_giai_Toan Isi
 
De2010&2011 chuyen
De2010&2011 chuyenDe2010&2011 chuyen
De2010&2011 chuyenToan Isi
 
Tổng hợp đề hsg 9 + key
Tổng hợp đề hsg 9 + keyTổng hợp đề hsg 9 + key
Tổng hợp đề hsg 9 + keyTommy Bảo
 
Tuyển tập 77 đề chuyên 2014
Tuyển tập 77 đề chuyên 2014Tuyển tập 77 đề chuyên 2014
Tuyển tập 77 đề chuyên 2014Toan Isi
 
De thi hsg tieng anh 9 cap tp ha noi 32011
De thi hsg tieng anh 9 cap tp ha noi 32011De thi hsg tieng anh 9 cap tp ha noi 32011
De thi hsg tieng anh 9 cap tp ha noi 32011binhvc
 
Bài giải chi tiết môn anh mã đề 415 kì thi thpt quốc gia 2016
Bài giải chi tiết môn anh mã đề 415 kì thi thpt quốc gia 2016Bài giải chi tiết môn anh mã đề 415 kì thi thpt quốc gia 2016
Bài giải chi tiết môn anh mã đề 415 kì thi thpt quốc gia 2016Tommy Bảo
 

Viewers also liked (20)

De tsl10 toan quang nam(chuyen) 13 14
De tsl10 toan quang nam(chuyen) 13 14De tsl10 toan quang nam(chuyen) 13 14
De tsl10 toan quang nam(chuyen) 13 14
 
De chuyen2009
De chuyen2009De chuyen2009
De chuyen2009
 
De tsl10 toan nghe an chuyen 13-14
De tsl10 toan nghe an chuyen  13-14De tsl10 toan nghe an chuyen  13-14
De tsl10 toan nghe an chuyen 13-14
 
De tsl10 toan hung yen chuyen toan 13-14_giai_
De tsl10 toan hung yen  chuyen toan  13-14_giai_De tsl10 toan hung yen  chuyen toan  13-14_giai_
De tsl10 toan hung yen chuyen toan 13-14_giai_
 
De tsl10 toan lang son chuyen 13-14_giai_
De tsl10 toan lang son chuyen  13-14_giai_De tsl10 toan lang son chuyen  13-14_giai_
De tsl10 toan lang son chuyen 13-14_giai_
 
77 de thi lop10 truong chuyen 2013
77 de thi lop10 truong chuyen 201377 de thi lop10 truong chuyen 2013
77 de thi lop10 truong chuyen 2013
 
De tsl10 toan hai duong(chuyen ng trai) 13 14(giai)
De tsl10 toan hai duong(chuyen ng trai) 13 14(giai)De tsl10 toan hai duong(chuyen ng trai) 13 14(giai)
De tsl10 toan hai duong(chuyen ng trai) 13 14(giai)
 
De tsl10 toan quang ninh chuyen 13-14_giai_
De tsl10 toan quang ninh chuyen  13-14_giai_De tsl10 toan quang ninh chuyen  13-14_giai_
De tsl10 toan quang ninh chuyen 13-14_giai_
 
De tsl10 toan bac giang chuyen 13-14_giai_
De tsl10 toan bac giang chuyen  13-14_giai_De tsl10 toan bac giang chuyen  13-14_giai_
De tsl10 toan bac giang chuyen 13-14_giai_
 
De tsl10 toan ha nam chuyen 13-14_giai_
De tsl10 toan ha nam chuyen  13-14_giai_De tsl10 toan ha nam chuyen  13-14_giai_
De tsl10 toan ha nam chuyen 13-14_giai_
 
De tsl10 toan amsterdam chuyen 13-14
De tsl10 toan amsterdam chuyen  13-14De tsl10 toan amsterdam chuyen  13-14
De tsl10 toan amsterdam chuyen 13-14
 
77 dechuyen2013
77 dechuyen201377 dechuyen2013
77 dechuyen2013
 
10.2014
10.201410.2014
10.2014
 
9.2014
9.20149.2014
9.2014
 
De tsl10 toan vinh phuc chuyen 13-14_giai_
De tsl10 toan vinh phuc chuyen  13-14_giai_De tsl10 toan vinh phuc chuyen  13-14_giai_
De tsl10 toan vinh phuc chuyen 13-14_giai_
 
De2010&2011 chuyen
De2010&2011 chuyenDe2010&2011 chuyen
De2010&2011 chuyen
 
Tổng hợp đề hsg 9 + key
Tổng hợp đề hsg 9 + keyTổng hợp đề hsg 9 + key
Tổng hợp đề hsg 9 + key
 
Tuyển tập 77 đề chuyên 2014
Tuyển tập 77 đề chuyên 2014Tuyển tập 77 đề chuyên 2014
Tuyển tập 77 đề chuyên 2014
 
De thi hsg tieng anh 9 cap tp ha noi 32011
De thi hsg tieng anh 9 cap tp ha noi 32011De thi hsg tieng anh 9 cap tp ha noi 32011
De thi hsg tieng anh 9 cap tp ha noi 32011
 
Bài giải chi tiết môn anh mã đề 415 kì thi thpt quốc gia 2016
Bài giải chi tiết môn anh mã đề 415 kì thi thpt quốc gia 2016Bài giải chi tiết môn anh mã đề 415 kì thi thpt quốc gia 2016
Bài giải chi tiết môn anh mã đề 415 kì thi thpt quốc gia 2016
 

Similar to De chuyen2012

De thi vao lop 10 chuyen toan tinh bac ninh
De thi vao lop 10 chuyen toan tinh bac ninhDe thi vao lop 10 chuyen toan tinh bac ninh
De thi vao lop 10 chuyen toan tinh bac ninhLe Thuy
 
De tsl10 toan phu tho chuyen 13-14
De tsl10 toan phu tho chuyen  13-14De tsl10 toan phu tho chuyen  13-14
De tsl10 toan phu tho chuyen 13-14Toan Isi
 
[Vnmath.com] de thi thpt qg 2015 quynh luu 3
[Vnmath.com]  de thi thpt qg 2015 quynh luu 3[Vnmath.com]  de thi thpt qg 2015 quynh luu 3
[Vnmath.com] de thi thpt qg 2015 quynh luu 3Dang_Khoi
 
Đề thi thử THPT Quốc gia môn Toán Trường THPT Lam Kinh – Thanh Hóa lần 1 năm ...
Đề thi thử THPT Quốc gia môn Toán Trường THPT Lam Kinh – Thanh Hóa lần 1 năm ...Đề thi thử THPT Quốc gia môn Toán Trường THPT Lam Kinh – Thanh Hóa lần 1 năm ...
Đề thi thử THPT Quốc gia môn Toán Trường THPT Lam Kinh – Thanh Hóa lần 1 năm ...schoolantoreecom
 
De thi-thu-l10-chuyen-tn-1415-toan-dk
De thi-thu-l10-chuyen-tn-1415-toan-dkDe thi-thu-l10-chuyen-tn-1415-toan-dk
De thi-thu-l10-chuyen-tn-1415-toan-dkHoa Nguyen
 
[Vnmath.com] de thi quoc gia lan 1 thpt hau loc 2
[Vnmath.com] de thi quoc gia lan 1 thpt hau loc 2[Vnmath.com] de thi quoc gia lan 1 thpt hau loc 2
[Vnmath.com] de thi quoc gia lan 1 thpt hau loc 2Marco Reus Le
 
BỘ ĐỀ CHÍNH THỨC KỲ THI TUYẾN SINH LỚP 10 THPT MÔN TOÁN CHUYÊN CÁC TỈNH NĂM H...
BỘ ĐỀ CHÍNH THỨC KỲ THI TUYẾN SINH LỚP 10 THPT MÔN TOÁN CHUYÊN CÁC TỈNH NĂM H...BỘ ĐỀ CHÍNH THỨC KỲ THI TUYẾN SINH LỚP 10 THPT MÔN TOÁN CHUYÊN CÁC TỈNH NĂM H...
BỘ ĐỀ CHÍNH THỨC KỲ THI TUYẾN SINH LỚP 10 THPT MÔN TOÁN CHUYÊN CÁC TỈNH NĂM H...Nguyen Thanh Tu Collection
 
BỘ ĐỀ CHÍNH THỨC TUYỂN SINH VÀO LỚP 10 THPT CHUYÊN TOÁN CÁC TỈNH NĂM HỌC 2023...
BỘ ĐỀ CHÍNH THỨC TUYỂN SINH VÀO LỚP 10 THPT CHUYÊN TOÁN CÁC TỈNH NĂM HỌC 2023...BỘ ĐỀ CHÍNH THỨC TUYỂN SINH VÀO LỚP 10 THPT CHUYÊN TOÁN CÁC TỈNH NĂM HỌC 2023...
BỘ ĐỀ CHÍNH THỨC TUYỂN SINH VÀO LỚP 10 THPT CHUYÊN TOÁN CÁC TỈNH NĂM HỌC 2023...Nguyen Thanh Tu Collection
 
150 dechuyen2008&2009
150 dechuyen2008&2009150 dechuyen2008&2009
150 dechuyen2008&2009Toan Isi
 
BỘ ĐỀ CHÍNH THỨC + TÁCH ĐỀ + ĐỀ LUYỆN THI VÀO LỚP 10 CHUYÊN TOÁN CÁC TỈNH NĂM...
BỘ ĐỀ CHÍNH THỨC + TÁCH ĐỀ + ĐỀ LUYỆN THI VÀO LỚP 10 CHUYÊN TOÁN CÁC TỈNH NĂM...BỘ ĐỀ CHÍNH THỨC + TÁCH ĐỀ + ĐỀ LUYỆN THI VÀO LỚP 10 CHUYÊN TOÁN CÁC TỈNH NĂM...
BỘ ĐỀ CHÍNH THỨC + TÁCH ĐỀ + ĐỀ LUYỆN THI VÀO LỚP 10 CHUYÊN TOÁN CÁC TỈNH NĂM...Nguyen Thanh Tu Collection
 
Toan pt.de026.2011
Toan pt.de026.2011Toan pt.de026.2011
Toan pt.de026.2011BẢO Hí
 
65 Đề Thi Học Sinh Giỏi Toán 6 Có Đáp Án.doc
65 Đề Thi Học Sinh Giỏi Toán 6 Có Đáp Án.doc65 Đề Thi Học Sinh Giỏi Toán 6 Có Đáp Án.doc
65 Đề Thi Học Sinh Giỏi Toán 6 Có Đáp Án.docLanNguyen176907
 
9 [htq] de thi hsg
9 [htq] de thi hsg9 [htq] de thi hsg
9 [htq] de thi hsgHồng Quang
 
1.bo de thi hoc ki i co dap an
1.bo de thi hoc ki i co dap an1.bo de thi hoc ki i co dap an
1.bo de thi hoc ki i co dap anChau Danh
 
[Vnmath.com] de thi thu thptqg lan 4 chuyen vinh phuc 2015
[Vnmath.com]  de thi thu thptqg lan 4  chuyen vinh phuc 2015[Vnmath.com]  de thi thu thptqg lan 4  chuyen vinh phuc 2015
[Vnmath.com] de thi thu thptqg lan 4 chuyen vinh phuc 2015Dang_Khoi
 
De thi-thu-thpt-quoc-gia-2015-mon-toan-truong-thpt-thanh-chuong-3
De thi-thu-thpt-quoc-gia-2015-mon-toan-truong-thpt-thanh-chuong-3De thi-thu-thpt-quoc-gia-2015-mon-toan-truong-thpt-thanh-chuong-3
De thi-thu-thpt-quoc-gia-2015-mon-toan-truong-thpt-thanh-chuong-3Hồng Nguyễn
 
Toan pt.de054.2011
Toan pt.de054.2011Toan pt.de054.2011
Toan pt.de054.2011BẢO Hí
 
Đề thi thử Toán - Chuyên Vĩnh Phúc 2014 lần 4 Khối B,D
Đề thi thử Toán - Chuyên Vĩnh Phúc 2014 lần 4 Khối B,DĐề thi thử Toán - Chuyên Vĩnh Phúc 2014 lần 4 Khối B,D
Đề thi thử Toán - Chuyên Vĩnh Phúc 2014 lần 4 Khối B,Ddlinh123
 
Toan pt.de077.2010
Toan pt.de077.2010Toan pt.de077.2010
Toan pt.de077.2010BẢO Hí
 

Similar to De chuyen2012 (20)

De thi vao lop 10 chuyen toan tinh bac ninh
De thi vao lop 10 chuyen toan tinh bac ninhDe thi vao lop 10 chuyen toan tinh bac ninh
De thi vao lop 10 chuyen toan tinh bac ninh
 
De tsl10 toan phu tho chuyen 13-14
De tsl10 toan phu tho chuyen  13-14De tsl10 toan phu tho chuyen  13-14
De tsl10 toan phu tho chuyen 13-14
 
[Vnmath.com] de thi thpt qg 2015 quynh luu 3
[Vnmath.com]  de thi thpt qg 2015 quynh luu 3[Vnmath.com]  de thi thpt qg 2015 quynh luu 3
[Vnmath.com] de thi thpt qg 2015 quynh luu 3
 
Đề thi thử THPT Quốc gia môn Toán Trường THPT Lam Kinh – Thanh Hóa lần 1 năm ...
Đề thi thử THPT Quốc gia môn Toán Trường THPT Lam Kinh – Thanh Hóa lần 1 năm ...Đề thi thử THPT Quốc gia môn Toán Trường THPT Lam Kinh – Thanh Hóa lần 1 năm ...
Đề thi thử THPT Quốc gia môn Toán Trường THPT Lam Kinh – Thanh Hóa lần 1 năm ...
 
De thi-thu-l10-chuyen-tn-1415-toan-dk
De thi-thu-l10-chuyen-tn-1415-toan-dkDe thi-thu-l10-chuyen-tn-1415-toan-dk
De thi-thu-l10-chuyen-tn-1415-toan-dk
 
[Vnmath.com] de thi quoc gia lan 1 thpt hau loc 2
[Vnmath.com] de thi quoc gia lan 1 thpt hau loc 2[Vnmath.com] de thi quoc gia lan 1 thpt hau loc 2
[Vnmath.com] de thi quoc gia lan 1 thpt hau loc 2
 
BỘ ĐỀ CHÍNH THỨC KỲ THI TUYẾN SINH LỚP 10 THPT MÔN TOÁN CHUYÊN CÁC TỈNH NĂM H...
BỘ ĐỀ CHÍNH THỨC KỲ THI TUYẾN SINH LỚP 10 THPT MÔN TOÁN CHUYÊN CÁC TỈNH NĂM H...BỘ ĐỀ CHÍNH THỨC KỲ THI TUYẾN SINH LỚP 10 THPT MÔN TOÁN CHUYÊN CÁC TỈNH NĂM H...
BỘ ĐỀ CHÍNH THỨC KỲ THI TUYẾN SINH LỚP 10 THPT MÔN TOÁN CHUYÊN CÁC TỈNH NĂM H...
 
BỘ ĐỀ CHÍNH THỨC TUYỂN SINH VÀO LỚP 10 THPT CHUYÊN TOÁN CÁC TỈNH NĂM HỌC 2023...
BỘ ĐỀ CHÍNH THỨC TUYỂN SINH VÀO LỚP 10 THPT CHUYÊN TOÁN CÁC TỈNH NĂM HỌC 2023...BỘ ĐỀ CHÍNH THỨC TUYỂN SINH VÀO LỚP 10 THPT CHUYÊN TOÁN CÁC TỈNH NĂM HỌC 2023...
BỘ ĐỀ CHÍNH THỨC TUYỂN SINH VÀO LỚP 10 THPT CHUYÊN TOÁN CÁC TỈNH NĂM HỌC 2023...
 
150 dechuyen2008&2009
150 dechuyen2008&2009150 dechuyen2008&2009
150 dechuyen2008&2009
 
BỘ ĐỀ CHÍNH THỨC + TÁCH ĐỀ + ĐỀ LUYỆN THI VÀO LỚP 10 CHUYÊN TOÁN CÁC TỈNH NĂM...
BỘ ĐỀ CHÍNH THỨC + TÁCH ĐỀ + ĐỀ LUYỆN THI VÀO LỚP 10 CHUYÊN TOÁN CÁC TỈNH NĂM...BỘ ĐỀ CHÍNH THỨC + TÁCH ĐỀ + ĐỀ LUYỆN THI VÀO LỚP 10 CHUYÊN TOÁN CÁC TỈNH NĂM...
BỘ ĐỀ CHÍNH THỨC + TÁCH ĐỀ + ĐỀ LUYỆN THI VÀO LỚP 10 CHUYÊN TOÁN CÁC TỈNH NĂM...
 
Toan pt.de026.2011
Toan pt.de026.2011Toan pt.de026.2011
Toan pt.de026.2011
 
65 Đề Thi Học Sinh Giỏi Toán 6 Có Đáp Án.doc
65 Đề Thi Học Sinh Giỏi Toán 6 Có Đáp Án.doc65 Đề Thi Học Sinh Giỏi Toán 6 Có Đáp Án.doc
65 Đề Thi Học Sinh Giỏi Toán 6 Có Đáp Án.doc
 
9 [htq] de thi hsg
9 [htq] de thi hsg9 [htq] de thi hsg
9 [htq] de thi hsg
 
1.bo de thi hoc ki i co dap an
1.bo de thi hoc ki i co dap an1.bo de thi hoc ki i co dap an
1.bo de thi hoc ki i co dap an
 
[Vnmath.com] de thi thu thptqg lan 4 chuyen vinh phuc 2015
[Vnmath.com]  de thi thu thptqg lan 4  chuyen vinh phuc 2015[Vnmath.com]  de thi thu thptqg lan 4  chuyen vinh phuc 2015
[Vnmath.com] de thi thu thptqg lan 4 chuyen vinh phuc 2015
 
De thi-thu-thpt-quoc-gia-2015-mon-toan-truong-thpt-thanh-chuong-3
De thi-thu-thpt-quoc-gia-2015-mon-toan-truong-thpt-thanh-chuong-3De thi-thu-thpt-quoc-gia-2015-mon-toan-truong-thpt-thanh-chuong-3
De thi-thu-thpt-quoc-gia-2015-mon-toan-truong-thpt-thanh-chuong-3
 
Đề Thi HK2 Toán 7 - THCS Nguyễn Huệ
Đề Thi HK2 Toán 7 - THCS Nguyễn HuệĐề Thi HK2 Toán 7 - THCS Nguyễn Huệ
Đề Thi HK2 Toán 7 - THCS Nguyễn Huệ
 
Toan pt.de054.2011
Toan pt.de054.2011Toan pt.de054.2011
Toan pt.de054.2011
 
Đề thi thử Toán - Chuyên Vĩnh Phúc 2014 lần 4 Khối B,D
Đề thi thử Toán - Chuyên Vĩnh Phúc 2014 lần 4 Khối B,DĐề thi thử Toán - Chuyên Vĩnh Phúc 2014 lần 4 Khối B,D
Đề thi thử Toán - Chuyên Vĩnh Phúc 2014 lần 4 Khối B,D
 
Toan pt.de077.2010
Toan pt.de077.2010Toan pt.de077.2010
Toan pt.de077.2010
 

More from Toan Isi

32 dedhkhtn hanoi1989-2005
32 dedhkhtn hanoi1989-200532 dedhkhtn hanoi1989-2005
32 dedhkhtn hanoi1989-2005Toan Isi
 
De dhkhtn tphcm1993-2007
De dhkhtn tphcm1993-2007De dhkhtn tphcm1993-2007
De dhkhtn tphcm1993-2007Toan Isi
 
Cau trucvao10hanoi
Cau trucvao10hanoiCau trucvao10hanoi
Cau trucvao10hanoiToan Isi
 
De tsl10 toan thai nguyen chuyen 13-14_giai_
De tsl10 toan thai nguyen chuyen  13-14_giai_De tsl10 toan thai nguyen chuyen  13-14_giai_
De tsl10 toan thai nguyen chuyen 13-14_giai_Toan Isi
 

More from Toan Isi (12)

8.2014
8.20148.2014
8.2014
 
7.2014
7.20147.2014
7.2014
 
6.2014
6.20146.2014
6.2014
 
5.2014
5.20145.2014
5.2014
 
4.2014
4.20144.2014
4.2014
 
3.2014
3.20143.2014
3.2014
 
2.2014
2.20142.2014
2.2014
 
1.2014
1.20141.2014
1.2014
 
32 dedhkhtn hanoi1989-2005
32 dedhkhtn hanoi1989-200532 dedhkhtn hanoi1989-2005
32 dedhkhtn hanoi1989-2005
 
De dhkhtn tphcm1993-2007
De dhkhtn tphcm1993-2007De dhkhtn tphcm1993-2007
De dhkhtn tphcm1993-2007
 
Cau trucvao10hanoi
Cau trucvao10hanoiCau trucvao10hanoi
Cau trucvao10hanoi
 
De tsl10 toan thai nguyen chuyen 13-14_giai_
De tsl10 toan thai nguyen chuyen  13-14_giai_De tsl10 toan thai nguyen chuyen  13-14_giai_
De tsl10 toan thai nguyen chuyen 13-14_giai_
 

De chuyen2012

  • 1. WWW.VNMATH.COM SÐ GIO DÖC V€ €O T„O THANH HÂA · Ch½nh Thùc · thi gçm câ 01 trang Kœ THI V€O LÎP 10 CHUY–N LAM SÌN N‹M HÅC 2012 - 2013 Mæn: To¡n (chuy¶n) (Dòng cho th½ sinh thi v o chuy¶n To¡n) Thíi gian l m b i: 150 phót (khæng kº thíi gian giao ·) Ng y thi: 18 th¡ng 6 n«m 2012 C¥u 1: (2,0 iºm) Cho a = x + 1 x ; b = y + 1 y ; c = xy + 1 xy vîi c¡c sè thüc x,y thäa m¢n xy6= 0 T½nh gi¡ trà biºu thùc: A = a2 + b2 + c2 abc C¥u 2: (2,0 iºm) Cho ph÷ìng tr¼nh (x 1)(x 2)(x 3)(x 6) = mx2 (m l  tham sè). Gi£ sû m nhªn c¡c gi¡ trà sao cho ph÷ìng tr¼nh câ 4 nghi»m x1; x2; x3; x4 ·u kh¡c 0. Chùng minh r¬ng biºu thùc P = 1 x1 + 1 x2 + 1 x3 + 1 x4 khæng phö thuëc m. C¥u 3: (2,0 iºm) T¼m sè nguy¶n d÷ìng n sao cho n(2n 1) 26 l  sè ch½nh ph÷ìng. C¥u 4: (3,0 iºm) 1) Cho tam gi¡c ABC vuæng t¤i A, ÷íng cao AH. Gåi (I), (K) l¦n l÷ñt l  c¡c ÷íng trán nëi ti¸p tam gi¡c ABH, ACH. ÷íng th¯ng KI c­t c¤nh AB t¤i M v  c¤nh AC t¤i N. a) Chùng minh HI HK = HB HA . b) Chùng minh r¬ng AM = AN. 2) Cho tam gi¡c nhån ABC, D l  iºm tr¶n c¤nh AB (D6= A,B), trung tuy¸n AM c­t CD t¤i E. Chùng minh r¬ng n¸uDBM +DEM = 180 th¼ BC AC p 2. C¥u 5: (1,0 iºm) Cho x,y l  c¡c sè thüc thay êi thäa m¢n: ( x 1; y 1 x + y 4 T¼m gi¡ trà nhä nh§t cõa biºu thùc: P = x4 (y 1)3 + y4 (x 1)3 H¸t WWW.VNMATH.COM
  • 2. UBND TỈNH BẮC NINH SỞ GIÁO DỤC VÀ ĐÀO TẠO ĐỀ THI TUYỂN SINH VÀO LỚP 10 THPT CHUYÊN Năm học 2012 – 2013 Môn thi: Toán (Dành cho thí sinh thi vào chuyên Toán, Tin) Thời gian làm bài: 150 phút (Không kể thời gian giao đề) Ngày thi: 30 tháng 6 năm 2012. ĐỀ CHÍNH THỨC Bài 1 (2,5 điểm) 1/ Rút gọn biểu thức sau: A  4  10  2 5  4  10  2 5 . 2/ Giải phương trình: x2  x2  2x 19  2x+39 . Bài 2 (2,0 điểm) 1/ Cho ba số a, b, c thỏa mãn: 4a 5b  9c  0 . Chứng minh rằng phương trình ax2  bx  c  0 luôn có nghiệm.     xy y2 x 7y x x y 12 y  2/ Giải hệ phương trình:      Bài 3 (1,5 điểm) 1/ Cho ba số dương a, b, c thỏa mãn: a  b  c 1. Chứng minh rằng: 1 a1 b1 c  81 a 1 b1 c . 2/ Phân chia chín số: 1,2,3,4,5,6,7,8,9 thành ba nhóm tùy ý, mỗi nhóm ba số. Gọi 1 T là tích ba số của nhóm thứ nhất, 2 T là tích ba số của nhóm thứ hai, 3 T là tích ba số của nhóm thứ ba. Hỏi tổng 1 2 3 T  T  T có giá trị nhỏ nhất là bao nhiêu? Bài 4 (2,5 điểm) Cho đường tròn tâm O bán kính R và dây cung BC cố định khác đường kính. Gọi A là một điểm chuyển động trên cung lớn BC của đường tròn (O) sao cho tam giác ABC nhọn; AD,BE,CF là các đường cao của tam giác ABC. Các đường thẳng BE, CF tương ứng cắt (O) tại các điểm thứ hai là Q, R. 1/ Chứng minh rằng QR song song với EF. 2/ Chứng minh rằng diện tích tứ giác AEOF bằng EF. R 2 . 3/ Xác định vị trí của điểm A để chu vi tam giác DEF lớn nhất. Bài 5 (1,5 điểm) 1/ Tìm hai số nguyên a,b để a4  4b4 là số nguyên tố. 2/ Hãy chia một tam giác bất kì thành 7 tam giác cân trong đó có 3 tam giác bằng nhau. -----------------------Hết----------------------- (Đề thi gồm có 01 trang) Họ và tên thí sinh:………………………..…………………..Số báo danh:……….……….
  • 3. UBND TỈNH BẮC NINH SỞ GIÁO DỤC VÀ ĐÀO TẠO HƯỚNG DẪN CHẤM ĐỀ THI TUYỂN SINH VÀO LỚP 10 THPT CHUYÊN Năm học 2012 – 2013 Môn thi: Toán (Dành cho thí sinh thi vào chuyên toán, tin) Bài Đáp án Điểm 1/ Rút gọn biểu thức sau: A  4  10  2 5  4  10  2 5 . 1,5 Nhận xét rằng A  0. 0,25 A2  4 10  2 5  4  10  2 5  2 4  10  2 5 4  10  2 5  0,25  8  2 6  2 5 0,25  2  8  2 5 1 0,25  2  6  2 5  5 1 . 0,25 Vậy A 1 5 0,25 Giải phương trình: x2  x2  2x 19  2x+39 (*) 1,0 Đặt t  x2  2x 19  0. 0,25 (*) trở thành: t2  t  20  0  t  4 (     nhËn) lo¹i t 5 ( ) 0,25 t  4x2  2x 19 16x2  2x 35  0. 0,25 1 (2,5 điểm) x 7 x 5        . 0,25 1/ Cho 4a 5b 9c  0 , chứng minh phương trình ax2  bx  c  0 luôn có nghiệm. 1,0 Xét trường hợp a = 0. Nếu b = 0 thì từ 4a 5b 9c  0 , ta suy ra c = 0, do đó phương trình (1) nghiệm đúng với mọi 0,25 x   . Còn nếu b  0 , phương trình (1) trở thành bx  c  0 , có nghiệm x   c . b Trường hợp a  0 , (1) là phương trình bậc hai. Từ 4a 5b 9c  0 , ta có  b 4a 9c  . Suy ra, 5 0,25 2 2 2 2 2 2 b 2 4ac (4a  9c) 4a 16a 28ac  81c (2a  7c) 12a  32c 25 2 5 c 5 0 2           . 0,25 Do đó, (1) có hai nghiệm phân biệt. Vậy trong mọi trường hợp, (1) luôn có nghiệm. 0,25     xy y2 x 7y x x y 12 y  2/ Giải hệ phương trình:      1,0 2 (2,0 điểm) ĐK: y  0 Hệ tương đương với x y x 7      y x  x  y   12 y     ta có hệ: u v 7 , đặt u x y, v x y      uv 12 0,25
  • 4. u 3 u 4 v 4 v 3            0,25 Với u  4, v  3ta có hệ x 3 x 3 y              y 1 x y 4 0,25 Với u  3, v  4 ta có hệ x x 12 4 5 y y 3 x y 3 5                0,25 1/ Cho ba số dương a, b, c thỏa mãn: a  b  c 1. Chứng minh rằng: 1 a1 b1 c  81 a 1 b1 c . 1,0 Từ a + b + c = 1 ta có 1 + a = (1 – b) + (1 – c)  2 (1 b)(1 c) (Vì a, b, c 1 nên 1 – b ; 1 – c ; 1 – a là các số dương). 0,25 Tương tự ta có 1 + b  2 (1 c)(1 a) và 1 + c  2 (1 a)(1 b). 0,25 Nhân các vế của ba BĐT ta có: 1 a1 b1 c  81 a 1 b1 c  đpcm. 0,25 Dấu bằng xảy ra khi và chỉ khi a  b  c  1 . 0,25 3 2/ Phân chia chín số: 1,2,3,4,5,6,7,8,9 thành ba nhóm tùy ý, mỗi nhóm ba số. Gọi 1 T là tích ba số của nhóm thứ nhất, 2 T là tích ba số của nhóm thứ hai, 3 T là tích ba số của nhóm thứ ba. Hỏi tổng 1 2 3 T  T  T có giá trị nhỏ nhất là bao nhiêu? 0,5 1 2 3 1 2 3 T  T  T  3 T .T .T Ta có: 3 3 1 2 3 T .T .T 1.2.3.4.5.6.7.8.9  72.72.70  71 0,25 3 (1,5 điểm) Do đó, 1 2 3 T  T  T  213 mà 1 2 3 T ,T ,T nguyên nên 1 2 3 T  T  T  214 . Ngoài ra, 214  72  72  70 1.8.9  3.4.6  2.5.7 . Nên giá trị nhỏ nhất của 1 2 3 T  T  T là 214. 0,25 Cho đường tròn tâm O bán kính R và dây cung BC cố định khác đường kính. Gọi A là một điểm chuyển động trên cung lớn BC của đường tròn (O) sao cho tam giác ABC nhọn; AD,BE,CF là các đường cao của tam giác ABC. Các đường thẳng BE, CF tương ứng cắt (O) tại các điểm thứ hai là Q, R. 1/ Chứng minh rằng QR song song với EF. 1,0 O R Q F E A B D C Vì BEC  BFC  900 nên tứ giác BCEF nội tiếp đường tròn đường kính BC. 0,25 4 (2,5 điểm) Suy ra, BEF  BCF. 0,25
  • 5. Mà BCF BQR 1 sđ BR       2   nên BEF  BQR . 0,25 Suy ra, QR / /EF . 0,25 2/ Chứng minh rằng diện tích tứ giác AEOF bằng EF. R 2 . 0,5 Vì tứ giác BCEF nội tiếp nên EBF  ECF mà EBF 1 sđ AQ,ECF 1 sđ A   R nên 2 2 AQ  AR . 0,25 Do đó, OA  QR mà QR / /EF nên OA  EF. Vì OA  EF nên AEOF   0,25 S EF.OA EF.R . 2 2 3/ Xác định vị trí của điểm A để chu vi tam giác DEF lớn nhất. 1,0 Tương tự câu 2, BFOD CDOE 2S  FD.R, 2S  DE.R . Mà tam giác ABC nhọn nên O nằm trong tam giác ABC. 0,25 Suy ra, 2S  2S  2S  2S  R  DE  EF  FD  . ABC AEOF BFOD CDOE 0,25 Vì R không đổi nên đẳng thức trên suy ra chu vi tam giác DEF lớn nhất khi và chỉ khi diện tích tam giác ABC lớn nhất. 0,25 Mà S  1 BC.AD với BC không đổi nên S lớn nhất khi AD lớn nhất. Khi đó, ABC ABC 2 A là điểm chính giữa của cung lớn BC. 0,25 1/ Tìm hai số nguyên a, b để a4  4b4 là số nguyên tố. 1,0 a4  4b4  a2  2ab  2b2 a2  2ab  2b2 . 0,25 Vì a2  2ab  2b2  0;a2  2ab  2b2  0 . 0,25 Nên a4  4b4 nguyên tố  Một thừa số là 1 còn thừa số kia là số nguyên tố .                TH1:     a b 1   2 2 2 2 2 2   2    2  (1) b 0 a 2ab 2b 1 a b b 1 a b 0 (2) b 1 *Với (1)b  0a2 1M 1 (loại).    a  b  1 *Với 2       a b 1 (thỏa mãn). 0,25 5 (1,5 điểm)                TH2:     a b 1   2 2 2 2 2 2   2    2  (3) b 0 a 2ab 2b 1 a b b 1 a b 0 (4) b 1 *Với (3) b  0a2  1 M  1 (loại).    a  1  a   1 *Với 4          b 1 b 1 (thỏa mãn). Vậy các cặp số a; b cần tìm là: 1;1,1;1,1;1,1;1. 0,25
  • 6. 2/ Hãy chia một tam giác bất kì thành 7 tam giác cân trong đó có 3 tam giác bằng nhau. 0,5 F E O G D C A B Trường hợp 1:Tam giác ABC không cân. Giả sử AB là cạnh lớn nhất của tam giác ABC. Vẽ cung tròn tâm A, bán kính AC cắt AB tại D. Vẽ cung tròn tâm B, bán kính BD cắt BC tại E. Vẽ cung tròn tâm C, bán kính CE cắt AC tại F. Vẽ cung tròn tâm A, bán kính AF cắt AB tại G. Dễ dàng chứng minh 5 điểm C,D,E,F,G thuộc đường tròn tâm O với O là tâm đường tròn nội tiếp tam giác ABC. Nối 5 điểm đó với O, nối A, B với O, nối F với G, D với E ta được 7 tam giác cân: AGF,OGF,ODG,BDE,ODE,OCE,OCF. Trong đó, có ba tam giác bằng nhau là: OCE,OCF,OGD. 0,25 A I G H F E D B C Trường hợp 2: Tam giác ABC cân. Giả sử tam giác ABC cân tại A. Gọi D, E, F, G, H, I lần lượt là trung điểm các đoạn thẳng: AB, BC, CA, DE, EF, FD. Khi đó, ta có 7 tam giác cân ADF, BDE, CEF, DGI, EGH, FHI, GHI trong đó ba tam giác bằng nhau là: ADF, BDE, CEF. 0,25 Các chú ý khi chấm: 1. Bài làm của học sinh phải chi tiết, lập luận chặt chẽ, tính toán chính xác mới được điểm tối đa. 2. Với các cách giải đúng nhưng khác đáp án, tổ chấm trao đổi và thống nhất điểm chi tiết (đến 0,25 điểm) nhưng không được vượt quá số điểm dành cho bài hoặc phần đó. Trong trường hợp sai sót nhỏ có thể cho điểm nhưng phải trừ điểm chỗ sai đó. 3. Với Bài 4 và Bài 5.2 không cho điểm bài làm nếu học sinh không vẽ hình. 4. Mọi vấn đề phát sinh trong quá trình chấm phải được trao đổi trong tổ chấm và chỉ cho điểm theo sự thống nhất của cả tổ. 5. Điểm toàn bài là tổng số điểm các phần đã chấm, không làm tròn điểm.
  • 7.
  • 8. 1
  • 9. Së gi¸o dôc vμ ®μo t¹o Hμ nam Kú thi tuyÓn sinh líp 10 THPT chuyªn N¨m häc 2012-2013 M«n thi : to¸n Thêi gian lμm bμi: 120 phót §Ò chÝnh thøc Bμi 1. (2,0 ®iÓm) Cho biÓu thøc   1  3 a  b 3     a3b  ab3                M .     3b a b b 3a a b a b 1. T×m ®iÒu kiÖn cña a, b ®Ó M x¸c ®Þnh vμ rót gän M 2. TÝnh gi¸ trÞ cña M khi a = 2 5     a 5 2, b 3 3 Bμi 2. (2,0®iÓm) Cho ph­ ¬ng tr×nh x4 2(m2 3)x2 m4 5 0 (mlμ thamsè) 1. Chøng minh r»ng ph­ ¬ng tr×nh cã bèn nghiÖm x1; x2; x3; x4 víi mäi m thuéc R 2. X¸c ®Þnh m ®Ó 2 2 2 2 1 2 3 4 1 2 3 4 2x x x x (x x x x )28 Bμi 3. (1,5 ®iÓm) T×m tÊt c¶ c¸c sè nguyªn x, y tho¶ m·n ph­ ¬ng tr×nh: x3 – x2y + 3x – 3y – 5 = 0 Bμi 4. (3,5 ®iÓm) Cho tam gi¸c ®Òu ABC néi tiÕp trong ®­êng trßn (O). Mét ®­êng thÈng d thay ®æi ®i qua A, c¾t (O) t¹i ®iÓm thø hai lμ E, c¾t hai tiÕp tuyÕn kÎ tõ B vμ C cña ®­êng trßn (O) lÇn l­ît t¹i M vμ N sao cho A, M, N n»m ë cïng nöa mÆt ph¼ng bê BC. Gäi giao ®iÓm cña hai ®­êng th¼ng MC vμ BN lμ F. Chøng minh r»ng: 1. Hai tam gi¸c MBA vμ CAN ®ång d¹ng vμ tÝch MB.CN kh«ng ®æi. 2. Tø gi¸c BMEF néi tiÕp trong mét ®­êng trßn. 3. §­êng th¼ng EF lu«n ®i qua mét ®iÓm cè ®Þnh khi (d) thay ®æi Bμi 5. (1.0 ®iÓm) Cho bèn sè thùc a, b, c, d tho¶ m·n: ad – bc = 3 . Chøng minh r»ng: a2 b2 c2 d2 acbd3. DÊu b»ng x¶y ra khi nμo? -----------------------HÕt----------------------- Hä vμ tªn thÝ sinh : ......................................................Sè b¸o danh :....................... Ch÷ kÝ cña gi¸m thÞ 1 : .............................Ch÷ kÝ cña gi¸m thÞ 2:............................
  • 10. ĐẠI HỌC QUỐC GIA HÀ NỘI ĐỀ THI TUYỂN SINH LỚP 10 TRƯỜNG THPT CHUYÊN NĂM 2012 MÔN THI: TOÁN (cho tất cả các thí sinh) Thời gian làm bài: 120 phút (Không kể thời gian phát đề) Câu I. 1) Giải phương trình  x  9  2012 x  6  2012  x  9x  6 2) Giải hệ phương trình    2 2 2 4 x y xy x y y    2    4 Câu II. 1) Tìm tất cả các cặp số nguyên x; y thỏa mãn đẳng thức: x  y 1xy  x  y  5  2x  y 2) Giả sử x, y là các số thực dương thỏa mãn điều kiện  x 1 y 1 4 Tìm giá trị nhỏ nhất của biểu thức : y 2 2 x P x   y Câu III. Cho tam giác nhọn ABC nội tiếp đường tròn tâm O .Gọi M là một điểm trên cung nhỏ BC ( M khác B,C và AM không đi qua O).Giả sử P là một điểm thuộc đoạn thẳng AM sao cho đường tròn đường kính MP cắt cung nhỏ BC tại điểm N khác M. 1) Gọi D là điểm đối xứng với điểm M qua O .Chứng minh rằng ba điểm N,P,D thẳng hàng 2) Đường tròn đường kính MP cắt MD tại Q khác M. Chứng minh rằng Q là tâm đườn tròn nội tiếp tam giác AQN. Câu IV. Giả sử a,b,c là các số thực dương thỏa mãn a  b  3  c;c  b 1;a  b  c Tìm giá trị nhỏ nhất của biểu thức: Q ab a b c ab 2    (  1) (  1)(  1)(  1)  a b c Cán bộ coi thi không giải thich gì thêm.
  • 11. ĐÁP ÁN VÀ BIỂU ĐIỂM Bài thi Toán vào lớp 10 chuyên( chuyên TN và chuyên ngữ) 2012-2013 Câu Đáp án Điểm I  1) x  9  2012 x  6  2012  x  9x  6 (1) Đặt x  9  a  0 , x  6  b  0  (1) a  2012  a  2012 b  2012  ab  ( a  2012 )( b  1 )  0   b  1    9  2012 x  6  1 x 9 4048144 x        6 1 x x 4048135 vậy tập nghiệm pt là S  4048135;5       5 x 2 2)    2 2 2 4(1) x y xy x y y    2    4     2 2 2 4 x y xy x y y    4  2  2  8  x2  y2  2xy  4x  4y 12 x y     0 12 ) ( 4 ) ( 2      y x y x  0 ) 2 )( 6 (      y x y x      2 6 x y   * x  y  6  x  6  y thế vào (1) có (6  y)2  y2  2y  4  2 y 2  17 y  16  0  y2  7y 16  0 vô nghiệm * x  y  2  x  2  y thế vào (1) có (2  y)2  y2  2y  4  2y2  2y  0  y  0  x  2; y  0   Vậy tập nghiệm của hệ S  (2;0);(1;1) y  1 x  1; y  1 1,5 II 1. x  y 1xy  x  y  5  2x  y  x  y 1xy  x  y  2x  y 1 3  x  y 1xy  x  y  2  3  3.1 1.3  (3).(1)  (1).(3) Xét các trường hợp: a)    1 3 x y    2 1 xy x y         x y    1 2 xy     x b) 1 y   1    1 1 x y     2 3 xy x y          x y     1 2 xy     1   1   x y c)    1 1 x y    2 3 xy x y         x y vô nghiệm d)    5 0 xy    x  y  1   3  2 1 xy x y         x y vô nghiệm     5 4 xy Vậy tập nghiệm của phương trình S  (1;1);(1;1) 1,5 2 2 y P x 2. Áp dụng BĐT Cauchy-Schwarz ta có x x  y      x y y x y  ( )2 (1) Đẳng thức có  x=y Gt  x 1 y 1 4  4  x  y  xy 1 1 x  y 1 x y = x  y  2 (BĐT 2 2 2 1      Cauchy)  x  y  2 (2) Đẳng thức có x=y=1 Từ (1) và (2)  P  x  y  2 . P=2 x=y=1. Vậy 2 min P  x=y=1 1
  • 12. III 1. M(O) góc MND=900 DNMN (1) M(O') góc MNP=900  PNMN (2) Từ (1) và (2) D, P, N thẳng hàng 2. Góc DNA=DMA (3) (=1/2sđ cung AD) Góc PNQ=PMQ (4) (=1/2sđ cung PQ) Từ (3) và (4)  gócPNA=PNQNP là p giác góc ANQ(5) Góc MAN=MDN (6) (=1/2sđ cung MN) Tứ giác ADQP có góc PAD=DQP=900 nội tiếp được  góc PDQ=PAQ (7) Q A P N O B C M D O' Từ (6) và (7)  góc PAN= PAQAP là phân giác góc NAQ (8) Từ (5) và (8)  P là tâm đường tròn nội tiếp tam giác ANQ 1,5 1,5 IV Q 2 ab  a  b  c ( ab  1) = Ta có (  1)(  1)(  1)  a b c a b ab c ( 1).( 1) ( 1).( 1)      ( a  1).( b  1).( c  1) = 1  ab ( 1).( 1) 1 1     a b c  1 1 1 1  ab     a b   ab a b Do a  b  c  b 1 a 1 ; b  a 1  (a 1).(b 1)  0  ab  a  b 1 c 1 2 Suy ra ab  1 2( 1) 2 1     ab ab Q Đặt ab  x  x  2  1  x 2( 1) 2 1     x x Q  5 x x ( do x  2 )  (  2).(  5)  Q 0 12 x 5  1  2( 1) 2 1 12      x x 12(  1).(  2)  x x Q  5 12 Q  5 a=1; b=2; c=3. Vậy 12 5 min Q  a=1; b=2; c=3. 12 1
  • 13. 1 Së gi¸o dôc vμ ®μo t¹o H­ng yªn ®Ò chÝnh thøc (§Ò thi cã 01 trang) kú thi tuyÓn sinh vμo líp 10 thpt chuyªn N¨m häc 2012 - 2013 M«n thi: To¸n (Dμnh cho thÝ sinh dù thi c¸c líp chuyªn: To¸n, Tin) Thêi gian lμm bμi: 150 phót Bài 1: (2 điểm) a) Cho A = 20122  20122.20132  20132 . Chứng minh A là một số tự nhiên. b) Giải hệ phương trình x 1 x 3   2    y 2 y x  1  x  3 y y  Bài 2: (2 điểm) a) Cho Parbol (P): y = x2 và đường thẳng (d): y = (m +2)x – m + 6. Tìm m để đường thẳng (d) cắt Parabol (P) tại hai điểm phân biệt có hoành độ dương. b) Giải phương trình: 5 + x + 2 (4  x)(2x  2)  4( 4  x  2x  2) Bài 3: (2 điểm) a) Tìm tất cả các số hữu tỷ x sao cho A = x2 + x+ 6 là một số chính phương. b) Cho x 1 và y 1. Chứng minh rằng : (x3  y3)  (x2  y2 ) 8 (x 1)(y 1)    Bài 4 (3 điểm) Cho tam giác ABC nhọn nội tiếp đường tròn tâm O, đường cao BE và CF. Tiếp tuyến tại B và C cắt nhau tại S, gọi BC và OS cắt nhau tại M a) Chứng minh AB. MB = AE.BS b) Hai tam giác AEM và ABS đồng dạng c) Gọi AM cắt EF tại N, AS cắt BC tại P. CMR NP vuông góc với BC Bài 5: (1 điểm) Trong một giải bóng đá có 12 đội tham dự, thi đấu vòng tròn một lượt (hai đội bất kỳ thi đấu với nhau đúng một trận). a) Chứng minh rằng sau 4 vòng đấu (mỗi đội thi đấu đúng 4 trận) luôn tìm được ba đội bóng đôi một chưa thi đấu với nhau. b) Khẳng định trên còn đúng không nếu các đội đã thi đấu 5 trận? HƯỚNG DẪN GIẢI Bài 1: (2 điểm)
  • 14. 2 a) Cho A = 20122  20122.20132  20132 Đặt 2012 = a, ta có 20122  20122.20132  20132  a2  a2 (a 1)2  (a 1)2  (a2  a 1)2  a2  a 1 b) Đặt x a y x 1 b y       Ta có x 1 x 3   2    y 2 y x  1  x  3 y y             2 x 1 x 3 y y x 1 x 3     y y nên                 b2 a 3 b2 b 6 0 b a 3 b a 3 a 6 a 1      v       b 3 b 2 Bài 2: a) ycbt tương đương với PT x2 = (m +2)x – m + 6 hay x2 - (m +2)x + m – 6 = 0 có hai nghiệm dương phân biệt. b) Đặt t = 4  x  2x  2 Bài 3: a) x = 0, x = 1, x= -1 không thỏa mãn. Với x khác các giá trị này, trước hết ta chứng minh x phải là số nguyên. +) x2 + x+ 6 là một số chính phương nên x2 + x phải là số nguyên. m +) Giả sử x  với m và n có ước nguyên lớn nhất là 1. n Ta có x2 + x = 2 2 2 2 m m m  mn n n n   là số nguyên khi m2 mn chia hết cho n2 nên m2 mn chia hết cho n, vì mn chia hết cho n nên m2 chia hết cho n và do m và n có ước nguyên lớn nhất là 1, suy ra m chia hết cho n( mâu thuẫn với m và n có ước nguyên lớn nhất là 1). Do đó x phải là số nguyên. Đặt x2 + x+ 6 = k2 Ta có 4x2 + 4x+ 24 = 4 k2 hay (2x+1)2 + 23 = 4 k2 tương đương với 4 k2 - (2x+1)2 = 23 (x3  y3)  (x2  y2 ) x2 (x  1)  y2 (y  1)  (x  1)(y  1) (x  1)(y  1) = x2  y2 y  1 x  1 (x  1)2  2(x  1)  1 (y  1)2  2(y  1)  1   y  1 x  1   2  2     (x 1) (y 1) 2(y 1)  2(x  1)   1       1  y  1 x  1   x  1 y  1     y  1 x  1   . Theo BĐT Côsi
  • 15. 3 (x  1)2 (y  2  1) 2 (x  1)2 . (y  1)2  2 (x  1)(y  1) y 1 x 1 y 1 x 1     2(y  1) 2(x   1)  2(y  1)  . 2(x 1)  4 x 1 y 1 x 1 y 1     1  1  2 1 . 1 y 1 x 1 y 1 x 1     2 1 . 1 (x 1)(y 1) 2.2 1 . 1 . (x 1)(y 1) 4            y  1 x  1   y  1 x  1 Nên có đpcm Bài 4 a) Suy ra từ hai tam giác đồng dạng là ABE và BSM b) Từ câu a) ta có AE MB  (1) AB BS Mà MB = EM( do tam giác BEC vuông tại E có M là trung điểm của BC Nên AE  EM AB BS Có MOB  BAE,EBA  BAE  900 ,MBOMOB  900 Nên MBO  EBA do đó MEB  OBA( MBE) Suy ra MEA  SBA(2) Từ (1) và (2) suy ra hai tam giác AEM và ABS đồng dạng(đpcm.) c) Dễ thấy SM vuông góc với BC nên để chứng minh bài toán ta chứng minh NP //SM. + Xét hai tam giác ANE và APB: Từ câu b) ta có hai tam giác AEM và ABS đồng dạng nên NAE  PAB, Mà AEN  ABP ( do tứ giác BCEF nội tiếp) AN AE Do đó hai tam giác ANE và APB đồng dạng nên  AP AB P N F E M S O A B C Q
  • 16. 4 Lại có AM AE  ( hai tam giác AEM và ABS đồng dạng) AS AB Suy ra AM AN  nên trong tam giác AMS có NP//SM( định lí Talet đảo) AS AP Do đó bài toán được chứng minh. Bài 5 a. Giả sử kết luận của bài toán là sai, tức là trong ba đội bất kỳ thì có hai đội đã đấu với nhau rồi. Giả sử đội đã gặp các đội 2, 3, 4, 5. Xét các bộ (1; 6; i) với i Є{7; 8; 9;…;12}, trong các bộ này phải có ít nhất một cặp đã đấu với nhau, tuy nhiên 1 không gặp 6 hay i nên 6 gặp i với mọi i Є{7; 8; 9;…;12} , vô lý vì đội 6 như thế đã đấu hơn 4 trận. Vậy có đpcm. b. Kết luận không đúng. Chia 12 đội thành 2 nhóm, mỗi nhóm 6 đội. Trong mỗi nhóm này, cho tất cả các đội đôi một đã thi đấu với nhau. Lúc này rõ ràng mỗi đội đã đấu 5 trận. Khi xét 3 đội bất kỳ, phải có 2 đội thuộc cùng một nhóm, do đó 2 đội này đã đấu với nhau. Ta có phản ví dụ. Có thể giải quyết đơn giản hơn cho câu a. như sau: Do mỗi đội đã đấu 4 trận nên tồn tại hai đội A, B chưa đấu với nhau. Trong các đội còn lại, vì A và B chỉ đấu 3 trận với họ nên tổng số trận của A, B với các đội này nhiều nhất là 6 và do đó, tồn tại đội C trong số các đội còn lại chưa đấu với cả A và B. Ta có A, B, C là bộ ba đội đôi một chưa đấu với nhau.
  • 17. ĐẠI HỌC QUỐC GIA HÀ NỘI ĐỀ THI TUYỂN SINH LỚP 10 TRƯỜNG THPT CHUYÊN NĂM 2012 MÔN THI: TOÁN (cho tất cả các thí sinh) Thời gian làm bài: 120 phút (Không kể thời gian phát đề) Câu I. 1) Giải phương trình  x  9  2012 x  6  2012  x  9x  6 2) Giải hệ phương trình    2 2 2 4 x y xy x y y    2    4 Câu II. 1) Tìm tất cả các cặp số nguyên x; y thỏa mãn đẳng thức: x  y 1xy  x  y  5  2x  y 2) Giả sử x, y là các số thực dương thỏa mãn điều kiện  x 1 y 1 4 Tìm giá trị nhỏ nhất của biểu thức : y 2 2 x P x   y Câu III. Cho tam giác nhọn ABC nội tiếp đường tròn tâm O .Gọi M là một điểm trên cung nhỏ BC ( M khác B,C và AM không đi qua O).Giả sử P là một điểm thuộc đoạn thẳng AM sao cho đường tròn đường kính MP cắt cung nhỏ BC tại điểm N khác M. 1) Gọi D là điểm đối xứng với điểm M qua O .Chứng minh rằng ba điểm N,P,D thẳng hàng 2) Đường tròn đường kính MP cắt MD tại Q khác M. Chứng minh rằng Q là tâm đườn tròn nội tiếp tam giác AQN. Câu IV. Giả sử a,b,c là các số thực dương thỏa mãn a  b  3  c;c  b 1;a  b  c Tìm giá trị nhỏ nhất của biểu thức: Q ab a b c ab 2    (  1) (  1)(  1)(  1)  a b c Cán bộ coi thi không giải thich gì thêm.
  • 18. ĐÁP ÁN VÀ BIỂU ĐIỂM Bài thi Toán vào lớp 10 chuyên( chuyên TN và chuyên ngữ) 2012-2013 Câu Đáp án Điểm I  1) x  9  2012 x  6  2012  x  9x  6 (1) Đặt x  9  a  0 , x  6  b  0  (1) a  2012  a  2012 b  2012  ab  ( a  2012 )( b  1 )  0   b  1    9  2012 x  6  1 x 9 4048144 x        6 1 x x 4048135 vậy tập nghiệm pt là S  4048135;5       5 x 2 2)    2 2 2 4(1) x y xy x y y    2    4     2 2 2 4 x y xy x y y    4  2  2  8  x2  y2  2xy  4x  4y 12 x y     0 12 ) ( 4 ) ( 2      y x y x  0 ) 2 )( 6 (      y x y x      2 6 x y   * x  y  6  x  6  y thế vào (1) có (6  y)2  y2  2y  4  2 y 2  17 y  16  0  y2  7y 16  0 vô nghiệm * x  y  2  x  2  y thế vào (1) có (2  y)2  y2  2y  4  2y2  2y  0  y  0  x  2; y  0   Vậy tập nghiệm của hệ S  (2;0);(1;1) y  1 x  1; y  1 1,5 II 1. x  y 1xy  x  y  5  2x  y  x  y 1xy  x  y  2x  y 1 3  x  y 1xy  x  y  2  3  3.1 1.3  (3).(1)  (1).(3) Xét các trường hợp: a)    1 3 x y    2 1 xy x y         x y    1 2 xy     x b) 1 y   1    1 1 x y     2 3 xy x y          x y     1 2 xy     1   1   x y c)    1 1 x y    2 3 xy x y         x y vô nghiệm d)    5 0 xy    x  y  1   3  2 1 xy x y         x y vô nghiệm     5 4 xy Vậy tập nghiệm của phương trình S  (1;1);(1;1) 1,5 2 2 y P x 2. Áp dụng BĐT Cauchy-Schwarz ta có x x  y      x y y x y  ( )2 (1) Đẳng thức có  x=y Gt  x 1 y 1 4  4  x  y  xy 1 1 x  y 1 x y = x  y  2 (BĐT 2 2 2 1      Cauchy)  x  y  2 (2) Đẳng thức có x=y=1 Từ (1) và (2)  P  x  y  2 . P=2 x=y=1. Vậy 2 min P  x=y=1 1
  • 19. III 1. M(O) góc MND=900 DNMN (1) M(O') góc MNP=900  PNMN (2) Từ (1) và (2) D, P, N thẳng hàng 2. Góc DNA=DMA (3) (=1/2sđ cung AD) Góc PNQ=PMQ (4) (=1/2sđ cung PQ) Từ (3) và (4)  gócPNA=PNQNP là p giác góc ANQ(5) Góc MAN=MDN (6) (=1/2sđ cung MN) Tứ giác ADQP có góc PAD=DQP=900 nội tiếp được  góc PDQ=PAQ (7) Q A P N O B C M D O' Từ (6) và (7)  góc PAN= PAQAP là phân giác góc NAQ (8) Từ (5) và (8)  P là tâm đường tròn nội tiếp tam giác ANQ 1,5 1,5 IV Q 2 ab  a  b  c ( ab  1) = Ta có (  1)(  1)(  1)  a b c a b ab c ( 1).( 1) ( 1).( 1)      ( a  1).( b  1).( c  1) = 1  ab ( 1).( 1) 1 1     a b c  1 1 1 1  ab     a b   ab a b Do a  b  c  b 1 a 1 ; b  a 1  (a 1).(b 1)  0  ab  a  b 1 c 1 2 Suy ra ab  1 2( 1) 2 1     ab ab Q Đặt ab  x  x  2  1  x 2( 1) 2 1     x x Q  5 x x ( do x  2 )  (  2).(  5)  Q 0 12 x 5  1  2( 1) 2 1 12      x x 12(  1).(  2)  x x Q  5 12 Q  5 a=1; b=2; c=3. Vậy 12 5 min Q  a=1; b=2; c=3. 12 1
  • 20. WWW.VNMATH.COM x+ x +2012 y+ y +2012 2012 x + z - 4(y+z)+8 0 WWW.VNMATH.COM 1 SỞ GIÁO DỤC VÀ ĐÀO TẠO HẢI DƯƠNG KÌ THI TUYỂN SINH LỚP 10 TRƯỜNG THPT CHUYÊN NGUYỄN TRÃI NĂM HỌC 2012- 2013 Môn thi: TOÁN (chuyên) Thời gian làm bài: 150 phút Đề thi gồm : 01 trang Ngày thi 20 tháng 6 năm 2012 ĐỀ CHÍNH THỨC Câu I (2,0 điểm) 1) Phân tích đa thức sau thành nhân tử a2 (b-2c)+b2 (c-a)+2c2 (a-b)+abc . 2) Cho x, y thỏa mãn x  3 y- y2+1+3 y+ y2+1 . Tính giá trị của biểu thức A  x4+x3y+3x2+xy- 2y2+1. Câu II ( 2,0 điểm) 1) Giải phương trình (x2 - 4x+11)(x4 - 8x2+21)  35 . 2) Giải hệ phương trình  2  2  2 2    . Câu III (2,0 điểm) 1) Chứng minh rằng với mọi số nguyên n thì (n2 + n + 1) không chia hết cho 9. 2) Xét phương trình x2 – m2x + 2m + 2 = 0 (1) (ẩn x). Tìm các giá trị nguyên dương của m để phương trình (1) có nghiệm nguyên. Câu IV (3,0 điểm) Cho tam giác ABC vuông tại A có AB AC ngoại tiếp đường tròn tâm O. Gọi D, E, F lần lượt là tiếp điểm của (O) với các cạnh AB, AC, BC; BO cắt EF tại I. M là điểm di chuyển trên đoạn CE. 1) Tính BIF. 2) Gọi H là giao điểm của BM và EF. Chứng minh rằng nếu AM = AB thì tứ giác ABHI nội tiếp. 3) Gọi N là giao điểm của BM với cung nhỏ EF của (O), P và Q lần lượt là hình chiếu của N trên các đường thẳng DE, DF. Xác định vị trí của điểm M để PQ lớn nhất. Câu V (1,0 điểm)
  • 21. WWW.VNMATH.COM Cho 3 số a, b, c thỏa mãn 0  a  b  c  1. Tìm giá trị lớn nhất của biểu thức B (a+b+c+3) 1 + 1 + 1        .                      x x       x x ( 2) 7 7 WWW.VNMATH.COM 2 a+1 b+1 c+1 ----------------------------Hết---------------------------- Họ và tên thí sinh………………………………. Số báo danh………………...……………… Chữ kí của giám thị 1: ……………………… Chữ kí của giám thị 2: …………………… SỞ GIÁO DỤC VÀ ĐÀO TẠO HẢI DƯƠNG KÌ THI TUYỂN SINH LỚP 10 TRƯỜNG THPT CHUYÊN NGUYỄN TRÃI NĂM HỌC 2012 - 2013 HƯỚNG DẪN VÀ BIỂU ĐIỂM CHẤM MÔN TOÁN (chuyên) Hướng dẫn chấm gồm : 03 trang I) HƯỚNG DẪN CHUNG. - Thí sinh làm bài theo cách riêng nhưng đáp ứng được yêu cầu cơ bản vẫn cho đủ điểm. - Việc chi tiết điểm số (nếu có) so với biểu điểm phải được thống nhất trong Hội đồng chấm. - Sau khi cộng điểm toàn bài, điểm lẻ đến 0,25 điểm. II) ĐÁP ÁN VÀ BIỂU ĐIỂM CHẤM. Câu Nội dung Điểm Câu I (2,0đ) a2 (b - 2c) +b2 (c - a) + 2c2 (a - b) + abc=2c2 (a - b)+ab(a-b)-c(a2 b2 )  ac(a b) 0,25  (a  b)[2c2  2ac  ab bc] 0,25  (a  b)[2c(c  a)  b(a  c)] 0,25 1) 1,0 điểm  (a  b)(a  c)(b  2c) 0,25 Có x = 3 y- y2+ 1  3 y+ y2+ 1  x3 = 2y +33 y - y2 + 1 . 3 y+ y2+ 1  3 y- y2+1  3 y+ y2+1    0,25  x3 + 3x -2y = 0 0,25 A = x4 + x3y + 3x2 - 2xy + 3xy - 2y2 + 1 = (x4+3x2 -2xy) +(x3y+3xy - 2y2 ) 1 0,25 2) 1,0 điểm  x(x3+3x-2y) +y(x3+3x - 2y) 1 1 0,25 Câu II (1,0đ) phương trình đã cho tương đương với (x  2)2  7 (x2  4)2  5  35(1) 0,25 ( 2) 2 7 7 Do 2 2 2 2 2 ( 2) 7 ( 4) 5 35 ( 4) 5 5 x x x x x 0,25 2 2 2 (1) ( 4) 5 5     0,25 1)1,0 điểm =x=2 0,25
  • 22. WWW.VNMATH.COM  (x+ x +2012)(y+ y +2012)  2012 (1)  x + z - 4(y+z)+8=0 (2) (1)x  x2  2012 y  y2  2012  y2  2012  y 2012 y2  2012  y (Do y2  2012  y  0y )  2   2   2 2 x x 2012 2012 2012 y 2012 y x x 2012 y 2012 y x y y x                2   2  2012 2012 2012 2012 2012 2012 y x y x       y  x y   y  x   x x y x y ( ) 0       y x y x                       y y y y  ( x  2)  0  x   2             x  x  m   x x  m  1 2 x  x  m  * WWW.VNMATH.COM 3 2)1,0 điểm 2 2 2 2  2 2  2 2  2 2 2012 2012 x y y x       2 2 2 2 2012 2012 2 2 2 2 2012 2012 2012 2012       0,25 Do 2 2 2 2 2012 | | 2012 2012 0 2012 | | y y x x y x x x x x 0,25 Thay y=-x vào(2) x2  z2  4x  4z 8  0(x  2)2  (z  2)2  0 0,25 2 2 2 ( 2) 0 2 y x z z Vậy hệ có nghiệm (x;y;z)=(-2;2;2). 0,25 Câu III (2,0đ) Đặt A = n2 + n + 1 do n n = 3k; n = 3k + 1; n = 3k + 2 (k  ) 0,25 * n = 3k = A không chia hết cho 9 (vì A không chia hết cho 3) 0,25 * n = 3k + 1 = A = 9k2 + 9k + 3 không chia hết cho 9. 0,25 1)1,0 điểm * n = 3k +2 = A = 9k2 +9k+7 không chia hết cho 9 Vậy với mọi số nguyên n thì A = n2 + n + 1 không chia hết cho 9. 0,25 Gi¶ sö tån t¹i m * ®Ó ph­ ¬ng tr×nh cã nghiÖm x1, x2 Theo vi-et: 2 1 2 1 2 2 2  (x1 - 1) (x2 - 1) = - m2 + 2m + 3 0,25 Với m* . Ta cã x1x2 1vμ x1 + x2  4 mà x1hoÆc x2 nguyªn vμ 2 * 1 2 1 2  x , x  (x 1)(x 1)  0 m2  2m 2  0(m1)(m3)  0 m  3m{1;2;3} 0,25 Víi m = 1; m = 2 thay vμo ta thÊy ph­ ¬ng tr×nh ®· cho v« nghiÖm. 0,25 2)1,0 điểm Víi m = 3 thay vμo ph­ ¬ng tr×nh ta ®­îc nghiÖm cña ph­ ¬ng tr×nh ®· cho lμ x =1; x = 8 tho¶ m·n. VËy m= 3 0,25 Câu IV (2,0đ) 1) 1,0 điểm Vẽ hình đúng theo yêu cầu chung của đề 0,25
  • 23. WWW.VNMATH.COM I WWW.VNMATH.COM 4 M H K A C E B O D F Gọi K là giao điểm của BO với DF = ΔIKF vuông tại K 0,25 Có DFE= 1 DOE=450 2 0,25 BIF  450 0,25 Khi AM = AB thì ΔABM vuông cân tại A = DBH=450 .Có DFH=450 0,25 = Tứ giác BDHF nội tiếp = 5 điểm B, D, O, H, F cùng thuộc một đường tròn. 0,25 = BFO=BHO  900 = OH  BM, mà OA  BM = A, O, H thẳng hàng 0,25 2) 1,0 điểm BAH=BIH  450 = Tứ giác ABHI nội tiếp. 0,25 3) 1,0 điểm P Q N C B A O D E F M Có tứ giác PNQD nội tiếp = QPN=QDN=EFN . Tương tự có NQP=NDP=FEN = ΔNEF và ΔNQP đồng dạng 0,25
  • 24. WWW.VNMATH.COM   )=3+3  x  x  y  y  z  z                                                               x y x y x y x y x y z y z y z y z z z y z y z y z y z y x y x y x y x x x y z y x z x x y y z z x z y z y x z x y z x z x y z x x z t t t t t t z x t t t t              Do (2 t  1)( t  1 t  2  2) t  5     WWW.VNMATH.COM 5 = PQ = NQ 1 PQ EF EF NE    0,25 Dấu “=” xẩy ra khi và chỉ khi P  F; Q E = DN là đường kính của (O) = PQ lớn nhất bằng EF. 0,25 Cách xác định điểm M : Kẻ đường kính DN của (O), BN cắt AC tại M thì PQ lớn nhất. 0,25 Đặt x=1+c, y=1+b, z=1+a do 0  a  b  c 1 = 1  z  y  x  2 Khi đó A= (x+y+z)( 1 1 1 x y z y z x z x y 0,25 1 1 0 1 . 0 1 . 1 1 0 1 . 0 1 . 2 2 2   0,25 Đặt x z = t =1 t  2 1 2 1 2 2 5 2 5 (2 1)( 2) 5 2 2 2 2 2  0  x z z x 2  A 3 2. 5 2 10 2 0,25 Câu V (1,0đ) Ta thấy khi a=b=0 và c=1 thì A=10 nên giá trị lớn nhất của A là 10 0,25 WWW.VNMATH.COM
  • 25. WWW.VNMATH.COM x x      x x y         . Tìm giá trị lớn nhất của biểu thức 1 1 2 2   WWW.VNMATH.COM 6 SỞ GIÁO DỤC VÀ ĐÀO TẠO HẢI DƯƠNG KÌ THI TUYỂN SINH LỚP 10 THPT CHUYÊN NGUYỄN TRÃI NĂM HỌC 2012- 2013 Môn thi: TOÁN (không chuyên) Thời gian làm bài: 120 phút Ngày thi 19 tháng 6 năm 2012 Đề thi gồm : 01 trang Câu I (2,0 điểm) 1) Giải phương trình 1 1 3   . 3) Giải hệ phương trình 3 3 3 0 3 2 11    . Câu II ( 1,0 điểm) Rút gọn biểu thức P = 1 + 1 : a + 1 2 a - a 2 - a a - 2 a với a 0 và a  4 . Câu III (1,0 điểm) Một tam giác vuông có chu vi là 30 cm, độ dài hai cạnh góc vuông hơn kém nhau 7cm. Tính độ dài các cạnh của tam giác vuông đó. Câu IV (2,0 điểm) Trong mặt phẳng tọa độ Oxy, cho đường thẳng (d): y = 2x -m+1 và parabol (P): y = 1 x2 2 . 1) Tìm m để đường thẳng (d) đi qua điểm A(-1; 3). 2) Tìm m để (d) cắt (P) tại hai điểm phân biệt có tọa độ (x1; y1) và (x2; y2) sao cho   1 2 1 2 x x y + y  48  0 . Câu V (3,0 điểm) Cho đường tròn tâm O đường kính AB. Trên đường tròn lấy điểm C sao cho AC BC (C  A). Các tiếp tuyến tại B và C của (O) cắt nhau ở điểm D, AD cắt (O) tại E (E  A) . 1) Chứng minh BE2 = AE.DE. 2) Qua C kẻ đường thẳng song song với BD cắt AB tại H, DO cắt BC tại F. Chứng minh tứ giác CHOF nội tiếp . 3) Gọi I là giao điểm của AD và CH. Chứng minh I là trung điểm của CH. Câu VI ( 1,0 điểm) Cho 2 số dương a, b thỏa mãn 1 1 2 a b 4 2 2 4 2 2 Q a  b  ab b  a  ba . ĐỀ CHÍNH THỨC
  • 26. WWW.VNMATH.COM x x x x           P= 1 + 1 : a +1 WWW.VNMATH.COM 7 ----------------------------Hết---------------------------- Họ và tên thí sinh……………………………………. Số báo danh………………...………… Chữ kí của giám thị 1: ……………………….……… Chữ kí của giám thị 2: ………………… SỞ GIÁO DỤC VÀ ĐÀO TẠO HẢI DƯƠNG KÌ THI TUYỂN SINH LỚP 10 THPT CHUYÊN NGUYỄN TRÃI NĂM HỌC 2012 - 2013 HƯỚNG DẪN VÀ BIỂU ĐIỂM CHẤM MÔN TOÁN (không chuyên) Hướng dẫn chấm gồm : 02 trang I) HƯỚNG DẪN CHUNG. - Thí sinh làm bài theo cách riêng nhưng đáp ứng được yêu cầu cơ bản vẫn cho đủ điểm. - Việc chi tiết điểm số (nếu có) so với biểu điểm phải được thống nhất trong Hội đồng chấm. - Sau khi cộng điểm toàn bài, điểm lẻ đến 0,25 điểm. II) ĐÁP ÁN VÀ BIỂU ĐIỂM CHẤM. Câu Nội dung Điểm Câu I (2,0đ) 1   1   1  3(  1) 3 0,25  x 1  3x  3 0,25   2x  4 0,25 1) 1,0 điểm  x  2.Vậy phương trình đã cho có một nghiệm x = -2 0,25     x x y 3 3 3 0(1) 3 2 11 (2)    Từ (1)= x 3  3 3 0,25 =x=3 0,25 Thay x=3 vào (2)=3.3 2y 11 =2y=2 0,25 2) 1,0 điểm =y=1 . Vậy hệ phương trình đã cho có nghiệm (x;y)=(3;1) 0,25   a 2- a 2- a a 2 a 0,25 a a = 1+ a 2 a   a (2  ) a +1 0,25     a a 2 =  a 2- a 0,25 Câu II (1,0đ)  = a 2 2- a =-1 0,25 Câu III (1,0đ) Gọi độ dài cạnh góc vuông nhỏ là x (cm) (điều kiện 0 x 15) = độ dài cạnh góc vuông còn lại là (x + 7 )(cm) 0,25
  • 27. WWW.VNMATH.COM Vì chu vi của tam giác là 30cm nên độ dài cạnh huyền là 30–(x + x +7)= 23–2x (cm) Theo định lí Py –ta- go ta có phương trình x2 + (x + 7)2 = (23 - 2x)2 0,25 x2 - 53x + 240 = 0 (1) Giải phương trình (1) được nghiệm x = 5; x = 48 0,25 Đối chiếu với điều kiện có x = 5 (TM đk); x = 48 (không TM đk) Vậy độ dài một cạnh góc vuông là 5cm, độ dài cạnh góc vuông còn lại là 12 cm, độ dài cạnh huyền là 30 – (5 + 12) = 13cm WWW.VNMATH.COM 8 0,25 Câu IV (2,0đ) Vì (d) đi qua điểm A(-1; 3) nên thay x = -1 và y = 3 vào hàm số y = 2x – m + 1 ta có 2.(-1) – m +1 = 3 0,25  -1 – m = 3 0,25  m = -4 0,25 1) 1,0 điểm Vậy m = -4 thì (d) đi qua điểm A(-1; 3) 0,25 Hoành độ giao điểm của (d) và (P) là nghiệm của phương trình 1 x2 2 1 2  x m 0,25 x2  4x  2m 20 (1) ; Để (d) cắt (P) tại hai điểm phân biệt nên (1) có hai nghiệm phân biệt ' 062m 0m  3 0,25 Vì (x1; y1) và (x2; y2) là tọa độ giao điểm của (d) và (P) nên x1; x2 là nghiệm của phương trình (1) và 1 1 y = 2x m1, 2 2 y = 2x m1 Theo hệ thức Vi-et ta có 1 2 1 2 x + x = 4, x x = 2m-2 .Thay y1,y2 vào   1 2 1 2 x x y +y  48  0 có   1 2 1 2 x x 2x +2x -2m+2  48  0 (2m - 2)(10 - 2m) + 48 = 0 0,25 2) 1,0 điểm  m2 - 6m - 7 = 0m=-1(thỏa mãn m3) hoặc m=7(không thỏa mãn m3) Vậy m = -1 thỏa mãn đề bài 0,25 Câu V (3,0đ) Vẽ đúng hình theo yêu cầu chung của đề bài 0,25 E D C A O B VìBD là tiếp tuyến của (O) nên BD  OB = ΔABD vuông tại B 0,25 Vì AB là đường kính của (O) nên AE  BE 0,25 1) 1,0 điểm Áp dụng hệ thức lượng trong ΔABD (ABD=900 ;BE  AD) ta có BE2 = AE.DE 0,25
  • 28. WWW.VNMATH.COM   1  1 (2) C    .      mà a  b  2 ab ab 1 2 Q  . Vậy giá trị lớn nhất của biểu thức là 1 WWW.VNMATH.COM 9 Có DB= DC (t/c hai tiếp tuyến cắt nhau), OB = OC (bán kính của (O)) = OD là đường trung trực của đoạn BC = OFC=900 (1) 0,25 Có CH // BD (gt), mà AB  BD (vì BD là tiếp tuyến của (O)) 0,25 = CH  AB = OHC=900 (2) 0,25 2) 1,0 điểm Từ (1) và (2) ta có OFC+ OHC = 1800 = tứ giác CHOF nội tiếp 0,25 Có CH //BD=HCB=CBD (hai góc ở vị trí so le trong) mà ΔBCD cân tại D = CBD  DCB nên CB là tia phân giác của HCD 0,25 do CA  CB = CA là tia phân giác góc ngoài đỉnh C của ΔICD AI = CI  (3) AD CD 0,25 Trong ΔABDcó HI // BD = AI = HI AD BD (4) 0,25 3)1,0 điểm Từ (3) và (4) = CI = HI CD BD mà CD=BDCI=HI I là trung điểm của CH 0,25 Với a  0;b  0 ta có: (a2 b)2  0 a4  2a2b  b2  0a4  b2  2a2b a4  b2  2ab2  2a2b  2ab2 1 1 (1) a 4  b 2  2ab 2 2ab  a  b  0,25 Tương tự có b 4  a 2  2a 2 b 2ab  a  b  Q 1 . Từ (1) và (2)   ab  a  b  0,25 Vì 1 1 2 a b 2ab a b 1 1 2( ) 2 Q ab 0,25 Câu VI (1,0đ) Khi a = b = 1 thì 1 2 2 0,25 E I F D A H O B
  • 29. ĐỀ CHÍNH THỨC A x x x       x x x x x  x    x  x  4 1 1          1 SỞ GIÁO DỤC VÀ ĐÀO TẠO HẢI PHÒNG KÌ THI TUYỂN SINH LỚP 10 THPT CHUYÊN NĂNG KHIẾU TRẦN PHÚ NĂM HỌC 2012- 2013 Môn thi: TOÁN (chuyên)Thời gian làm bài: 150 phút Ngày thi 25 tháng 6 năm 2012 Đề thi gồm : 01 trang Câu I (2,0 điểm) 1) Cho 15 11 3 2 2 3 2 3 1 3     Rút gọn và tìm giá trị lớn nhất của A 2) Cho phương trình x2  ax  b  0 có hai nghiệm nguyên dương biết a,b là hai số dương thỏa mãn 5a + b = 22.Tìm hai nghiệm đó. Câu II ( 2,0 điểm) 1) Giải phương trình:4 2 6 1 3 16 4 4 2 1 3 2) Giải hệ phương trình: 2 2 2 4 x x y y y xy a 4b 9c Câu III (1,0 điểm) Cho ba số dương a,b,c .Chứng minh rằng:    4 b  c c  a a  b Câu IV (2,0 điểm) Cho tam giác ABC ( AB AC) có trực tâm H, nội tiếp đường tròn tâm O, đường kính AA’.Gọi AD là đường phân giác trong của góc BAC (DBC).M,I lần lượt là trung điểm của BC và AH. 1) Lấy K đối xứng với H qua AD.Chứng minh K thuộc đường thẳng AA’. 2) Gọi P là giao điểm của AD với HM.Đường thẳng HK cắt AB và AC lần lượt tại Q và R.Chứng minh rằng Q và R lần lượt là hình chiếu vuông góc của P lên AB,AC. Câu V (3,0 điểm) 1) Tìm nghiệm nguyên của phương trình x4  y4  z4  2012 2) Cho hình vuông 12x12, được chia thành lưới các hình vuông đơn vị. Mỗi đỉnh của hình vuông đơn vị này được tô bằng một trong hai màu xanh đỏ. Có tất cả 111 đỉnh màu đỏ. Hai trong số những đỉnh màu đỏ này nằm ở đỉnh hình vuông lớn, 22 đỉnh màu đỏ khác nằm trên cạnh cạnh của hình vuông lớn (không trùng với đỉnh của hình vuông lớn ) hình vuông đơn vị được tô màu theo các quy luật sau: cạnh có hai đầu mút màu đỏ được tô màu đỏ, cạnh có hai đầu mút màu xanh được tô màu xanh, cạnh có một đầu mút màu đỏ và một đầu mút màu xanh thì được tô màu vàng. Giả sứ có tất cả 66 cạnh vàng. Hỏi có bao nhiêu cạnh màu xanh. ----------------------------Hết---------------------------- Họ và tên thí sinh……………………………………. Số báo danh………………...………… Chữ kí của giám thị 1: ……………………….……… Chữ kí của giám thị 2: ………………… Từ :Nguyễn Hồng Vân – THPT Trần Hưng Đạo – Hải Phòng- http://trakhuc66.violet.vn/
  • 30. A x x x x x         x  x   x  x    x  x  x  x  Dễ thấy 4x2  2x 1 3x2  (x 1)2  0,x4x2  2x 1 3x2  (x 1)2  0,x nên đặt a  4x2  2x 1,b  4x2  2x 1  b,a  0,b  0 Ta có phương trình 2 2 2 3             a b x x x a x x TM b 3 2 4 2 1 1 1 3 4 2 1 3 2 ,( ) 3 2 Lời giải một số câu Câu I 1) 15 11 3 2 2 3 A x x x       x x x x 2 3 1 3     15 11 (3 2)( 3) (2 3)( 1) x x ( 1)( 3)     5 17 3 A x      , A lớn nhất  x  0 khi đó A lớn nhất bằng 2 3 . 2) Gọi x1, x2 là hai nghiệm nguyên dương của phương trình (x1 x2) Ta có a = –x1 – x2 và b = x1x2 nên 5(–x1 – x2) + x1x2 = 22  x1(x2 – 5) – 5(x2 – 5) = 47  (x1 – 5)(x2 – 5) = 47 (*) Vì 1 1 x Z x 1 nên với giả sử 1 2 x  x Ta có: –4 ≤ x1 – 5 x2 – 5 nên (*)  1 x 5 1 x 5 47    2    x 6 x 52      1 2 . Khi đó: a = – 58 và b = 312 thoả 5a + b = 22. Vậy hai nghiệm cần tìm là x1 = 6; x2 = 52. Câu II: 1) 4 x 2  6 x  1   3 16 x 4  4 x 2  1 3 2(4 2 2 1) (4 2 2 1) 3 (4 2 2 1)(4 2 2 1) 3 a  b   ab 3 6a2  3ab  3b2  0  6(a)2  3(a)  3  0 b b 2 2      2)Giải hệ phương trình
  • 31.         Nếu y = 0 thì (2) vô lí nên y  0 vậy      2   và (1 ;2)       b c a c a b b  c a  c a  b            b c a c a b b c a c a b 3 4 2 1 1 (1) 2 2 4 (2) x x y y y xy (2) 1 1 x 4 y y Đặt 1 b y  ta có hệ  4 x 2  x  b  1 (1')  4 b 2  b  x  1 (2') Lấy ( 1’) – ( 2’) ta có (x-b) (2x+2b-1) = 0 1 *) Nếu x = b  ta có hai nghiệm ( , 2) 2 2 *) Nếu 2x + 2b = 1 thì hệ vô nghiệm Vậy hệ có hai nghiệm ( 1 , 2)   và (1 ;2) 2 2 Câu V 1) Giả sử một số nguyên là số chẵn có dạng 2k thì (2k)4 16k4  0(mod8) Nếu Số nguyên là số nguyên lẻ có dạng 2k + 1 thì (2k 1)4  (4t 1)2 16h 11(mod8) nên với k ,t,h là các số nguyên x, y, zZ  x4  y4  z4  0,1,2,3(mod8) Nhưng 2012  4 ( mod 8) Vậy phương trình đã cho không có nghiệm nguyên. 2) Có 111 đỉnh màu đỏ,trong đó có 22 đỉnh nằm trên cạnh của hình vuông,, 87 đỉnh nằm lọt trong hình vuông lớn.Từ đó ta thấy có hai điểm màu xanh ở hai góc của hỉnh vuông lớn, 22 điểm màu xanh trên các cạnh của hình vuông lớn không nằm trên đỉnh của hình vuông lớn còn lại có 34 điểm màu xanh nằm lọt trong hình vuông.Với 312 cạnh của cả hình, ta cho đình của mỗi cạnh như sau: trong 2 mút của nó có i điểm màu xanh thì cho i điểm.Gọi tổng số điểm là S, ta có S = 2 ( số cạnh màu xanh) + số cạnh vàng.Ta lại có thể đếm số S theo cách khác:Mỗi điểm xanh ở góc là mút của hai đoạn, các điểm còn lại là mút của 4 đoạn.Vậy S = 2 x 2 + 22 x 3+ 34 x 4 = 206, suy ra số cạnh xanh là : ( 206 – 66):2 = 70 cạnh màu xanh. Câu III: Chứng minh rằng: a  4b  9c  4 b  c c  a a  b (a b c)( 1 4 9 ) 18 b  c a  c a  b Thật vậy: [( ) ( ) )]( 1 4 9 ) ( 4( ) 9( ) )2 36 ( ) ( )       (a b c)( 1 4 9 ) 18       b  c a  c a  b Điều phải chứng minh .
  • 32. Bài hình: 1) Tam giác ABA’ có: ABC A'BC  900,ABC  BAN A'BC  BAN 4 P R Q K M I H D o A A' B C Lại có   ' ' A AC A BC  ( cùng chắn cung ' A C ) nên  ' BAN  A AC Cũng có BAD  CAD BAD  BAN  CAD CAN  Mặt khác H đối xứng với K qua AD HAD  KAD , H thuộc AN nên K thuộc AA’ 2) Bạn tự giải nhé.
  • 33. 1 ĐẠI HỌC QUỐC GIA TH HCM ĐỀ THI TUYỂN SINH VÀO LỚP 10 NĂM 2012 TRƯỜNG LÊ HỒNG PHONG TP HCM MÔN : TOÁN ( Chuyên) Câu 1: Giải phương trình : 8x 1  46 10x  x3  5x2  4x 1 Câu 2: Cho đa thức f(x) = ax3 + bx2 + cx + d. với a là số nguyên dương, biết: f(5) – f(4) = 2012 . Chứng minh: f(7) – f(2) là hợp số. 2 2 2 A 14 a b c ab bc ca Câu 3: Cho ba số dương a; b và c thỏa a + b + c = 1. Tìm GTNN của :  2 2 2    a b b c c a       Câu 4:Cho tứ giác ABCD nội tiếp (O; R) có AC vuông góc BD tại H. Trên cạnh AB lấy điểm M sao cho: AM = 1/3 AB. Trên cạnh HC lấy trung điểm N. chứng minh MH vuông góc với DN. Câu 5: Cho đường tròn tâm O và đường tròn tâm I cắt nhau tại hai điểm A và B(O và I khác phía đối với A và B). IB cắt (O) tại E: OB cắt (I) tại F. Qua B vẽ MN // EF( M thuộc (O) và N thuộc (I). a) Chứng minh :Tứ giác OAIE nội tiếp ; b) Chứng minh :AE + AF = MN Câu 6: Trên mặt phẳng cho 2013 điểm tùy ý sao cho khi 3 điểm bất kỳ thì tồn tại 2 điểm mà khoảng cách giữa 2 điểm đó luôn bé hơn 1. Chứng minh rằng tồn tại một đường tròn có bán kính bằng 1 chứa ít nhất 1007 điểm( kể cả biên). …………………………………. Hết ………………………………….
  • 34. 2 GỢI Ý Câu 1: Giải phương trình : 8x 1  46 10x  x3  5x2  4x 1  Điều kiện : 1  x  46 8 10 3 2 3 2 x x x x x x x x x x 8  1  46  10    5  4  1  8  1  3  46  10  6    5  4  8  8 x  1  3  8 x  1  3   46  10 x  6  46  10 x  6                                2 2      2   1 4 8 8 1 3 46 10 6 8 1 10 1 1 4 8    8 1 3 46 10 6 1 0 1        8 10 4 8 2      8 1 3 46 10 6 x x x x x x x x x x x x x x x x x Từ (1) suy ra: x = 1 . Từ (2), ta có : x2 – 4x + 8 = (x – 2)2 + 4  4 với mọi x 46  10  0  46  10  6  6  10  10  5 46 10 6 6 3 x x x   10  suy ra :  8  10  8  5 46  10 x  6 8 x  1  3 46  10 x  6 8 x  1  3 3 Vậy : 10   8  2  4  8 46 10 6 8 1 3 x x x x     , với mọi x. Suy ra phương trình có nghiệm duy nhất : x = 1. Câu 2: Cho đa thức f(x) = ax3 + bx2 + cx + d. với a là số nguyên dương, biết: f(5) – f(4) = 2012 . Chứng minh: f(7) – f(2) là hợp số. Ta có : f(5) – f(4) = 2012  (125a + 25b + 5c + d) – ( 64a + 16b + 4c + d) = 2012  61a + 9b + c = 2012. f(7) – f(2) = (343a + 49b + 7c + d) – ( 8a + 4b + 2c + d) = 335a + 45b + 5c = 305a + 45b + 5c +30a = 5(61a + 9b + c) + 30a = 2012 + 30a = 2( 1006 + 15a) Vì a là số nguyên nên ta được : 2( 1006 + 15a) chia hết cho 2. Vậy f(7) – f(2) là hợp số 2 2 2 A 14 a b c ab bc ca Câu 3: Cho ba số dương a; b và c thỏa a + b + c = 1. Tìm GTNN của :  2 2 2    a b b c c a       Ta có : (a + b + c)2 = a2 + b2 + c2 + 2(ab + bc + ca)  1  a 2 b 2 c 2  2 ab bc ca       Ta có: a2 + b2 + c2 = (a + b + c) (a2 + b2 + c2) = a3 +b2a+ b3 + bc2 + c3 + ca2 + a2b + b2c + c2a. Áp dụng bất đẳng thức Cô – Si: a3 + b2a ≥ 2a2b ; b3 + bc2 ≥ 2b2c ; c3 + ca2 ≥ 2c2a , dấu “=” xảy ra khi a = b = c. suy ra: a2 + b2 + c2 = a3 +b2a+ b3 + bc2 + c3 + ca2 + a2b + b2c + c2a ≥ 3(a2b + b2c + c2a) suy ra:    2 2 2  ab bc ca ab bc ca a b c 1 3 3 3 3          2 2 2 2 2 2 2 2 2 2 2 2 2 2 2 a b b c c a a b c a b b c c a a b c 2 a b c               Đặt : t = a2 + b2 + c2, ta có : 3(a2 + b2 + c2) ≥ (a + b + c)2 = 1  t ≥ 1 3 , dấu “=” xảy ra khi a = b = c = 1 3 . t t t t t t  Ta được : A = 14 3 3 28 3 3 27 3 3         . t t t t 2 2 2 2 2 2 2 2
  • 35. 3 t t    dấu “=” xảy ra khi : t = 1 Áp dụng bất đẳng thức Cô – Si : 27 3 2 27 . 3 9 t t 2 2 2 2 3 . t       t     Mặt khác : 3 1 3 4 vì : 1 2 2 6 2 3 3   Suy ra: A 9 4 23    dấu “=” xảy ra khi : a2 + b2 + c2 = 1 3 3 3 và a = b = c suy ra: a = b = c = 1 3 . Vậy A đạt giá trị nhỏ nhất bằng 23 3 , khi a= b = c = 1 3 . Câu 4: Cho tứ giác ABCD nội tiếp (O; R) có AC vuông góc BD tại H. Trên cạnh AB lấy điểm M sao cho: AM = 1/3 AB. Trên cạnh HC lấy trung điểm N. chứng minh MH vuông góc với DN + Gọi E; F lần lượt là trung điểm của HB và MB, Suy ra: AM = MF = FB = 1/3 AB. + Gọi K và G lần lượt là giao điểm của MH với DN và AE. + Ta có:  AHB ~  DHC = AH : HB = DH : HC = AH : (2HE) = DH : ( 2HN)  AH : HE = DH : HN =  AHE ~  DHN = NDH  EAH + Ta có : EF là đường trung bình của tam giác HMB = HM // EF + Xét  AEF : AM = MF và MG // EF = AG = GE. + Xét  AEH: vuông tại H có G là trung điểm của AE, suy ra: AG = HG = EG =  AHG cân tại G = AHG  EAH + Ta có : KDH DHK  EAH DHK  AHG  DHK  900 , suy ra  DHK vuông tại K. Vậy MH vuông góc với DN.(đpcm) Câu 5: Cho đường tròn tâm O và đường tròn tâm I cắt nhau tại hai điểm A và B(O và I khác phía đối với A và B). IB cắt (O) tại E: OB cắt (I) tại F. Qua B vẽ MN // EF( M thuộc (O) và N thuộc (I). a) Chứng minh :Tứ giác OAIE nội tiếp ; b) Chứng minh :AE + AF = MN a) +  BOE cân tại O = OBE  OEB; +  BIF cân tại I = IBF  IFB; Do : OBE  IBFOEB  IFB , suy ra: tứ giác OIFE nội tiếp. + Do : AOI = BOI ( c – c – c) = OAI  OBI + Ta có : OAI OEI  OBI OBE 1800 , suy ra tứ giác AOEI nội tiếp Vậy 5 điểm O; A; I; E; F nằm trên cùng một đường tròn. Vậy Tứ giác OAIE nội tiếp được. b) + Xét đường tròn (O) : AMB FOI 1 Sd AB 2   A + Do : MN // EF ta được : BEF  MBE ( slt) + Do 5 điểm O; A; I; E; F nằm trên cùng một đường tròn, suy ra: BEF  FOI Suy ra: AMB  FOI  BEF  MBE suy ra: AM // EB. Vậy tứ giác MABE là hình thang và nội tiếp đường tròn (O) suy ra: MABE là hình thang cân = MB = AE. + Chứng minh tương tự ta được : NB = AF, suy ra: AE + AF = MB + NB = MN. ( đpcm). Câu 6: Trên mặt phẳng cho 2013 điểm tùy ý sao cho khi 3 điểm bất kỳ thì khoảng cách giữa hai điểm luôn bé hơn 1. Chứng minh rằng tồn tại một đường tròn có bán kính bằng 1 chứa ít nhất 1007 điểm( kể cả biên). Gọi các điểm là : A1; A2; A3; …; Ai; Ai + 1 ; A2012; A2013. Ta chia các cặp điểm như sau: (A1; A2013); ( A2; A2012); …( Ai; A2013 – i)…;(A1006; A1008) , và điểm A1007. O G K F E N H M D C B A O I F E N M B
  • 36. 4 Xét điểm A1007 với các cặp điểm đã cho, theo giả thiết trong mỗi cặp điểm tồn tại một điểm Am sao cho đoạn thẳng A1007Am có độ dài nhỏ hơn 1. Không mất tính tổng quát giả sử các điểm A1; A2; …; A1006 có khoảng cách đến điểm A1007 nhỏ hơn 1, suy ra các điểm A1; A2; …; A1006 nằm trong đường tròn tâm A1007 bán kính bằng 1. Vậy tồ tại đường tròn có bán kính bằng 1 chứa 1007 điểm trong 2013 điểm đã cho. (đpcm). Rất mong nhận được sự góp ý của các bạn. Giáo viên : Hà Gia Có – trường THCS Lý Thường Kiệt – Định Quán – Đồng Nai.
  • 37. TUYỂN TẬP ĐỀ TOÁN – VÀO 10 – CHUYÊN LAM SƠN – THANH HOÁ
  • 38. TUYỂN TẬP ĐỀ TOÁN – VÀO 10 – CHUYÊN LAM SƠN – THANH HOÁ
  • 39. TUYỂN TẬP ĐỀ TOÁN – VÀO 10 – CHUYÊN LAM SƠN – THANH HOÁ
  • 40. TUYỂN TẬP ĐỀ TOÁN – VÀO 10 – CHUYÊN LAM SƠN – THANH HOÁ
  • 41. TUYỂN TẬP ĐỀ TOÁN – VÀO 10 – CHUYÊN LAM SƠN – THANH HOÁ
  • 42. TUYỂN TẬP ĐỀ TOÁN – VÀO 10 – CHUYÊN LAM SƠN – THANH HOÁ Đề thi vào lớp 10 chuyên Lê Quý Đôn, TP. Đà Nẵng Số :411 - 9/2011
  • 43. TUYỂN TẬP ĐỀ TOÁN – VÀO 10 – CHUYÊN LAM SƠN – THANH HOÁ
  • 44. TUYỂN TẬP ĐỀ TOÁN – VÀO 10 – CHUYÊN LAM SƠN – THANH HOÁ
  • 45. TUYỂN TẬP ĐỀ TOÁN – VÀO 10 – CHUYÊN LAM SƠN – THANH HOÁ
  • 46. TUYỂN TẬP ĐỀ TOÁN – VÀO 10 – CHUYÊN LAM SƠN – THANH HOÁ
  • 47. TUYỂN TẬP ĐỀ TOÁN – VÀO 10 – CHUYÊN LAM SƠN – THANH HOÁ Lời giải Đề thi vào lớp 10 trường THPT chuyên ĐHSP Hà Nội Số :410 - 8/2011
  • 48. TUYỂN TẬP ĐỀ TOÁN – VÀO 10 – CHUYÊN LAM SƠN – THANH HOÁ
  • 49. TUYỂN TẬP ĐỀ TOÁN – VÀO 10 – CHUYÊN LAM SƠN – THANH HOÁ
  • 50. TUYỂN TẬP ĐỀ TOÁN – VÀO 10 – CHUYÊN LAM SƠN – THANH HOÁ
  • 51. TUYỂN TẬP ĐỀ TOÁN – VÀO 10 – CHUYÊN LAM SƠN – THANH HOÁ Đề thi vào lớp 10 chuyên ĐHSP Hà Nội và Giải đề thi vào lớp 10 PTNK, ĐHQG TP. HCM 2010 – 2011 Số :409 - 7/2011
  • 52. TUYỂN TẬP ĐỀ TOÁN – VÀO 10 – CHUYÊN LAM SƠN – THANH HOÁ
  • 53. TUYỂN TẬP ĐỀ TOÁN – VÀO 10 – CHUYÊN LAM SƠN – THANH HOÁ
  • 54. TUYỂN TẬP ĐỀ TOÁN – VÀO 10 – CHUYÊN LAM SƠN – THANH HOÁ
  • 55. TUYỂN TẬP ĐỀ TOÁN – VÀO 10 – CHUYÊN LAM SƠN – THANH HOÁ Đề thi vào lớp 10 trường PTNK, ĐHQG TP. HCM, năm học 2010 – 2011 Số :408 - 6/2011
  • 56. TUYỂN TẬP ĐỀ TOÁN – VÀO 10 – CHUYÊN LAM SƠN – THANH HOÁ
  • 57. TUYỂN TẬP ĐỀ TOÁN – VÀO 10 – CHUYÊN LAM SƠN – THANH HOÁ
  • 58. TUYỂN TẬP ĐỀ TOÁN – VÀO 10 – CHUYÊN LAM SƠN – THANH HOÁ
  • 59. TUYỂN TẬP ĐỀ TOÁN – VÀO 10 – CHUYÊN LAM SƠN – THANH HOÁ Đề thi vào lớp 10 chuyên Quang Trung, Bình Phước vàLời giải Đề thi vào lớp 10 chuyên Lê Quý Đôn, Bình Định, năm học 2010 – 2011 Số :407 - 5/2011
  • 60. TUYỂN TẬP ĐỀ TOÁN – VÀO 10 – CHUYÊN LAM SƠN – THANH HOÁ
  • 61. TUYỂN TẬP ĐỀ TOÁN – VÀO 10 – CHUYÊN LAM SƠN – THANH HOÁ
  • 62. TUYỂN TẬP ĐỀ TOÁN – VÀO 10 – CHUYÊN LAM SƠN – THANH HOÁ Đề thi vào lớp 10 chuyên Lê Quý Đôn, Bình Định và giải Đề thi vào lớp 10 chuyên Lê Khiết, Quảng Ngãi năm học 2010 – 2011 Số :406 - 4/2011
  • 63. TUYỂN TẬP ĐỀ TOÁN – VÀO 10 – CHUYÊN LAM SƠN – THANH HOÁ
  • 64. TUYỂN TẬP ĐỀ TOÁN – VÀO 10 – CHUYÊN LAM SƠN – THANH HOÁ
  • 65. TUYỂN TẬP ĐỀ TOÁN – VÀO 10 – CHUYÊN LAM SƠN – THANH HOÁ Đề thi vào lớp 10 THPT chuyên Lê Khiết, Quảng Ngãi và giải Đề thi vào lớp 10 THPT chuyên Lam Sơn, Thanh Hóa, năm học 2010 – 2011 Số :405 - 2/2011
  • 66. TUYỂN TẬP ĐỀ TOÁN – VÀO 10 – CHUYÊN LAM SƠN – THANH HOÁ
  • 67. TUYỂN TẬP ĐỀ TOÁN – VÀO 10 – CHUYÊN LAM SƠN – THANH HOÁ
  • 68. TUYỂN TẬP ĐỀ TOÁN – VÀO 10 – CHUYÊN LAM SƠN – THANH HOÁ Đề thi vào lớp 10 trường THPT chuyên Lam Sơn, Thanh Hóa vàHướng dẫn giải Đề thi vào lớp 10 trường THPT chuyên Phan Bội Châu, Nghệ An, năm học 2010 – 2011 Số :404 - 2/2011
  • 69. TUYỂN TẬP ĐỀ TOÁN – VÀO 10 – CHUYÊN LAM SƠN – THANH HOÁ
  • 70. TUYỂN TẬP ĐỀ TOÁN – VÀO 10 – CHUYÊN LAM SƠN – THANH HOÁ
  • 71. TUYỂN TẬP ĐỀ TOÁN – VÀO 10 – CHUYÊN LAM SƠN – THANH HOÁ Đề thi vào lớp 10 THPT chuyên Phan Bội Châu, NA và giải Đề thi chọn học sinh giỏi lớp 9 tỉnh Vĩnh Phúc - năm 2010 Số :403 - 1/2011
  • 72. TUYỂN TẬP ĐỀ TOÁN – VÀO 10 – CHUYÊN LAM SƠN – THANH HOÁ Đề thi chọn học sinh giỏi lớp 9 tỉnh Vĩnh Phúc năm 2010
  • 73. TUYỂN TẬP ĐỀ TOÁN – VÀO 10 – CHUYÊN LAM SƠN – THANH HOÁ Số :402 - 12/2010
  • 74. TUYỂN TẬP ĐỀ TOÁN – VÀO 10 – CHUYÊN LAM SƠN – THANH HOÁ
  • 75. TUYỂN TẬP ĐỀ TOÁN – VÀO 10 – CHUYÊN LAM SƠN – THANH HOÁ
  • 76. TUYỂN TẬP ĐỀ TOÁN – VÀO 10 – CHUYÊN LAM SƠN – THANH HOÁ Đề thi vào lớp 10 Khối THPT chuyên ĐH Vinh năm học 2010-2011 và Lời giải Đề HSG lớp 9 TP.HCM năm 2010 Số :398 - 8/2010
  • 77. TUYỂN TẬP ĐỀ TOÁN – VÀO 10 – CHUYÊN LAM SƠN – THANH HOÁ
  • 78. TUYỂN TẬP ĐỀ TOÁN – VÀO 10 – CHUYÊN LAM SƠN – THANH HOÁ
  • 79. TUYỂN TẬP ĐỀ TOÁN – VÀO 10 – CHUYÊN LAM SƠN – THANH HOÁ
  • 80. TUYỂN TẬP ĐỀ TOÁN – VÀO 10 – CHUYÊN LAM SƠN – THANH HOÁ Đề thi học sinh giỏi lớp 9 TP. HCM và Lời giải đề vào lớp 10 chuyên toán chuyên Lê Quý Đôn, Bình Định, năm học 2009 – 2010 Số :397 - 7/2010
  • 81. TUYỂN TẬP ĐỀ TOÁN – VÀO 10 – CHUYÊN LAM SƠN – THANH HOÁ
  • 82. TUYỂN TẬP ĐỀ TOÁN – VÀO 10 – CHUYÊN LAM SƠN – THANH HOÁ
  • 83. TUYỂN TẬP ĐỀ TOÁN – VÀO 10 – CHUYÊN LAM SƠN – THANH HOÁ Đề vào lớp 10 chuyên Lê Quý Đôn và Lời giả ivào lớp 10 chuyên Quang Trung Số :396 - 6/2010
  • 84. TUYỂN TẬP ĐỀ TOÁN – VÀO 10 – CHUYÊN LAM SƠN – THANH HOÁ
  • 85. TUYỂN TẬP ĐỀ TOÁN – VÀO 10 – CHUYÊN LAM SƠN – THANH HOÁ
  • 86. TUYỂN TẬP ĐỀ TOÁN – VÀO 10 – CHUYÊN LAM SƠN – THANH HOÁ Đề thi vào lớp 10 chuyên Toán Quang Trung, Bình Phước năm học 2009 - 2010 Số :395 - 5/2010
  • 87. TUYỂN TẬP ĐỀ TOÁN – VÀO 10 – CHUYÊN LAM SƠN – THANH HOÁ
  • 88. TUYỂN TẬP ĐỀ TOÁN – VÀO 10 – CHUYÊN LAM SƠN – THANH HOÁ Lời giải đề thi vào lớp 10 chuyên toán Phan Bội Châu, Nghệ An năm học 2009 - 2010 Số :394 - 4/2010
  • 89. TUYỂN TẬP ĐỀ TOÁN – VÀO 10 – CHUYÊN LAM SƠN – THANH HOÁ
  • 90. TUYỂN TẬP ĐỀ TOÁN – VÀO 10 – CHUYÊN LAM SƠN – THANH HOÁ
  • 91. TUYỂN TẬP ĐỀ TOÁN – VÀO 10 – CHUYÊN LAM SƠN – THANH HOÁ
  • 92. TUYỂN TẬP ĐỀ TOÁN – VÀO 10 – CHUYÊN LAM SƠN – THANH HOÁ Đề thi vào lớp 10 chuyên toán THPT chuyên Phan Bội Châu năm học 2009-2010 Số :393 - 3/2010
  • 93. TUYỂN TẬP ĐỀ TOÁN – VÀO 10 – CHUYÊN LAM SƠN – THANH HOÁ
  • 94. TUYỂN TẬP ĐỀ TOÁN – VÀO 10 – CHUYÊN LAM SƠN – THANH HOÁ
  • 95. TUYỂN TẬP ĐỀ TOÁN – VÀO 10 – CHUYÊN LAM SƠN – THANH HOÁ Đề thi vào lớp 10 chuyên toán Lê Hồng Phong, Nam Định và Lời giải Đề thi vào lớp 10 chuyên toán Vĩnh Phúc, năm học 2009 - 2010 Số :392 - 2/2010
  • 96. TUYỂN TẬP ĐỀ TOÁN – VÀO 10 – CHUYÊN LAM SƠN – THANH HOÁ
  • 97. TUYỂN TẬP ĐỀ TOÁN – VÀO 10 – CHUYÊN LAM SƠN – THANH HOÁ
  • 98. TUYỂN TẬP ĐỀ TOÁN – VÀO 10 – CHUYÊN LAM SƠN – THANH HOÁ
  • 99. TUYỂN TẬP ĐỀ TOÁN – VÀO 10 – CHUYÊN LAM SƠN – THANH HOÁ
  • 100. TUYỂN TẬP ĐỀ TOÁN – VÀO 10 – CHUYÊN LAM SƠN – THANH HOÁ
  • 101. TUYỂN TẬP ĐỀ TOÁN – VÀO 10 – CHUYÊN LAM SƠN – THANH HOÁ
  • 102. TUYỂN TẬP ĐỀ TOÁN – VÀO 10 – CHUYÊN LAM SƠN – THANH HOÁ Lời giải Đề thi vào lớp 10 chuyên toán Lam Sơn, Thanh Hóa và Đề thi vào lớp 10 chuyên toán Vĩnh Phúc, năm học 2009 – 2010 Số :391 - 1/2010
  • 103. TUYỂN TẬP ĐỀ TOÁN – VÀO 10 – CHUYÊN LAM SƠN – THANH HOÁ
  • 104. TUYỂN TẬP ĐỀ TOÁN – VÀO 10 – CHUYÊN LAM SƠN – THANH HOÁ
  • 105. TUYỂN TẬP ĐỀ TOÁN – VÀO 10 – CHUYÊN LAM SƠN – THANH HOÁ
  • 106. TUYỂN TẬP ĐỀ TOÁN – VÀO 10 – CHUYÊN LAM SƠN – THANH HOÁ Giải Đề thi vào lớp 10 chuyên Lê Khiết, Quảng Ngãi vàĐề thi vào lớp 10 chuyên Toán Lam Sơn, Thanh Hóa, năm học 2009 - 2010 Số :390 - 12/2009
  • 107. TUYỂN TẬP ĐỀ TOÁN – VÀO 10 – CHUYÊN LAM SƠN – THANH HOÁ
  • 108. TUYỂN TẬP ĐỀ TOÁN – VÀO 10 – CHUYÊN LAM SƠN – THANH HOÁ
  • 109. TUYỂN TẬP ĐỀ TOÁN – VÀO 10 – CHUYÊN LAM SƠN – THANH HOÁ THI HỌC SINH GIỎI VÀ TRƯỜNG CHUYÊN Số :389 - 11/2009
  • 110. TUYỂN TẬP ĐỀ TOÁN – VÀO 10 – CHUYÊN LAM SƠN – THANH HOÁ
  • 111. TUYỂN TẬP ĐỀ TOÁN – VÀO 10 – CHUYÊN LAM SƠN – THANH HOÁ THI HỌC SINH GIỎI VÀ TRƯỜNG CHUYÊN Số :389 - 11/2009
  • 112. TUYỂN TẬP ĐỀ TOÁN – VÀO 10 – CHUYÊN LAM SƠN – THANH HOÁ
  • 113. TUYỂN TẬP ĐỀ TOÁN – VÀO 10 – CHUYÊN LAM SƠN – THANH HOÁ CÁC KÌ THI HỌC SINH GIỎI VÀ TRƯỜNG CHUYÊN Số :388 - 10/2009
  • 114. TUYỂN TẬP ĐỀ TOÁN – VÀO 10 – CHUYÊN LAM SƠN – THANH HOÁ
  • 115. TUYỂN TẬP ĐỀ TOÁN – VÀO 10 – CHUYÊN LAM SƠN – THANH HOÁ
  • 116. TUYỂN TẬP ĐỀ TOÁN – VÀO 10 – CHUYÊN LAM SƠN – THANH HOÁ
  • 117. TUYỂN TẬP ĐỀ TOÁN – VÀO 10 – CHUYÊN LAM SƠN – THANH HOÁ
  • 118. TUYỂN TẬP ĐỀ TOÁN – VÀO 10 – CHUYÊN LAM SƠN – THANH HOÁ
  • 119. TUYỂN TẬP ĐỀ TOÁN – VÀO 10 – CHUYÊN LAM SƠN – THANH HOÁ
  • 120. TUYỂN TẬP ĐỀ TOÁN – VÀO 10 – CHUYÊN LAM SƠN – THANH HOÁ
  • 121. TUYỂN TẬP ĐỀ TOÁN – VÀO 10 – CHUYÊN LAM SƠN – THANH HOÁ CÁC KÌ THI HỌC SINH GIỎI VÀ TRƯỜNG CHUYÊN Số :387 - 9/2009
  • 122. TUYỂN TẬP ĐỀ TOÁN – VÀO 10 – CHUYÊN LAM SƠN – THANH HOÁ
  • 123. TUYỂN TẬP ĐỀ TOÁN – VÀO 10 – CHUYÊN LAM SƠN – THANH HOÁ
  • 124. TUYỂN TẬP ĐỀ TOÁN – VÀO 10 – CHUYÊN LAM SƠN – THANH HOÁ Lời giải đề thi vào Lớp 10 Chuyên Tóan Lam Sơn, Thanh Hóa và Đề thi vào Lớp 10 Chuyên Toán Trần Phú, Hải Phòng Số :386 - 8/2009
  • 125. TUYỂN TẬP ĐỀ TOÁN – VÀO 10 – CHUYÊN LAM SƠN – THANH HOÁ
  • 126. TUYỂN TẬP ĐỀ TOÁN – VÀO 10 – CHUYÊN LAM SƠN – THANH HOÁ
  • 127. TUYỂN TẬP ĐỀ TOÁN – VÀO 10 – CHUYÊN LAM SƠN – THANH HOÁ CÁC KÌ THI HỌC SINH GIỎI VÀ TRƯỜNG CHUYÊN Số :385 - 7/2009
  • 128. TUYỂN TẬP ĐỀ TOÁN – VÀO 10 – CHUYÊN LAM SƠN – THANH HOÁ
  • 129. TUYỂN TẬP ĐỀ TOÁN – VÀO 10 – CHUYÊN LAM SƠN – THANH HOÁ
  • 130. TUYỂN TẬP ĐỀ TOÁN – VÀO 10 – CHUYÊN LAM SƠN – THANH HOÁ Đề thi vào Lớp 10 Khối chuyên THPT ĐHSP Hà Nội năm học 2008-2009 Vòng 2 và Lời giải Vòng 1. Số :384 - 6/2009
  • 131. TUYỂN TẬP ĐỀ TOÁN – VÀO 10 – CHUYÊN LAM SƠN – THANH HOÁ
  • 132. TUYỂN TẬP ĐỀ TOÁN – VÀO 10 – CHUYÊN LAM SƠN – THANH HOÁ
  • 133. TUYỂN TẬP ĐỀ TOÁN – VÀO 10 – CHUYÊN LAM SƠN – THANH HOÁ
  • 134. TUYỂN TẬP ĐỀ TOÁN – VÀO 10 – CHUYÊN LAM SƠN – THANH HOÁ
  • 135. TUYỂN TẬP ĐỀ TOÁN – VÀO 10 – CHUYÊN LAM SƠN – THANH HOÁ
  • 136. TUYỂN TẬP ĐỀ TOÁN – VÀO 10 – CHUYÊN LAM SƠN – THANH HOÁ
  • 137. 1 Së gi¸o dôc vμ ®μo t¹o H­ng yªn ®Ò chÝnh thøc (§Ò thi cã 01 trang) kú thi tuyÓn sinh vμo líp 10 thpt chuyªn N¨m häc 2012 - 2013 M«n thi: To¸n (Dμnh cho thÝ sinh dù thi c¸c líp chuyªn: To¸n, Tin) Thêi gian lμm bμi: 150 phót Bài 1: (2 điểm) a) Cho A = 20122  20122.20132  20132 . Chứng minh A là một số tự nhiên. b) Giải hệ phương trình x 1 x 3   2    y 2 y x  1  x  3 y y  Bài 2: (2 điểm) a) Cho Parbol (P): y = x2 và đường thẳng (d): y = (m +2)x – m + 6. Tìm m để đường thẳng (d) cắt Parabol (P) tại hai điểm phân biệt có hoành độ dương. b) Giải phương trình: 5 + x + 2 (4  x)(2x  2)  4( 4  x  2x  2) Bài 3: (2 điểm) a) Tìm tất cả các số hữu tỷ x sao cho A = x2 + x+ 6 là một số chính phương. b) Cho x 1 và y 1. Chứng minh rằng : (x3  y3)  (x2  y2 ) 8 (x 1)(y 1)    Bài 4 (3 điểm) Cho tam giác ABC nhọn nội tiếp đường tròn tâm O, đường cao BE và CF. Tiếp tuyến tại B và C cắt nhau tại S, gọi BC và OS cắt nhau tại M a) Chứng minh AB. MB = AE.BS b) Hai tam giác AEM và ABS đồng dạng c) Gọi AM cắt EF tại N, AS cắt BC tại P. CMR NP vuông góc với BC Bài 5: (1 điểm) Trong một giải bóng đá có 12 đội tham dự, thi đấu vòng tròn một lượt (hai đội bất kỳ thi đấu với nhau đúng một trận). a) Chứng minh rằng sau 4 vòng đấu (mỗi đội thi đấu đúng 4 trận) luôn tìm được ba đội bóng đôi một chưa thi đấu với nhau. b) Khẳng định trên còn đúng không nếu các đội đã thi đấu 5 trận? HƯỚNG DẪN GIẢI Bài 1: (2 điểm)
  • 138. 2 a) Cho A = 20122  20122.20132  20132 Đặt 2012 = a, ta có 20122  20122.20132  20132  a2  a2 (a 1)2  (a 1)2  (a2  a 1)2  a2  a 1 b) Đặt x a y x 1 b y       Ta có x 1 x 3   2    y 2 y x  1  x  3 y y             2 x 1 x 3 y y x 1 x 3     y y nên                 b2 a 3 b2 b 6 0 b a 3 b a 3 a 6 a 1      v       b 3 b 2 Bài 2: a) ycbt tương đương với PT x2 = (m +2)x – m + 6 hay x2 - (m +2)x + m – 6 = 0 có hai nghiệm dương phân biệt. b) Đặt t = 4  x  2x  2 Bài 3: a) x = 0, x = 1, x= -1 không thỏa mãn. Với x khác các giá trị này, trước hết ta chứng minh x phải là số nguyên. +) x2 + x+ 6 là một số chính phương nên x2 + x phải là số nguyên. m +) Giả sử x  với m và n có ước nguyên lớn nhất là 1. n Ta có x2 + x = 2 2 2 2 m m m  mn n n n   là số nguyên khi m2 mn chia hết cho n2 nên m2 mn chia hết cho n, vì mn chia hết cho n nên m2 chia hết cho n và do m và n có ước nguyên lớn nhất là 1, suy ra m chia hết cho n( mâu thuẫn với m và n có ước nguyên lớn nhất là 1). Do đó x phải là số nguyên. Đặt x2 + x+ 6 = k2 Ta có 4x2 + 4x+ 24 = 4 k2 hay (2x+1)2 + 23 = 4 k2 tương đương với 4 k2 - (2x+1)2 = 23 (x3  y3)  (x2  y2 ) x2 (x  1)  y2 (y  1)  (x  1)(y  1) (x  1)(y  1) = x2  y2 y  1 x  1 (x  1)2  2(x  1)  1 (y  1)2  2(y  1)  1   y  1 x  1   2  2     (x 1) (y 1) 2(y 1)  2(x  1)   1       1  y  1 x  1   x  1 y  1     y  1 x  1   . Theo BĐT Côsi
  • 139. 3 (x  1)2 (y  2  1) 2 (x  1)2 . (y  1)2  2 (x  1)(y  1) y 1 x 1 y 1 x 1     2(y  1) 2(x   1)  2(y  1)  . 2(x 1)  4 x 1 y 1 x 1 y 1     1  1  2 1 . 1 y 1 x 1 y 1 x 1     2 1 . 1 (x 1)(y 1) 2.2 1 . 1 . (x 1)(y 1) 4            y  1 x  1   y  1 x  1 Nên có đpcm Bài 4 a) Suy ra từ hai tam giác đồng dạng là ABE và BSM b) Từ câu a) ta có AE MB  (1) AB BS Mà MB = EM( do tam giác BEC vuông tại E có M là trung điểm của BC Nên AE  EM AB BS Có MOB  BAE,EBA  BAE  900 ,MBOMOB  900 Nên MBO  EBA do đó MEB  OBA( MBE) Suy ra MEA  SBA(2) Từ (1) và (2) suy ra hai tam giác AEM và ABS đồng dạng(đpcm.) c) Dễ thấy SM vuông góc với BC nên để chứng minh bài toán ta chứng minh NP //SM. + Xét hai tam giác ANE và APB: Từ câu b) ta có hai tam giác AEM và ABS đồng dạng nên NAE  PAB, Mà AEN  ABP ( do tứ giác BCEF nội tiếp) AN AE Do đó hai tam giác ANE và APB đồng dạng nên  AP AB P N F E M S O A B C Q
  • 140. 4 Lại có AM AE  ( hai tam giác AEM và ABS đồng dạng) AS AB Suy ra AM AN  nên trong tam giác AMS có NP//SM( định lí Talet đảo) AS AP Do đó bài toán được chứng minh. Bài 5 a. Giả sử kết luận của bài toán là sai, tức là trong ba đội bất kỳ thì có hai đội đã đấu với nhau rồi. Giả sử đội đã gặp các đội 2, 3, 4, 5. Xét các bộ (1; 6; i) với i Є{7; 8; 9;…;12}, trong các bộ này phải có ít nhất một cặp đã đấu với nhau, tuy nhiên 1 không gặp 6 hay i nên 6 gặp i với mọi i Є{7; 8; 9;…;12} , vô lý vì đội 6 như thế đã đấu hơn 4 trận. Vậy có đpcm. b. Kết luận không đúng. Chia 12 đội thành 2 nhóm, mỗi nhóm 6 đội. Trong mỗi nhóm này, cho tất cả các đội đôi một đã thi đấu với nhau. Lúc này rõ ràng mỗi đội đã đấu 5 trận. Khi xét 3 đội bất kỳ, phải có 2 đội thuộc cùng một nhóm, do đó 2 đội này đã đấu với nhau. Ta có phản ví dụ. Có thể giải quyết đơn giản hơn cho câu a. như sau: Do mỗi đội đã đấu 4 trận nên tồn tại hai đội A, B chưa đấu với nhau. Trong các đội còn lại, vì A và B chỉ đấu 3 trận với họ nên tổng số trận của A, B với các đội này nhiều nhất là 6 và do đó, tồn tại đội C trong số các đội còn lại chưa đấu với cả A và B. Ta có A, B, C là bộ ba đội đôi một chưa đấu với nhau.
  • 141. SỞ GIÁO DỤC ĐÀO TẠO KỲ THI VÀO LỚP 10 CHUYÊN LÝ TỰ TRỌNG 1 NGHỆ AN NĂM HỌC 2012 - 2013 ĐỀ CHÍNH THỨC Môn thi : TOÁN (Đề gồm có 1 trang) (Dùng cho thí sinh thi vào lớp chuyên) Thời gian làm bài :150 phút (Không kể thời gian giao đề) Ngày thi : 18 tháng 6 năm 2012 Câu 1: (2.0 điểm )                         A x x x x Cho biểu thức : 2 3 2 : 2 x x x x x 5 6 2 3 1 1/ Rút gọn biểu thức A. 2/ Tìm các giá trị của x để 1   5 A 2 Câu 2 (2,0 điểm ) Trong mặt phẳng toạ độ Oxy, cho Parabol (P) : y = ax2 a  0và đường thẳng (d): y = bx + 1 1/ Tìm các giá trị của a và b để (P) và (d) cùng đi qua điểm M(1; 2) 2/ Với a, b vừa tìm được, chứng minh rằng (P) và (d) còn có một điểm chung N khác M. Tính diện tích tam giác MON (với O là gốc toạ độ) Câu 3 (2.0 điểm) 1/ Cho phương trình: x2 (2m1)x m2 m6  0 (m là tham số). Tìm m để phương trình có hai nghiệm dương phân biệt 2/ Giải hệ phương trình:      x y 1 1 2 1 1 1 x y     Câu 4 (3.0 điểm) : Cho A là điểm cố định nằm ngoài đường tròn (O). Từ A kẻ tiếp tuyến AP và AQ tới đường tròn (P và Q là các tiếp điểm). Đường thẳng đi qua O và vuông góc với OP cắt đường thẳng OQ tại M. 1/ Chứng minh rằng: MO = MA 2/ Lấy điểm N trên cung lớn PQ của đường tròn (O) sao cho tiếp tuyến với (O) tại N cắt các tia AP, AQ lần lượt tại B và C. Chứng minh rằng: a) AB  AC  BC không phụ thuộc vào vị trí của điểm N. b) Nếu tứ giác BCQP nội tiếp được trong một đường tròn thì PQ//BC Câu 5 (1.0 điểm) Cho x, y là các số thực dương thoả mãn : 1 2 2 x y   . Chứng minh rằng : 5x2  y  4xy  y2  3 ---------- Hết ---------- Họ tên thí sinh …………………………………………….. Số báo danh: ………………………… Chữ ký giám thị 1: ………………………………… Chữ ký giám thị 2: ……………………
  • 142. 2 Bài giải Câu 1: (2.0 điểm )                         A x x x x Cho biểu thức : 2 3 2 : 2 x x x x x 5 6 2 3 1 1/ Rút gọn biểu thức A.                         A x 2 x 3 x 2 : 2 x x x x x x 5 6 2 3 1 (ĐK: x  0, x  4, x  9 ) A = … = 1 4 x x   2/ Tìm các giá trị của x để 1   5 A 2 1 5 4 5 2 8 5 5 2 1 2 2 5 3 0 3 1 0 1 2 2 0 1 4 x x x A x x x x x x                           Kết hợp với ĐK  0 1 4  x  Câu 2 (2,0 điểm ) Trong mặt phẳng toạ độ Oxy, cho Parabol (P) : y = ax2 a  0và đường thẳng (d): y = bx + 1 1/ Tìm các giá trị của a và b để (P) và (d) cùng đi qua điểm M(1; 2) M (P)  …  a = 2  y = 2x2 M  (d)  …  b = 1  y = x + 1 2/ Với a, b vừa tìm được, chứng minh rằng (P) và (d) còn có một điểm chung N khác M. Tính diện tích tam giác MON (với O là gốc toạ độ) Xét pt hoành độ gđ: 2x2 = x + 1  2x2 - x - 1 = 0                      x y 1 2 1 1 M 1;2 ; N 1 ; 1 2 2 2 2 x y   1 2 ... 0,75 (dvv) MON thang S S S S       Câu 3 (2.0 điểm) 1/ Cho phương trình: x2 (2m1)x m2 m6  0(m là tham số). Tìm m để phương trình có hai nghiệm dương phân biệt? phương trình có hai nghiệm dương phân biệt
  • 143. 3        m                               0 25 0 3 . 0 6 0 2 2 a c m 2 m m m b 0 2 m 1 0 m 1 a 2 2/ Giải hệ phương trình:      x y 1 1 2 (1) 1 1 1 (2) x y     (ĐK: x  1; y  1) (2)  x + y = xy (3) Hai vế của (1) đều dương ta bình phương hai vế ta có:    x y x y x y xy x y 2 2 1 1 4 2 2 1 4                 Thay (3) vào ta có: x + y = 4 kết hợp với (3) có hệ: x+y=4 xy=4  Áp dụng hệ thức Vi Ét ta có x; y là hai nghiệm của pt: X2 - 4x + 4 = 0  x = 2; y = 2 Câu 4 (3.0 điểm) : Cho A là điểm cố định nằm ngoài đường tròn (O). Từ A kẻ tiếp tuyến AP và AQ tới đường tròn (P và Q là các tiếp điểm). Đường thẳng đi qua O và vuông góc với OP cắt đường thẳng OQ tại M. 1 1 1 2 1 1 C B M P O Q A N 1/ Chứng minh rằng: MO = MA A1 = O1 và A1 = A2  A2 = O1  MAO cân  MO = MA 2/ Lấy điểm N trên cung lớn PQ của đường tròn (O) sao cho tiếp tuyến với (O) tại N cắt các tia AP, AQ lần lượt tại B và C. Chứng minh rằng: a) AB  AC  BC không phụ thuộc vào vị trí của điểm N. Theo t/c hai tia tiếp tuyến ta có …  AB + AC - BC = … = 2.AP (không đổi) b) Nếu tứ giác BCQP nội tiếp được trong một đường tròn thì PQ//BC Nếu tứ giác BCQP nội tiếp được  P1 = C1
  • 144.   (1) 4 mà P1 = Q1  C1 = Q1  PQ//BC Câu 5 (1.0 điểm) Cho x, y là các số thực dương thoả mãn : 1 2 2 x y   . Chứng minh rằng : 5x2  y  4xy  y2  3 * Ta có: 2 2 x y xy y x xy y x y x y x y 5   4   3 4 2 4 2 2 3 0 2 3 0                2 2 * x y x 1 2 2 2 2 1 2 2 1 2 x y y x y x x 2 1            Vì : y 0 ; x 0  2x - 1 0  x 1/2 Thay y = … vào x2  y 3  0 Ta có: 3 2 x y x x x x x x 2 2 2 2       3  0    3  0  2 6 3  0 x x 2 1 2 1 Vì 2x - 1 0  (1)  2x3  x2  2x  6x  3  0 2x3  x2  4x  3  0 Mà 2x3  x2  4x 3 3 2 2    x x x x x x x x 2 2 3 3         1 2 2   3   x 1 2  2x  3   0 x  0 Vậy 2x  y2  x2  y  3  0 x  0; y  0
  • 145. 1 SỞ GIÁO DỤC VÀ ĐÀO TẠO PHÚ THỌ KÌ THI TUYỂN SINH VÀO LỚP 10 TRUNG HỌC PHỔ THÔNG CHUYÊN HÙNG VƯƠNG NĂM HỌC 2012-2013 Môn Toán (Dành cho thí sinh thi vào chuyên Toán) Thời gian làm bài :150 phút không kể thời gian giao đề Đề thi có 1 trang Câu 1 ( 2,0 điểm) Tính giá trị của biểu thức A  29  30 2  9  4 2  5 2 Câu 2 ( 2,0 điểm) Cho phương trình x2 +mx+1=0 ( m là tham số) a) Xác định các giá trị của m để phương trình có nghiệm b) Tim m để phương trình có 2 nghiệm x1 , x2 Thỏa mãn 2 2 1 2 2 2 2 1 x  x  7 x x Câu 3 ( 2,0 điểm) a) Giải hệ phương trình  x 2  xy  x  y    x   y  x  2 2 5 2 0 4 2 2 2 3 b)Giải phương trình x 1  x 16  x  4  x  9 Câu 4( 4 điểm) Cho đường tròn (O;R) có dây AB  R 2 , M là điểm chuyển động trên cung lớn AB sao cho tam giác MAB nhọn.Gọi H là trực tâm tam giác MAB, C,D lần lượt là giao điểm thứ 2 của AH và BH với đường tròn (O).Giải sử N là giao của BC và AD a) Tính số đo góc AOB, góc MCD b) Chứng minh CD là đường kính của đường tròn (O) và HN có độ dài không đổi c) Chứng minh HN luôn đi qua điểm cố định Câu 5 (1,0điểm) Cho x.y.z là các số không âm thỏa mãn 3 x  y  z  .Tìm giá trị nhỏ nhất 2 S  x3  y3  z3  x2 y2z2 ----------------Hết--------------------- ĐỀ CHÍNH THỨC
  • 146. Câu 1(1đ) tính A = 29  30 2  9  4 2  5 2 HD A  29  30 2  9  4 2  5 2  29  30 2  2 2 1  5 2  59  30 2  5 2  5 2  3  5 2 2 Câu 2(2đ) Cho phương trình x2 +mx +1=0 a)Xác định m để phương trình có nghiệm. 2 2 2 1 x  x 7 b) Tìm m để phương trình có nghiệm x1 ; x2 thỏa mãn 2 1 2 2 x x  HD a)Có =m2 -4 để pt có nghiệm thì 0 m2 -4 0           2 2 m m 2 2 2 1 x  x 7  9 b) Có 2 1 2 2 x x ( ) 2 2    x x x x (*) 1 2 1 2 2  2 1    x x 2 2 2  m    theo viet ta có x1 +x2 =-m ; x1x2 =1 = (*)  9 1            2 3 2            5 5 5 2 3 2 2 m m m m m Câu 3 (2đ) a) giải hệ pt    2 x y x x xy x y 2  2  5   2  0(1) 2 2 4   2  3(2) b) giải pt x 1  x 16  x  4  x  9 (*)   HD  a) Từ (1) ta được (2x-1)(x+y-2)=0   1 (3) 2  2 (4) x   y x Thay (3) vào (2) ta được y=1 hoặc y=-1 Thay (4) vào (2) ta được 5y2 -18y+17=0 ( vô nghiệm) Vậy hệ có 2 nghiệm x=1/2, y=1 hoặc x=1/2, y=-1 b) ĐK x  -1 (*)  2x+17+2 (x 1)(x 16) =2x+13+2 (x  4)(x  9)  2+ (x 1)(x 16) = (x  4)(x  9) 4+x2 +17x+16+4 (x 1)(x 16) =x2 +13x+36  (x 1)(x 16) =4-x (x  4)  x2 +17x+16=x2 +16-18x  25x=0 x=0 Vậy pt có nghiệm x=0, Câu 4 (4đ) Cho (O;R) có dây cung AB=R 2 cố định. Lấy M di động trên cung lớn AB sao cho tam giác AMB có 3 góc nhọn. Gọi H là trực tâm tam giác AMB và C;D lần lượt là giao điểm thứ 2 của các đường thẳng AH;BH với (O) Giả sử N là giao điểm của đường thẳng BC và DA. a) Tính số đo góc AOB và MCD
  • 147. 3 b) CMR : CD là đường kính của (O) và đoạn NH có độ dài không đổi. c) CMR : NH luôn đi qua 1 điểm cố định. HD Gọi K;L lần lượt là trân đương cao hạ từ B; A của tam giác ABM a) có OA2 + OB2 = 2R2 =AB2 = Tam giác OBA vuông tại O = góc AOB=900 có góc BMA=45 = BKM vuông cân tại K = góc DBM =45= gócDCM =45(1) L K O H D M C B P A N b) tương tự ta có ALM vuông cân tại L = gócLAM=45=gócCDM (2) Từ (1) và(2) = DCM vuông tại M = CD là đường kính của (O) NHB và DCB có góc BNH=gócBDC =NHB đồng dạng DCB (g-g)  NH/DC=HB/BC (3) Lại có HBC vuông tại C mà gócBCA=1/2gócAOB=45=HBC vuông cân tại B  BH=HC (4) Từ (3) và (4) = NH/DC=1 = NH=CD không đổi. c) Gọi P là trung điểm của NH  PB=PA=1/2NH (AHN và BHN vuôngtại A và B) Mà OB=OA=1/2CD  OB=OA=PA=PB ( vì CD=HN) Lại cố gócAOB=90  OBPA là hình vuông , mà B; O; A không đổi =P không đổi = PO=AB=R 2 không đỏi. Vậy NH luôn đi qua điêm P cố định Câu 5 (1đ) Cho x.y.z là các số không âm thỏa mãn 3 x  y  z  .Tìm giá trị nhỏ nhất 2 S= x3+y3+z3+x2y2z2
  • 148. 4 HD Áp dụng BĐT Bunhia cho 2 dãy Dãy 1 x x; y y; z z dãy 2 x; y; z Ta có ( x 2  y 2  z 2 )[(x x) 2  (y y ) 2  (z z ) 2 ]  (x 2  y 2  z 2 ) 2 3 ( x 3  y 3  z 3 )  ( x 2  y 2  z 2 )2  x 3  y 3  z 3  2 ( x 2  y 2  z 2 )2 (*) 2 3 Mặt khác 2 2 2 2 x  x  ( y  z )  x  ( x  y  z )( x  y  z )(1) y 2  y  x  z y  x  z z 2  z  y  x z  y  x ( )( )(2); ( )( )(3) Từ (1), (2), (3) ta có ( )( )( ) 3 2 3 2 3 2                        xyz x y z x z y y z x z x y       27 9  x y z  6  xy yz xz  8 xyz 8 2 9 27 3 3 (**)             xyz x y z x y z x y z   2 2 2 2 2 2 2 2 2 2 2 2 2           8 8 3   Mặt khác Bunhia cho x; y; z và 1;1;1; ta có 2 t x y z x y z 2 2 2 (   ) 3 (***)      3 4 Từ (*) , (**) , (***)ta có 2 2 2 2 2 S  t    t   t  t  t   t  t    t    t       2 2 3 2 9 7 9 1 3 11 2 3 25 3 8 3 3 9 4 64 9 4 64 6 4 8 64 64     ( ) 25 3 1 Min S  t   x  y  z  64 4 2 GV Trần Bình Trân THCS Phượng Lâu –Việt Trì - Phú Thọ mọi góp ý lời giải liên hệ gmail: tbtran1234@gmail.com số điện thoại: 0988280207
  • 149. SỞ GIÁO DỤC VÀ ĐÀO TẠO KỲ THI TUYỂN SINH LỚP 10 THPT CHUYÊN QUẢNG NAM Năm học: 2012-2013 Khóa thi: Ngày 4 tháng 7 năm 2012 Môn: TOÁN (Chuyên Toán) Thời gian làm bài: 150 phút (không kể thời gian giao đề) ĐỀ CHÍNH THỨC Câu 1: (1,5 điểm) a) Rút gọn biểu thức: A = a  a  6  1 4  a a  2 (với a ≥ 0 và a ≠ 4). b) Cho x  28  16 3 3  1 . Tính giá trị của biểu thức: P  (x2  2x 1)2012 . Câu 2: (2,0 điểm) a) Giải phương trình: 3(1 x)  3  x  2 . b) Giải hệ phương trình: 2 2       x xy 4x 6 y xy 1     Câu 3: (1,5 điểm) Cho parabol (P): y = − x2 và đường thẳng (d): y = (3 − m)x + 2 − 2m (m là tham số). a) Chứng minh rằng với m ≠ −1 thì (d) luôn cắt (P) tại 2 điểm phân biệt A, B. b) Gọi yA, yB lần lượt là tung độ các điểm A, B. Tìm m để |yA − yB| = 2. Câu 4: (4,0 điểm) Cho hình chữ nhật ABCD có AB = 4 cm, AD = 2 cm. Đường thẳng vuông góc với AC tại C cắt các đường thẳng AB và AD lần lượt tại E và F. a) Chứng minh tứ giác EBDF nội tiếp trong đường tròn. b) Gọi I là giao điểm của các đường thẳng BD và EF. Tính độ dài đoạn thẳng ID. c) M là điểm thay đổi trên cạnh AB (M khác A, M khác B), đường thẳng CM cắt đường thẳng AD tại N. Gọi S1 là diện tích tam giác CME, S2 là diện tích tam giác AMN. Xác định vị trí điểm M để S 1  3S 2 . 2 Câu 5: (1,0 điểm) Cho a, b là hai số thực không âm thỏa: a + b ≤ 2. 2  Chứng minh: a  1  2b  8 1  a 1  2b 7 . --------------- Hết --------------- Họ và tên thí sinh: ......................................................... Số báo danh: ...................................